Programs & Examples On #Owl

The Web Ontology Language (OWL) is a vocabulary for creating ontologies, i.e. definitions of classes, properties and the relationships between them. Please do not use this tag in place of 'owl-carousel'!

Comparing a variable with a string python not working when redirecting from bash script

When you read() the file, you may get a newline character '\n' in your string. Try either

if UserInput.strip() == 'List contents': 

or

if 'List contents' in UserInput: 

Also note that your second file open could also use with:

with open('/Users/.../USER_INPUT.txt', 'w+') as UserInputFile:     if UserInput.strip() == 'List contents': # or if s in f:         UserInputFile.write("ls")     else:         print "Didn't work" 

500 Error on AppHarbor but downloaded build works on my machine

Just a wild guess: (not much to go on) but I have had similar problems when, for example, I was using the IIS rewrite module on my local machine (and it worked fine), but when I uploaded to a host that did not have that add-on module installed, I would get a 500 error with very little to go on - sounds similar. It drove me crazy trying to find it.

So make sure whatever options/addons that you might have and be using locally in IIS are also installed on the host.

Similarly, make sure you understand everything that is being referenced/used in your web.config - that is likely the problem area.

Xcode couldn't find any provisioning profiles matching

I opened XCode -> Preferences -> Accounts and clicked on Download certificate. That fixed my problem

Angular - ng: command not found

Try uninstalling the angular cli installed

npm uninstall -g angular-cli npm uninstall -g @angular/cli

The clean the node cache npm cache clean

Then npm install -g @angular/cli@latest

set Path the path C:\Users\admin\AppData\Roaming\npm\node_modules@angular\cli

If condition inside of map() React

This one I found simple solutions:

row = myArray.map((cell, i) => {

    if (i == myArray.length - 1) {
      return <div> Test Data 1</div>;
    }
    return <div> Test Data 2</div>;
  });

Prevent content from expanding grid items

The previous answer is pretty good, but I also wanted to mention that there is a fixed layout equivalent for grids, you just need to write minmax(0, 1fr) instead of 1fr as your track size.

Waiting for Target Device to Come Online

Make sure you have enabled USB debugging on your device


Depending on the version of Android you're using, proceed as follows:

  • On Android 8.0 and higher, go to Settings > System > About phone and tap Build number seven times.
  • On Android 4.2 through 7.1.2, go to Settings > About phone and tap Build number seven times.

Return to the main Settings menu to find Developer options at the bottom. In the Developer options menu, scroll down and enable USB debugging.

How to declare a Fixed length Array in TypeScript

Actually, You can achieve this with current typescript:

type Grow<T, A extends Array<T>> = ((x: T, ...xs: A) => void) extends ((...a: infer X) => void) ? X : never;
type GrowToSize<T, A extends Array<T>, N extends number> = { 0: A, 1: GrowToSize<T, Grow<T, A>, N> }[A['length'] extends N ? 0 : 1];

export type FixedArray<T, N extends number> = GrowToSize<T, [], N>;

Examples:

// OK
const fixedArr3: FixedArray<string, 3> = ['a', 'b', 'c'];

// Error:
// Type '[string, string, string]' is not assignable to type '[string, string]'.
//   Types of property 'length' are incompatible.
//     Type '3' is not assignable to type '2'.ts(2322)
const fixedArr2: FixedArray<string, 2> = ['a', 'b', 'c'];

// Error:
// Property '3' is missing in type '[string, string, string]' but required in type 
// '[string, string, string, string]'.ts(2741)
const fixedArr4: FixedArray<string, 4> = ['a', 'b', 'c'];

EDIT (after a long time)

This should handle bigger sizes (as basically it grows array exponentially until we get to closest power of two):

type Shift<A extends Array<any>> = ((...args: A) => void) extends ((...args: [A[0], ...infer R]) => void) ? R : never;

type GrowExpRev<A extends Array<any>, N extends number, P extends Array<Array<any>>> = A['length'] extends N ? A : {
  0: GrowExpRev<[...A, ...P[0]], N, P>,
  1: GrowExpRev<A, N, Shift<P>>
}[[...A, ...P[0]][N] extends undefined ? 0 : 1];

type GrowExp<A extends Array<any>, N extends number, P extends Array<Array<any>>> = A['length'] extends N ? A : {
  0: GrowExp<[...A, ...A], N, [A, ...P]>,
  1: GrowExpRev<A, N, P>
}[[...A, ...A][N] extends undefined ? 0 : 1];

export type FixedSizeArray<T, N extends number> = N extends 0 ? [] : N extends 1 ? [T] : GrowExp<[T, T], N, [[T]]>;

Error: Cannot invoke an expression whose type lacks a call signature

I think what you want is:

abstract class Component {
  public deps: any = {};
  public props: any = {};

  public makePropSetter<T>(prop: string): (val: T) => T {
    return function(val) {
      this.props[prop] = val
      return val
    }
  }
}

class Post extends Component {
  public toggleBody: (val: boolean) => boolean;

  constructor () {
    super()
    this.toggleBody = this.makePropSetter<boolean>('showFullBody')
  }

  showMore (): boolean {
    return this.toggleBody(true)
  }

  showLess (): boolean {
    return this.toggleBody(false)
  }
}

The important change is in setProp (i.e., makePropSetter in the new code). What you're really doing there is to say: this is a function, which provided with a property name, will return a function which allows you to change that property.

The <T> on makePropSetter allows you to lock that function in to a specific type. The <boolean> in the subclass's constructor is actually optional. Since you're assigning to toggleBody, and that already has the type fully specified, the TS compiler will be able to work it out on its own.

Then, in your subclass, you call that function, and the return type is now properly understood to be a function with a specific signature. Naturally, you'll need to have toggleBody respect that same signature.

Angular2 RC5: Can't bind to 'Property X' since it isn't a known property of 'Child Component'

<create-report-card-form [currentReportCardCount]="providerData.reportCards.length" ...
                         ^^^^^^^^^^^^^^^^^^^^^^^^

In your HomeComponent template, you are trying to bind to an input on the CreateReportCardForm component that doesn't exist.

In CreateReportCardForm, these are your only three inputs:

@Input() public reportCardDataSourcesItems: SelectItem[];
@Input() public reportCardYearItems: SelectItem[];
@Input() errorMessages: Message[];

Add one for currentReportCardCount and you should be good to go.

Restart pods when configmap updates in Kubernetes?

Another way is to stick it into the command section of the Deployment:

...
command: [ "echo", "
  option = value\n
  other_option = value\n
" ]
...

Alternatively, to make it more ConfigMap-like, use an additional Deployment that will just host that config in the command section and execute kubectl create on it while adding an unique 'version' to its name (like calculating a hash of the content) and modifying all the deployments that use that config:

...
command: [ "/usr/sbin/kubectl-apply-config.sh", "
  option = value\n
  other_option = value\n
" ]
...

I'll probably post kubectl-apply-config.sh if it ends up working.

(don't do that; it looks too bad)

setInterval in a React app

Manage setInterval with React Hooks:

  const [seconds, setSeconds] = useState(0)

  const interval = useRef(null)

  useEffect(() => { if (seconds === 60) stopCounter() }, [seconds])

  const startCounter = () => interval.current = setInterval(() => {
    setSeconds(prevState => prevState + 1)
  }, 1000)

  const stopCounter = () => clearInterval(interval.current)

Raw_Input() Is Not Defined

For Python 3.x, use input(). For Python 2.x, use raw_input(). Don't forget you can add a prompt string in your input() call to create one less print statement. input("GUESS THAT NUMBER!").

NPM vs. Bower vs. Browserify vs. Gulp vs. Grunt vs. Webpack

You can find some technical comparison on npmcompare

Comparing browserify vs. grunt vs. gulp vs. webpack

As you can see webpack is very well maintained with a new version coming out every 4 days on average. But Gulp seems to have the biggest community of them all (with over 20K stars on Github) Grunt seems a bit neglected (compared to the others)

So if need to choose one over the other i would go with Gulp

How to create JSON object Node.js

The other answers are helpful, but the JSON in your question isn't valid. I have formatted it to make it clearer below, note the missing single quote on line 24.

  1 {
  2     'Orientation Sensor':
  3     [
  4         {
  5             sampleTime: '1450632410296',
  6             data: '76.36731:3.4651554:0.5665419'
  7         },
  8         {
  9             sampleTime: '1450632410296',
 10             data: '78.15431:0.5247617:-0.20050584'
 11         }
 12     ],
 13     'Screen Orientation Sensor':
 14     [
 15         {
 16             sampleTime: '1450632410296',
 17             data: '255.0:-1.0:0.0'
 18         }
 19     ],
 20     'MPU6500 Gyroscope sensor UnCalibrated':
 21     [
 22         {
 23             sampleTime: '1450632410296',
 24             data: '-0.05006743:-0.013848438:-0.0063915867
 25         },
 26         {
 27             sampleTime: '1450632410296',
 28             data: '-0.051132694:-0.0127831735:-0.003325345'
 29         }
 30     ]
 31 }

There are a lot of great articles on how to manipulate objects in Javascript (whether using Node JS or a browser). I suggest here is a good place to start: https://developer.mozilla.org/en-US/docs/Web/JavaScript/Guide/Working_with_Objects

Copy output of a JavaScript variable to the clipboard

When you need to copy a variable to the clipboard in the Chrome dev console, you can simply use the copy() command.

https://developers.google.com/web/tools/chrome-devtools/console/command-line-reference#copyobject

Connect to mysql in a docker container from the host

if you running docker under docker-machine?

execute to get ip:

docker-machine ip <machine>

returns the ip for the machine and try connect mysql:

mysql -h<docker-machine-ip>

How to send a POST request with BODY in swift

I don't like any of the other answers so far (except perhaps the one by SwiftDeveloper), because they either require you to deserialize your JSON, only for it to be serialized again, or care about the structure of the JSON itself.

The correct answer has been posted by afrodev in another question. You should go and upvote it.

Below is just my adaption, with some minor changes (primarily explicit UTF-8 charset).

let urlString = "https://example.org/some/api"
let json = "{\"What\":\"Ever\"}"

let url = URL(string: urlString)!
let jsonData = json.data(using: .utf8, allowLossyConversion: false)!

var request = URLRequest(url: url)
request.httpMethod = HTTPMethod.post.rawValue
request.setValue("application/json; charset=UTF-8", forHTTPHeaderField: "Content-Type")
request.httpBody = jsonData

Alamofire.request(request).responseJSON {
    (response) in

    print(response)
}

How to make a owl carousel with arrows instead of next previous

If you using latest Owl Carousel 2 version. You can replace the Navigation text by fontawesome icon. Code is below.

$('.your-class').owlCarousel({
        loop: true,
        items: 1, // Select Item Number
        autoplay:true,
        dots: false,
        nav: true,
        navText: ["<i class='fa fa-long-arrow-left'></i>","<i class='fa fa-long-arrow-right'></i>"],

    });

Owl Carousel, making custom navigation

You can use a JS and SCSS/Fontawesome combination for the Prev/Next buttons.

In your JS (this includes screenreader only/accessibility classes with Zurb Foundation):

$('.whatever-carousel').owlCarousel({
    ... ...
    navText: ["<span class='show-for-sr'>Previous</span>","<span class='show-for-sr'>Next</span>"]
    ... ...
})

In your SCSS this:

.owl-theme {

    .owl-nav {
        .owl-prev,
        .owl-next {
            font-family: FontAwesome;
            //border-radius: 50%;
            //padding: whatever-to-get-a-circle;
            transition: all, .2s, ease;
        }
        .owl-prev {
            &::before {
                content: "\f104";
            }
        }
        .owl-next {
            &::before {
                content: "\f105";
            }
        }
    }
}

For the FontAwesome font-family I happen to use the embed code in the document header:

<script src="//use.fontawesome.com/123456whatever.js"></script>

There are various ways to include FA, strokes/folks, but I find this is pretty fast and as I'm using webpack I can just about live with that 1 extra js server call.

And to update this - there's also this JS option for slightly more complex arrows, still with accessibility in mind:

$('.whatever-carousel').owlCarousel({
    navText: ["<span class=\"fa-stack fa-lg\" aria-hidden=\"true\"><span class=\"show-for-sr\">Previous</span><i class=\"fa fa-circle fa-stack-2x\"></i><i class=\"fa fa-chevron-left fa-stack-1x fa-inverse\" aria-hidden=\"true\"></i></span>","<span class=\"fa-stack fa-lg\" aria-hidden=\"true\"><span class=\"show-for-sr\">Next</span><i class=\"fa fa-circle fa-stack-2x\"></i><i class=\"fa fa-chevron-right fa-stack-1x fa-inverse\" aria-hidden=\"true\"></i></span>"]
})

Loads of escaping there, use single quotes instead if preferred.

And in the SCSS just comment out the ::before attrs:

.owl-prev {
        //&::before { content: "\f104"; }
    }
    .owl-next {
        //&::before { content: "\f105"; }
    }

Not an enclosing class error Android Studio

String user_email = email.getText().toString().trim();
firebaseAuth
    .createUserWithEmailAndPassword(user_email,user_password)
    .addOnCompleteListener(new OnCompleteListener<AuthResult>() {
        @Override
        public void onComplete(@NonNull Task<AuthResult> task) {
            if(task.isSuccessful()) {
                Toast.makeText(RegistraionActivity.this, "Registration sucessful", Toast.LENGTH_SHORT).show();
                startActivities(new Intent(RegistraionActivity.this,MainActivity.class));
            }else{
                Toast.makeText(RegistraionActivity.this, "Registration failed", Toast.LENGTH_SHORT).show();
            }
        }
    });

Correct way to set Bearer token with CURL

Replace:

$authorization = "Bearer 080042cad6356ad5dc0a720c18b53b8e53d4c274"

with:

$authorization = "Authorization: Bearer 080042cad6356ad5dc0a720c18b53b8e53d4c274";

to make it a valid and working Authorization header.

TLS 1.2 not working in cURL

TLS 1.1 and TLS 1.2 are supported since OpenSSL 1.0.1

Forcing TLS 1.1 and 1.2 are only supported since curl 7.34.0

You should consider an upgrade.

Android statusbar icons color

Yes it's possible to change it to gray (no custom colors) but this only works from API 23 and above you only need to add this in your values-v23/styles.xml

<item name="android:windowLightStatusBar">true</item>

enter image description here

R Markdown - changing font size and font type in html output

To change the font size, you don't need to know a lot of html for this. Open the html output with notepad ++. Control F search for "font-size". You should see a section with font sizes for the headers (h1, h2, h3,...).

Add the following somewhere in this section.

body {
  font-size: 16px;
}

The font size above is 16 pt font. You can change the number to whatever you want.

What exactly is the difference between Web API and REST API in MVC?

ASP.NET Web API is a framework that makes it easy to build HTTP services that reach a broad range of clients, including browsers and mobile devices. ASP.NET Web API is an ideal platform for building RESTful applications on the .NET Framework.

REST

RESTs sweet spot is when you are exposing a public API over the internet to handle CRUD operations on data. REST is focused on accessing named resources through a single consistent interface.

SOAP

SOAP brings it’s own protocol and focuses on exposing pieces of application logic (not data) as services. SOAP exposes operations. SOAP is focused on accessing named operations, each implement some business logic through different interfaces.

Though SOAP is commonly referred to as “web services” this is a misnomer. SOAP has very little if anything to do with the Web. REST provides true “Web services” based on URIs and HTTP.

Reference: http://spf13.com/post/soap-vs-rest

And finally: What they could be referring to is REST vs. RPC See this: http://encosia.com/rest-vs-rpc-in-asp-net-web-api-who-cares-it-does-both/

How to set cell spacing and UICollectionView - UICollectionViewFlowLayout size ratio?

Swift 4

let collectionViewLayout = collectionView.collectionViewLayout as? UICollectionViewFlowLayout
collectionViewLayout?.sectionInset = UIEdgeInsetsMake(0, 20, 0, 40) 
collectionViewLayout?.invalidateLayout()

UICollectionView - dynamic cell height?

Follow bolnad answer up to Step 4.

Then make it simpler by replacing all the other steps with:

func collectionView(_ collectionView: UICollectionView, layout collectionViewLayout: UICollectionViewLayout, sizeForItemAt indexPath: IndexPath) -> CGSize {

    // Configure your cell
    sizingNibNew.configureCell(data as! CustomCellData, delegate: self)

    // We use the full width minus insets
    let width = collectionView.frame.size.width - collectionView.sectionInset.left - collectionView.sectionInset.right

    // Constrain our cell to this width 
    let height = sizingNibNew.systemLayoutSizeFitting(CGSize(width: width, height: .infinity), withHorizontalFittingPriority: UILayoutPriorityRequired, verticalFittingPriority: UILayoutPriorityFittingSizeLevel).height

    return CGSize(width: width, height: height)
}

How to fix symbol lookup error: undefined symbol errors in a cluster environment

After two dozens of comments to understand the situation, it was found that the libhdf5.so.7 was actually a symlink (with several levels of indirection) to a file that was not shared between the queued processes and the interactive processes. This means even though the symlink itself lies on a shared filesystem, the contents of the file do not and as a result the process was seeing different versions of the library.

For future reference: other than checking LD_LIBRARY_PATH, it's always a good idea to check a library with nm -D to see if the symbols actually exist. In this case it was found that they do exist in interactive mode but not when run in the queue. A quick md5sum revealed that the files were actually different.

The client and server cannot communicate, because they do not possess a common algorithm - ASP.NET C# IIS TLS 1.0 / 1.1 / 1.2 - Win32Exception

There are several other posts about this now and they all point to enabling TLS 1.2. Anything less is unsafe.

You can do this in .NET 3.5 with a patch.
You can do this in .NET 4.0 and 4.5 with a single line of code

ServicePointManager.SecurityProtocol = SecurityProtocolType.Tls12; // .NET 4.5
ServicePointManager.SecurityProtocol = (SecurityProtocolType)3072; // .NET 4.0

In .NET 4.6, it automatically uses TLS 1.2.

See here for more details: .NET support for TLS

How to add a jar in External Libraries in android studio

The GUI based approach would be to add an additional module in your project.

  1. From the File menu select Project Structure and click on the green plus icon on the top left.
  2. The new Module dialog pops
  3. From the phone and tablet application group select the "Import JAR or AAR package" option and click next.
  4. Follow the steps to create a new module that contains your JAR file.
  5. Click on the entry that corresponds to your main project and select the dependencies tab.
  6. Add a dependency to the module that you created in step 4.

One final piece of advice. Make sure that the JAR file you include is build with at most JDK 1.7. Many problems relating to error message "com.android.dx.cf.iface.ParseException: bad class file magic (cafebabe) or version (0034.0000)" root straight to this :0.

What is difference between mutable and immutable String in java

What is difference between mutable and immutable String in java

immutable exist, mutable don't.

Change grid interval and specify tick labels in Matplotlib

There are several problems in your code.

First the big ones:

  1. You are creating a new figure and a new axes in every iteration of your loop ? put fig = plt.figure and ax = fig.add_subplot(1,1,1) outside of the loop.

  2. Don't use the Locators. Call the functions ax.set_xticks() and ax.grid() with the correct keywords.

  3. With plt.axes() you are creating a new axes again. Use ax.set_aspect('equal').

The minor things: You should not mix the MATLAB-like syntax like plt.axis() with the objective syntax. Use ax.set_xlim(a,b) and ax.set_ylim(a,b)

This should be a working minimal example:

import numpy as np
import matplotlib.pyplot as plt

fig = plt.figure()
ax = fig.add_subplot(1, 1, 1)

# Major ticks every 20, minor ticks every 5
major_ticks = np.arange(0, 101, 20)
minor_ticks = np.arange(0, 101, 5)

ax.set_xticks(major_ticks)
ax.set_xticks(minor_ticks, minor=True)
ax.set_yticks(major_ticks)
ax.set_yticks(minor_ticks, minor=True)

# And a corresponding grid
ax.grid(which='both')

# Or if you want different settings for the grids:
ax.grid(which='minor', alpha=0.2)
ax.grid(which='major', alpha=0.5)

plt.show()

Output is this:

result

Why is it that "No HTTP resource was found that matches the request URI" here?

I got the similiar issue, and resolved it by the following. The issue looks not related to the Route definition but definition of the parameters, just need to give it a default value.

----Code with issue: Message: "No HTTP resource was found that matches the request URI

    [HttpGet]
    [Route("students/list")]
    public StudentListResponse GetStudents(int? ClassId, int? GradeId)
    {
       ...
    }

----Code without issue.

    [HttpGet]
    [Route("students/list")]
    public StudentListResponse GetStudents(int? ClassId=null, int? GradeId=null)
    {
       ...
    }

self.tableView.reloadData() not working in Swift

Try it: tableView.reloadSections(IndexSet(integersIn: 0...0), with: .automatic) It helped me

Could not find method compile() for arguments Gradle

Hope Below steps will help

Add the dependency to your project-level build.gradle:

classpath 'com.google.gms:google-services:3.0.0'

Add the plugin to your app-level build.gradle:

apply plugin: 'com.google.gms.google-services'

app-level build.gradle:

dependencies {
        compile 'com.google.android.gms:play-services-auth:9.8.0'
}

Remove x-axis label/text in chart.js

UPDATE chart.js 2.1 and above

var chart = new Chart(ctx, {
    ...
    options:{
        scales:{
            xAxes: [{
                display: false //this will remove all the x-axis grid lines
            }]
        }
    }
});


var chart = new Chart(ctx, {
    ...
    options: {
        scales: {
            xAxes: [{
                ticks: {
                    display: false //this will remove only the label
                }
            }]
        }
    }
});

Reference: chart.js documentation

Old answer (written when the current version was 1.0 beta) just for reference below:

To avoid displaying labels in chart.js you have to set scaleShowLabels : false and also avoid to pass the labels:

<script>
    var options = {
        ...
        scaleShowLabels : false
    };
    var lineChartData = {
        //COMMENT THIS LINE TO AVOID DISPLAYING THE LABELS
        //labels : ["1","2","3","4","5","6","7"],
        ... 
    }
    ...
</script>

org.hibernate.hql.internal.ast.QuerySyntaxException: table is not mapped

Other persons that are using mapping classes for Hibernate, make sure that have addressed correctly to model package in sessionFactory bean declaration in the following part:

<property name="packagesToScan" value="com.mblog.model"></property>

Paging UICollectionView by cells, not screen

Horizontal Paging With Custom Page Width (Swift 4 & 5)

Many solutions presented here result in some weird behaviour that doesn't feel like properly implemented paging.


The solution presented in this tutorial, however, doesn't seem to have any issues. It just feels like a perfectly working paging algorithm. You can implement it in 5 simple steps:

  1. Add the following property to your type: private var indexOfCellBeforeDragging = 0
  2. Set the collectionView delegate like this: collectionView.delegate = self
  3. Add conformance to UICollectionViewDelegate via an extension: extension YourType: UICollectionViewDelegate { }
  4. Add the following method to the extension implementing the UICollectionViewDelegate conformance and set a value for pageWidth:

    func scrollViewWillBeginDragging(_ scrollView: UIScrollView) {
        let pageWidth = // The width your page should have (plus a possible margin)
        let proportionalOffset = collectionView.contentOffset.x / pageWidth
        indexOfCellBeforeDragging = Int(round(proportionalOffset))
    }
    
  5. Add the following method to the extension implementing the UICollectionViewDelegate conformance, set the same value for pageWidth (you may also store this value at a central place) and set a value for collectionViewItemCount:

    func scrollViewWillEndDragging(_ scrollView: UIScrollView, withVelocity velocity: CGPoint, targetContentOffset: UnsafeMutablePointer<CGPoint>) {
        // Stop scrolling
        targetContentOffset.pointee = scrollView.contentOffset
    
        // Calculate conditions
        let pageWidth = // The width your page should have (plus a possible margin)
        let collectionViewItemCount = // The number of items in this section
        let proportionalOffset = collectionView.contentOffset.x / pageWidth
        let indexOfMajorCell = Int(round(proportionalOffset))
        let swipeVelocityThreshold: CGFloat = 0.5
        let hasEnoughVelocityToSlideToTheNextCell = indexOfCellBeforeDragging + 1 < collectionViewItemCount && velocity.x > swipeVelocityThreshold
        let hasEnoughVelocityToSlideToThePreviousCell = indexOfCellBeforeDragging - 1 >= 0 && velocity.x < -swipeVelocityThreshold
        let majorCellIsTheCellBeforeDragging = indexOfMajorCell == indexOfCellBeforeDragging
        let didUseSwipeToSkipCell = majorCellIsTheCellBeforeDragging && (hasEnoughVelocityToSlideToTheNextCell || hasEnoughVelocityToSlideToThePreviousCell)
    
        if didUseSwipeToSkipCell {
            // Animate so that swipe is just continued
            let snapToIndex = indexOfCellBeforeDragging + (hasEnoughVelocityToSlideToTheNextCell ? 1 : -1)
            let toValue = pageWidth * CGFloat(snapToIndex)
            UIView.animate(
                withDuration: 0.3,
                delay: 0,
                usingSpringWithDamping: 1,
                initialSpringVelocity: velocity.x,
                options: .allowUserInteraction,
                animations: {
                    scrollView.contentOffset = CGPoint(x: toValue, y: 0)
                    scrollView.layoutIfNeeded()
                },
                completion: nil
            )
        } else {
            // Pop back (against velocity)
            let indexPath = IndexPath(row: indexOfMajorCell, section: 0)
            collectionView.scrollToItem(at: indexPath, at: .left, animated: true)
        }
    }
    

How to fix Hibernate LazyInitializationException: failed to lazily initialize a collection of roles, could not initialize proxy - no Session

First of all I'd like to say that all users who said about lazy and transactions were right. But in my case there was a slight difference in that I used result of @Transactional method in a test and that was outside real transaction so I got this lazy exception.

My service method:

@Transactional
User get(String uid) {};

My test code:

User user = userService.get("123");
user.getActors(); //org.hibernate.LazyInitializationException: failed to lazily initialize a collection of role

My solution to this was wrapping that code in another transaction like this:

List<Actor> actors = new ArrayList<>();
transactionTemplate.execute((status) 
 -> actors.addAll(userService.get("123").getActors()));

Owl Carousel Won't Autoplay

Your Javascript should be

<script>
$("#intro").owlCarousel({

// Most important owl features

//Autoplay
autoplay: false,
autoplayTimeout: 5000,
autoplayHoverPause: true
)}
</script>

What is difference between Implicit wait and Explicit wait in Selenium WebDriver?

Implicit wait --

Implicit waits are basically your way of telling WebDriver the latency that you want to see if specified web element is not present that WebDriver looking for. So in this case, you are telling WebDriver that it should wait 10 seconds in cases of specified element not available on the UI (DOM).

Explicit wait--

Explicit waits are intelligent waits that are confined to a particular web element. Using explicit waits you are basically telling WebDriver at the max it is to wait for X units of time before it gives up.

Extension exists but uuid_generate_v4 fails

The extension is available but not installed in this database.

CREATE EXTENSION IF NOT EXISTS "uuid-ossp";

@RequestParam in Spring MVC handling optional parameters

You need to give required = false for name and password request parameters as well. That's because, when you provide just the logout parameter, it actually expects for name and password as well as they are still mandatory.

It worked when you just gave name and password because logout wasn't a mandatory parameter thanks to required = false already given for logout.

Creating a REST API using PHP

In your example, it’s fine as it is: it’s simple and works. The only things I’d suggest are:

  1. validating the data POSTed
  2. make sure your API is sending the Content-Type header to tell the client to expect a JSON response:

    header('Content-Type: application/json');
    echo json_encode($response);
    

Other than that, an API is something that takes an input and provides an output. It’s possible to “over-engineer” things, in that you make things more complicated that need be.

If you wanted to go down the route of controllers and models, then read up on the MVC pattern and work out how your domain objects fit into it. Looking at the above example, I can see maybe a MathController with an add() action/method.

There are a few starting point projects for RESTful APIs on GitHub that are worth a look.

ERROR 1045 (28000): Access denied for user 'root'@'localhost' (using password: YES)

if the problem still exists try to force changing the pass

Stop MySQL Server (on Linux):

/etc/init.d/mysql stop

Stop MySQL Server (on Mac OS X):

mysql.server stop

Start mysqld_safe daemon with --skip-grant-tables

mysqld_safe --skip-grant-tables &
mysql -u root

Setup new MySQL root user password

use mysql;
update user set password=PASSWORD("NEW-ROOT-PASSWORD") where User='root';
flush privileges;
quit;

Stop MySQL Server (on Linux):

/etc/init.d/mysql stop

Stop MySQL Server (on Mac OS X):

mysql.server stop

Start MySQL server service and test to login by root:

mysql -u root -p

Posting raw image data as multipart/form-data in curl

In case anyone had the same problem: check this as @PravinS suggested. I used the exact same code as shown there and it worked for me perfectly.

This is the relevant part of the server code that helped:

if (isset($_POST['btnUpload']))
{
$url = "URL_PATH of upload.php"; // e.g. http://localhost/myuploader/upload.php // request URL
$filename = $_FILES['file']['name'];
$filedata = $_FILES['file']['tmp_name'];
$filesize = $_FILES['file']['size'];
if ($filedata != '')
{
    $headers = array("Content-Type:multipart/form-data"); // cURL headers for file uploading
    $postfields = array("filedata" => "@$filedata", "filename" => $filename);
    $ch = curl_init();
    $options = array(
        CURLOPT_URL => $url,
        CURLOPT_HEADER => true,
        CURLOPT_POST => 1,
        CURLOPT_HTTPHEADER => $headers,
        CURLOPT_POSTFIELDS => $postfields,
        CURLOPT_INFILESIZE => $filesize,
        CURLOPT_RETURNTRANSFER => true
    ); // cURL options
    curl_setopt_array($ch, $options);
    curl_exec($ch);
    if(!curl_errno($ch))
    {
        $info = curl_getinfo($ch);
        if ($info['http_code'] == 200)
            $errmsg = "File uploaded successfully";
    }
    else
    {
        $errmsg = curl_error($ch);
    }
    curl_close($ch);
}
else
{
    $errmsg = "Please select the file";
}
}

html form should look something like:

<form action="uploadpost.php" method="post" name="frmUpload" enctype="multipart/form-data">
<tr>
  <td>Upload</td>
  <td align="center">:</td>
  <td><input name="file" type="file" id="file"/></td>
</tr>
<tr>
  <td>&nbsp;</td>
  <td align="center">&nbsp;</td>
  <td><input name="btnUpload" type="submit" value="Upload" /></td>
</tr>

What are OLTP and OLAP. What is the difference between them?

OLTP: It stands for OnLine Transaction Processing and is used for managing current day to day data information.
OLAP: It stands for OnLine Analytical Processing and is used to maintain the past history of data and mainly used for data analysis, it can also be referred to as warehouse.

How to use a parameter in ExecStart command line?

Although systemd indeed does not provide way to pass command-line arguments for unit files, there are possibilities to write instances: http://0pointer.de/blog/projects/instances.html

For example: /lib/systemd/system/[email protected] looks something like this:

[Unit]
Description=Serial Getty on %I
BindTo=dev-%i.device
After=dev-%i.device systemd-user-sessions.service

[Service]
ExecStart=-/sbin/agetty -s %I 115200,38400,9600
Restart=always
RestartSec=0

So, you may start it like:

$ systemctl start [email protected]
$ systemctl start [email protected]

For systemd it will different instances:

$ systemctl status [email protected]
[email protected] - Getty on ttyUSB0
      Loaded: loaded (/lib/systemd/system/[email protected]; static)
      Active: active (running) since Mon, 26 Sep 2011 04:20:44 +0200; 2s ago
    Main PID: 5443 (agetty)
      CGroup: name=systemd:/system/[email protected]/ttyUSB0
          + 5443 /sbin/agetty -s ttyUSB0 115200,38400,9600

It also mean great possibility enable and disable it separately.

Off course it lack much power of command line parsing, but in common way it is used as some sort of config files selection. For example you may look at Fedora [email protected]: http://pkgs.fedoraproject.org/cgit/openvpn.git/tree/[email protected]

data.table vs dplyr: can one do something well the other can't or does poorly?

In direct response to the Question Title...

dplyr definitely does things that data.table can not.

Your point #3

dplyr abstracts (or will) potential DB interactions

is a direct answer to your own question but isn't elevated to a high enough level. dplyr is truly an extendable front-end to multiple data storage mechanisms where as data.table is an extension to a single one.

Look at dplyr as a back-end agnostic interface, with all of the targets using the same grammer, where you can extend the targets and handlers at will. data.table is, from the dplyr perspective, one of those targets.

You will never (I hope) see a day that data.table attempts to translate your queries to create SQL statements that operate with on-disk or networked data stores.

dplyr can possibly do things data.table will not or might not do as well.

Based on the design of working in-memory, data.table could have a much more difficult time extending itself into parallel processing of queries than dplyr.


In response to the in-body questions...

Usage

Are there analytical tasks that are a lot easier to code with one or the other package for people familiar with the packages (i.e. some combination of keystrokes required vs. required level of esotericism, where less of each is a good thing).

This may seem like a punt but the real answer is no. People familiar with tools seem to use the either the one most familiar to them or the one that is actually the right one for the job at hand. With that being said, sometimes you want to present a particular readability, sometimes a level of performance, and when you have need for a high enough level of both you may just need another tool to go along with what you already have to make clearer abstractions.

Performance

Are there analytical tasks that are performed substantially (i.e. more than 2x) more efficiently in one package vs. another.

Again, no. data.table excels at being efficient in everything it does where dplyr gets the burden of being limited in some respects to the underlying data store and registered handlers.

This means when you run into a performance issue with data.table you can be pretty sure it is in your query function and if it is actually a bottleneck with data.table then you've won yourself the joy of filing a report. This is also true when dplyr is using data.table as the back-end; you may see some overhead from dplyr but odds are it is your query.

When dplyr has performance issues with back-ends you can get around them by registering a function for hybrid evaluation or (in the case of databases) manipulating the generated query prior to execution.

Also see the accepted answer to when is plyr better than data.table?

Difference between Relative path and absolute path in javascript

Imagine you have a window open on http://www.foo.com/bar/page.html In all of them (HTML, Javascript and CSS):

opened_url = http://www.foo.com/bar/page.html
base_path = http://www.foo.com/bar/
home_path = http://www.foo.com/
/kitten.png = Home_path/kitten.png
kitten.png = Base_path/kitten.png

In HTML and Javascript, the base_path is based on the opened window. In big javascript projects you need a BASEPATH or root variable to store the base_path occasionally. (like this)

In CSS the opened url is the address of which your .css is stored or loaded, its not the same like javascript with current opened window in this case.

And for being more secure in absolute paths it is recommended to use // instead of http:// for possible future migrations to https://. In your own example, use it this way:

<img src="//www.foo.com/images/kitten.png">

SSH Key - Still asking for password and passphrase

Use ssh remote url provided by Github not https.

Cloudfront custom-origin distribution returns 502 "ERROR The request could not be satisfied." for some URLs

In my case, it was because we had an invalid ssl cert. The problem was on our staging box and we use our prod cert on that as well. It had worked for the past couple of years with this configuration, but all of a sudden we started getting this error. Strange.

If others are getting this error, check that the ssl certificate is valid. You can enable logging to s3 via the AWS CloudFront Distribution interface to aid debugging.

Also, you can refer to amazon's docs on the matter here: http://docs.aws.amazon.com/AmazonCloudFront/latest/DeveloperGuide/SecureConnections.html

Ansible playbook shell output

Expanding on what leucos said in his answer, you can also print information with Ansible's humble debug module:

- hosts: all
  gather_facts: no
  tasks:
    - shell: ps -eo pcpu,user,args | sort -r -k1 | head -n5
      register: ps

    # Print the shell task's stdout.
    - debug: msg={{ ps.stdout }}

    # Print all contents of the shell task's output.
    - debug: var=ps

Spring Boot: How can I set the logging level with application.properties?

In case of eclipse IDE and your project is maven, remember to clean and build the project to reflect the changes.

Loading scripts after page load?

http://jsfiddle.net/c725wcn9/2/embedded

You will need to inspect the DOM to check this works. Jquery is needed.

$(document).ready(function(){
   var el = document.createElement('script');
   el.type = 'application/ld+json';
   el.text = JSON.stringify({ "@context": "http://schema.org",  "@type": "Recipe", "name": "My recipe name" });

   document.querySelector('head').appendChild(el);
});

Show popup after page load

When the DOM is finished loading you can add your code in the $(document).ready() function.

Remove the onclick from here:

<input type="submit" name="submit" value="Submit" onClick="PopUp()" />

Try this:

$(document).ready(function(){
   setTimeout(function(){
      PopUp();
   },5000); // 5000 to load it after 5 seconds from page load
});

Replace all elements of Python NumPy Array that are greater than some value

Since you actually want a different array which is arr where arr < 255, and 255 otherwise, this can be done simply:

result = np.minimum(arr, 255)

More generally, for a lower and/or upper bound:

result = np.clip(arr, 0, 255)

If you just want to access the values over 255, or something more complicated, @mtitan8's answer is more general, but np.clip and np.minimum (or np.maximum) are nicer and much faster for your case:

In [292]: timeit np.minimum(a, 255)
100000 loops, best of 3: 19.6 µs per loop

In [293]: %%timeit
   .....: c = np.copy(a)
   .....: c[a>255] = 255
   .....: 
10000 loops, best of 3: 86.6 µs per loop

If you want to do it in-place (i.e., modify arr instead of creating result) you can use the out parameter of np.minimum:

np.minimum(arr, 255, out=arr)

or

np.clip(arr, 0, 255, arr)

(the out= name is optional since the arguments in the same order as the function's definition.)

For in-place modification, the boolean indexing speeds up a lot (without having to make and then modify the copy separately), but is still not as fast as minimum:

In [328]: %%timeit
   .....: a = np.random.randint(0, 300, (100,100))
   .....: np.minimum(a, 255, a)
   .....: 
100000 loops, best of 3: 303 µs per loop

In [329]: %%timeit
   .....: a = np.random.randint(0, 300, (100,100))
   .....: a[a>255] = 255
   .....: 
100000 loops, best of 3: 356 µs per loop

For comparison, if you wanted to restrict your values with a minimum as well as a maximum, without clip you would have to do this twice, with something like

np.minimum(a, 255, a)
np.maximum(a, 0, a)

or,

a[a>255] = 255
a[a<0] = 0

How can I specify my .keystore file with Spring Boot and Tomcat?

It turns out that there is a way to do this, although I'm not sure I've found the 'proper' way since this required hours of reading source code from multiple projects. In other words, this might be a lot of dumb work (but it works).

First, there is no way to get at the server.xml in the embedded Tomcat, either to augment it or replace it. This must be done programmatically.

Second, the 'require_https' setting doesn't help since you can't set cert info that way. It does set up forwarding from http to https, but it doesn't give you a way to make https work so the forwarding isnt helpful. However, use it with the stuff below, which does make https work.

To begin, you need to provide an EmbeddedServletContainerFactory as explained in the Embedded Servlet Container Support docs. The docs are for Java but the Groovy would look pretty much the same. Note that I haven't been able to get it to recognize the @Value annotation used in their example but its not needed. For groovy, simply put this in a new .groovy file and include that file on the command line when you launch spring boot.

Now, the instructions say that you can customize the TomcatEmbeddedServletContainerFactory class that you created in that code so that you can alter web.xml behavior, and this is true, but for our purposes its important to know that you can also use it to tailor server.xml behavior. Indeed, reading the source for the class and comparing it with the Embedded Tomcat docs, you see that this is the only place to do that. The interesting function is TomcatEmbeddedServletContainerFactory.addConnectorCustomizers(), which may not look like much from the Javadocs but actually gives you the Embedded Tomcat object to customize yourself. Simply pass your own implementation of TomcatConnectorCustomizer and set the things you want on the given Connector in the void customize(Connector con) function. Now, there are about a billion things you can do with the Connector and I couldn't find useful docs for it but the createConnector() function in this this guys personal Spring-embedded-Tomcat project is a very practical guide. My implementation ended up looking like this:

package com.deepdownstudios.server

import org.springframework.boot.context.embedded.tomcat.TomcatConnectorCustomizer
import org.springframework.boot.context.embedded.EmbeddedServletContainerFactory
import org.springframework.boot.context.embedded.tomcat.TomcatEmbeddedServletContainerFactory
import org.apache.catalina.connector.Connector;
import org.apache.coyote.http11.Http11NioProtocol;
import org.springframework.boot.*
import org.springframework.stereotype.*

@Configuration
class MyConfiguration {

@Bean
public EmbeddedServletContainerFactory servletContainer() {
final int port = 8443;
final String keystoreFile = "/path/to/keystore"
final String keystorePass = "keystore-password"
final String keystoreType = "pkcs12"
final String keystoreProvider = "SunJSSE"
final String keystoreAlias = "tomcat"

TomcatEmbeddedServletContainerFactory factory = 
        new TomcatEmbeddedServletContainerFactory(this.port);
factory.addConnectorCustomizers( new TomcatConnectorCustomizer() {
    void    customize(Connector con) {
        Http11NioProtocol proto = (Http11NioProtocol) con.getProtocolHandler();
            proto.setSSLEnabled(true);
        con.setScheme("https");
        con.setSecure(true);
        proto.setKeystoreFile(keystoreFile);
        proto.setKeystorePass(keystorePass);
        proto.setKeystoreType(keystoreType);
        proto.setProperty("keystoreProvider", keystoreProvider);
        proto.setKeyAlias(keystoreAlias);
    }
});
return factory;
}
}

The Autowiring will pick up this implementation an run with it. Once I fixed my busted keystore file (make sure you call keytool with -storetype pkcs12, not -storepass pkcs12 as reported elsewhere), this worked. Also, it would be far better to provide the parameters (port, password, etc) as configuration settings for testing and such... I'm sure its possible if you can get the @Value annotation to work with Groovy.

Find all matches in workbook using Excel VBA

Below code avoids creating infinite loop. Assume XYZ is the string which we are looking for in the workbook.

   Private Sub CommandButton1_Click()
   Dim Sh As Worksheet, myCounter
   Dim Loc As Range

   For Each Sh In ThisWorkbook.Worksheets
   With Sh.UsedRange
   Set Loc = .Cells.Find(What:="XYZ")

    If Not Loc Is Nothing Then

           MsgBox ("Value is found  in " & Sh.Name)
           myCounter = 1
            Set Loc = .FindNext(Loc)

    End If
End With
Next
If myCounter = 0 Then
MsgBox ("Value not present in this worrkbook")
End If

End Sub

What is a None value?

I love code examples (as well as fruit), so let me show you

apple = "apple"
print(apple)
>>> apple
apple = None
print(apple)
>>> None

None means nothing, it has no value.

None evaluates to False.

Stop form refreshing page on submit

Most people would prevent the form from submitting by calling the event.preventDefault() function.

Another means is to remove the onclick attribute of the button, and get the code in processForm() out into .submit(function() { as return false; causes the form to not submit. Also, make the formBlaSubmit() functions return Boolean based on validity, for use in processForm();

katsh's answer is the same, just easier to digest.

(By the way, I'm new to stackoverflow, give me guidance please. )

How do Python's any and all functions work?

list = [1,1,1,0]
print(any(list)) # will return True because there is  1 or True exists
print(all(list)) # will return False because there is a 0 or False exists
return all(a % i for i in range(3, int(a ** 0.5) + 1)) # when number is divisible it will return False else return True but the whole statement is False .

UICollectionView - Horizontal scroll, horizontal layout?

This code works well in Swift 3.1 and Xcode 8.3.2

override func viewDidLayoutSubviews() {
        super.viewDidLayoutSubviews()

        let layout = UICollectionViewFlowLayout()
        layout.scrollDirection = .horizontal
        self.collectionView.collectionViewLayout = layout
        self.collectionView!.contentInset = UIEdgeInsets(top: -10, left: 0, bottom:0, right: 0)

        if let layout = self.collectionView.collectionViewLayout as? UICollectionViewFlowLayout {
            layout.minimumInteritemSpacing = 0
            layout.minimumLineSpacing = 0
            layout.itemSize = CGSize(width: self.view.frame.size.width-40, height: self.collectionView.frame.size.height-10)
            layout.invalidateLayout()
        }

    }

How do I get bootstrap-datepicker to work with Bootstrap 3?

I also use Stefan Petre’s http://www.eyecon.ro/bootstrap-datepicker and it does not work with Bootstrap 3 without modification. Note that http://eternicode.github.io/bootstrap-datepicker/ is a fork of Stefan Petre's code.

You have to change your markup (the sample markup will not work) to use the new CSS and form grid layout in Bootstrap 3. Also, you have to modify some CSS and JavaScript in the actual bootstrap-datepicker implementation.

Here is my solution:

<div class="form-group row">
  <div class="col-xs-8">
    <label class="control-label">My Label</label>
    <div class="input-group date" id="dp3" data-date="12-02-2012" data-date-format="mm-dd-yyyy">
      <input class="form-control" type="text" readonly="" value="12-02-2012">
      <span class="input-group-addon"><i class="glyphicon glyphicon-calendar"></i></span>
    </div>
  </div>
</div>

CSS changes in datepicker.css on lines 176-177:

.input-group.date .input-group-addon i,
.input-group.date .input-group-addon i {

Javascript change in datepicker-bootstrap.js on line 34:

this.component = this.element.is('.date') ? this.element.find('.input-group-addon') : false;

UPDATE

Using the newer code from http://eternicode.github.io/bootstrap-datepicker/ the changes are as follows:

CSS changes in datepicker.css on lines 446-447:

.input-group.date .input-group-addon i,
.input-group.date .input-group-addon i {

Javascript change in datepicker-bootstrap.js on line 46:

 this.component = this.element.is('.date') ? this.element.find('.input-group-addon, .btn') : false;

Finally, the JavaScript to enable the datepicker (with some options):

 $(".input-group.date").datepicker({ autoclose: true, todayHighlight: true });

Tested with Bootstrap 3.0 and JQuery 1.9.1. Note that this fork is better to use than the other as it is more feature rich, has localization support and auto-positions the datepicker based on the control position and window size, avoiding the picker going off the screen which was a problem with the older version.

CMake output/build directory

There's little need to set all the variables you're setting. CMake sets them to reasonable defaults. You should definitely not modify CMAKE_BINARY_DIR or CMAKE_CACHEFILE_DIR. Treat these as read-only.

First remove the existing problematic cache file from the src directory:

cd src
rm CMakeCache.txt
cd ..

Then remove all the set() commands and do:

cd Compile && rm -rf *
cmake ../src

As long as you're outside of the source directory when running CMake, it will not modify the source directory unless your CMakeList explicitly tells it to do so.

Once you have this working, you can look at where CMake puts things by default, and only if you're not satisfied with the default locations (such as the default value of EXECUTABLE_OUTPUT_PATH), modify only those you need. And try to express them relative to CMAKE_BINARY_DIR, CMAKE_CURRENT_BINARY_DIR, PROJECT_BINARY_DIR etc.

If you look at CMake documentation, you'll see variables partitioned into semantic sections. Except for very special circumstances, you should treat all those listed under "Variables that Provide Information" as read-only inside CMakeLists.

How should I have explained the difference between an Interface and an Abstract class?

hmm now the people are hungery practical approach, you are quite right but most of interviewer looks as per their current requirment and want a practical approach.

after finishing your answer you should jump on the example:

Abstract:

for example we have salary function which have some parametar common to all employee. then we can have a abstract class called CTC with partialy defined method body and it will got extends by all type of employee and get redeined as per their extra beefits. For common functonality.

public abstract class CTC {

    public int salary(int hra, int da, int extra)
    {
        int total;
        total = hra+da+extra;
        //incentive for specific performing employee
        //total = hra+da+extra+incentive;
        return total;
    }
}

class Manger extends CTC
{
}


class CEO extends CTC
{
}

class Developer extends CTC
{   
}

Interface

interface in java allow to have interfcae functionality without extending that one and you have to be clear with the implementation of signature of functionality that you want to introduce in your application. it will force you to have definiton. For different functionality.

public interface EmployeType {

    public String typeOfEmployee();
}

class ContarctOne implements EmployeType
{

    @Override
    public String typeOfEmployee() {
        return "contract";
    }

}

class PermanentOne implements EmployeType
{

    @Override
    public String typeOfEmployee() {
        return "permanent";
    }

}

you can have such forced activity with abstract class too by defined methgos as a abstract one, now a class tha extends abstract class remin abstract one untill it override that abstract function.

apache server reached MaxClients setting, consider raising the MaxClients setting

I recommend to use bellow formula suggested on Apache:

MaxClients = (total RAM - RAM for OS - RAM for external programs) / (RAM per httpd process)

Find my script here which is running on Rhel 6.7. you can made change according to your OS.

#!/bin/bash

echo "HostName=`hostname`"

#Formula
#MaxClients . (RAM - size_all_other_processes)/(size_apache_process)
total_httpd_processes_size=`ps -ylC httpd --sort:rss | awk '{ sum += $9 } END { print sum }'`
#echo "total_httpd_processes_size=$total_httpd_processes_size"
total_http_processes_count=`ps -ylC httpd --sort:rss | wc -l`
echo "total_http_processes_count=$total_http_processes_count"
AVG_httpd_process_size=$(expr $total_httpd_processes_size / $total_http_processes_count)
echo "AVG_httpd_process_size=$AVG_httpd_process_size"
total_httpd_process_size_MB=$(expr $AVG_httpd_process_size / 1024)
echo "total_httpd_process_size_MB=$total_httpd_process_size_MB"
total_pttpd_used_size=$(expr $total_httpd_processes_size / 1024)
echo "total_pttpd_used_size=$total_pttpd_used_size"
total_RAM_size=`free -m |grep Mem |awk '{print $2}'`
echo "total_RAM_size=$total_RAM_size"
total_used_size=`free -m |grep Mem |awk '{print $3}'`
echo "total_used_size=$total_used_size"
size_all_other_processes=$(expr $total_used_size - $total_pttpd_used_size)
echo "size_all_other_processes=$size_all_other_processes"
remaining_memory=$(($total_RAM_size - $size_all_other_processes))
echo "remaining_memory=$remaining_memory"
MaxClients=$((($total_RAM_size - $size_all_other_processes) / $total_httpd_process_size_MB))
echo "MaxClients=$MaxClients"
exit

UICollectionView current visible cell index

It will probably be best to use UICollectionViewDelegate methods: (Swift 3)

// Called before the cell is displayed    
func collectionView(_ collectionView: UICollectionView, willDisplay cell: UICollectionViewCell, forItemAt indexPath: IndexPath) {
    print(indexPath.row)
}

// Called when the cell is displayed
func collectionView(_ collectionView: UICollectionView, didEndDisplaying cell: UICollectionViewCell, forItemAt indexPath: IndexPath) {
    print(indexPath.row)
}

Split a large dataframe into a list of data frames based on common value in column

You can just as easily access each element in the list using e.g. path[[1]]. You can't put a set of matrices into an atomic vector and access each element. A matrix is an atomic vector with dimension attributes. I would use the list structure returned by split, it's what it was designed for. Each list element can hold data of different types and sizes so it's very versatile and you can use *apply functions to further operate on each element in the list. Example below.

#  For reproducibile data
set.seed(1)

#  Make some data
userid <- rep(1:2,times=4)
data1 <- replicate(8 , paste( sample(letters , 3 ) , collapse = "" ) )
data2 <- sample(10,8)
df <- data.frame( userid , data1 , data2 )

#  Split on userid
out <- split( df , f = df$userid )
#$`1`
#  userid data1 data2
#1      1   gjn     3
#3      1   yqp     1
#5      1   rjs     6
#7      1   jtw     5

#$`2`
#  userid data1 data2
#2      2   xfv     4
#4      2   bfe    10
#6      2   mrx     2
#8      2   fqd     9

Access each element using the [[ operator like this:

out[[1]]
#  userid data1 data2
#1      1   gjn     3
#3      1   yqp     1
#5      1   rjs     6
#7      1   jtw     5

Or use an *apply function to do further operations on each list element. For instance, to take the mean of the data2 column you could use sapply like this:

sapply( out , function(x) mean( x$data2 ) )
#   1    2 
#3.75 6.25 

Is there a way to follow redirects with command line cURL?

Use the location header flag:

curl -L <URL>

live output from subprocess command

We can also use the default file iterator for reading stdout instead of using iter construct with readline().

import subprocess
import sys
process = subprocess.Popen(your_command, stdout=subprocess.PIPE)
for line in process.stdout:
    sys.stdout.write(line)

How to add a fragment to a programmatically generated layout?

At some point, I suppose you will add your programatically created LinearLayout to some root layout that you defined in .xml. This is just a suggestion of mine and probably one of many solutions, but it works: Simply set an ID for the programatically created layout, and add it to the root layout that you defined in .xml, and then use the set ID to add the Fragment.

It could look like this:

LinearLayout rowLayout = new LinearLayout();
rowLayout.setId(whateveryouwantasid);
// add rowLayout to the root layout somewhere here

FragmentManager fragMan = getFragmentManager();
FragmentTransaction fragTransaction = fragMan.beginTransaction();   

Fragment myFrag = new ImageFragment();
fragTransaction.add(rowLayout.getId(), myFrag , "fragment" + fragCount);
fragTransaction.commit();

Simply choose whatever Integer value you want for the ID:

rowLayout.setId(12345);

If you are using the above line of code not just once, it would probably be smart to figure out a way to create unique-IDs, in order to avoid duplicates.

UPDATE:

Here is the full code of how it should be done: (this code is tested and works) I am adding two Fragments to a LinearLayout with horizontal orientation, resulting in the Fragments being aligned next to each other. Please also be aware, that I used a fixed height and width of 200dp, so that one Fragment does not use the full screen as it would with "match_parent".

MainActivity.java:

public class MainActivity extends Activity {

    @SuppressLint("NewApi")
    @Override
    protected void onCreate(Bundle savedInstanceState) {
        super.onCreate(savedInstanceState);
        setContentView(R.layout.activity_main);     

        LinearLayout fragContainer = (LinearLayout) findViewById(R.id.llFragmentContainer);

        LinearLayout ll = new LinearLayout(this);
        ll.setOrientation(LinearLayout.HORIZONTAL);

        ll.setId(12345);

        getFragmentManager().beginTransaction().add(ll.getId(), TestFragment.newInstance("I am frag 1"), "someTag1").commit();
        getFragmentManager().beginTransaction().add(ll.getId(), TestFragment.newInstance("I am frag 2"), "someTag2").commit();

        fragContainer.addView(ll);
    }
}

TestFragment.java:

public class TestFragment extends Fragment {

    public static TestFragment newInstance(String text) {

        TestFragment f = new TestFragment();

        Bundle b = new Bundle();
        b.putString("text", text);
        f.setArguments(b);
        return f;
    }

    @Override
    public View onCreateView(LayoutInflater inflater, ViewGroup container, Bundle savedInstanceState) {

        View v =  inflater.inflate(R.layout.fragment, container, false);

        ((TextView) v.findViewById(R.id.tvFragText)).setText(getArguments().getString("text"));     
        return v;
    }
}

activity_main.xml:

<RelativeLayout xmlns:android="http://schemas.android.com/apk/res/android"
    xmlns:tools="http://schemas.android.com/tools"
    android:id="@+id/rlMain"
    android:layout_width="match_parent"
    android:layout_height="match_parent"
    android:padding="5dp"
    tools:context=".MainActivity" >

    <TextView
        android:id="@+id/textView1"
        android:layout_width="wrap_content"
        android:layout_height="wrap_content"
        android:text="@string/hello_world" />

    <LinearLayout
        android:id="@+id/llFragmentContainer"
        android:layout_width="match_parent"
        android:layout_height="match_parent"
        android:layout_alignLeft="@+id/textView1"
        android:layout_below="@+id/textView1"
        android:layout_marginTop="19dp"
        android:orientation="vertical" >
    </LinearLayout>
</RelativeLayout>

fragment.xml:

  <?xml version="1.0" encoding="utf-8"?>
<RelativeLayout xmlns:android="http://schemas.android.com/apk/res/android"
    android:layout_width="200dp"
    android:layout_height="200dp" >

    <TextView
        android:id="@+id/tvFragText"
        android:layout_width="wrap_content"
        android:layout_height="wrap_content"
        android:layout_centerHorizontal="true"
        android:layout_centerVertical="true"
        android:text="" />

</RelativeLayout>

And this is the result of the above code: (the two Fragments are aligned next to each other) result

How do I print the content of a .txt file in Python?

This will give you the contents of a file separated, line-by-line in a list:

with open('xyz.txt') as f_obj:
    f_obj.readlines()

Upload video files via PHP and save them in appropriate folder and have a database entry

sample code:

<em><b>
<h2>Upload,Save and Download video </h2>
<form method="POST" action="" enctype="multipart/form-data">
<input type="file" name="video"/>
<input type="submit" name="submit" value="Upload"/></b>
</form></em>

<?php>
include("connect.php");
$errors=1;
//Targeting Folder
$target="videos/";
if(isset($_POST['submit'])){
//Targeting Folder 
$target=$target.basename($_FILES['video']['name']);
//Getting Selected video Type
$type=pathinfo($target,PATHINFO_EXTENSION);
 //Allow Certain File Format To Upload
 if($type!='mp4' && $type!='3gp' && $type!='avi'){
  echo "Only mp4,3gp,avi file format are allowed to Upload";
  $errors=0;
 }
 //checking for Exsisting video Files
  if(file_exists($target)){
   echo "File Exist";
   $errors=0;
   }
  $filesize=$_FILES['video']['size'];
  if($filesize>250*2000000){
  echo 'You Can not Upload Large File(more than 500MB) by Default ini setting..<a     href="http://www.codenair.com/2018/03/how-to-upload-large-file-in-php.html">How to   upload large file in php</a>'; 
    $errors=0;
    }
   if($errors == 0){
   echo ' Your File Not Uploaded';
    }else{
 //Moving The video file to Desired Directory
  $uplaod_success=move_uploaded_file($_FILES['video']['tmp_name'],$target);
  if($uplaod_success){
  //Getting Selected video Information
    $name=$_FILES['video']['name'];
    $size=$_FILES['video']['size'];
    $result=mysqli_query($con,"INSERT INTO VIdeos           (name,size,type)VALUES('".$name."','".$size."','".$type."')");
    if($result==TRUE){
    echo "Your video '$name' Successfully Upload and Information Saved Our  Database";
    }
   }
  }
  }
 ?>

Understanding the grid classes ( col-sm-# and col-lg-# ) in Bootstrap 3

Here you have a very good tutorial, that explains, how to use the new grid classes in Bootstrap 3.

It also covers mixins etc.

How to get raw text from pdf file using java

Extracting all keywords from PDF(from a web page) file on your local machine or Base64 encoded string:

import org.apache.commons.codec.binary.Base64;
import org.apache.pdfbox.pdmodel.PDDocument;
import org.apache.pdfbox.text.PDFTextStripper;

import java.io.File;
import java.io.FileInputStream;
import java.io.FileNotFoundException;
import java.io.IOException;
import java.util.HashMap;
import java.util.Map;

public class WebPagePdfExtractor {

    public static void main(String arg[]) {
        WebPagePdfExtractor webPagePdfExtractor = new WebPagePdfExtractor();

        System.out.println("From file:   " + webPagePdfExtractor.processRecord(createByteArray()).get("text"));

        System.out.println("From string: " + webPagePdfExtractor.processRecord(getArrayFromBase64EncodedString()).get("text"));
    }

    public Map<String, Object> processRecord(byte[] byteArray) {
        Map<String, Object> map = new HashMap<>();
        try {
            PDFTextStripper stripper = new PDFTextStripper();
            stripper.setSortByPosition(false);
            stripper.setShouldSeparateByBeads(true);

            PDDocument document = PDDocument.load(byteArray);
            String text = stripper.getText(document);
            map.put("text", text.replaceAll("\n|\r|\t", " "));
        } catch (Exception exception) {
            exception.printStackTrace();
        }
        return map;
    }

    private static byte[] getArrayFromBase64EncodedString() {
        String encodedContent = "data:application/pdf;base64,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" +
                "j4+CmVuZG9iago2IDAgb2JqCjw8IC9Qcm9jU2V0IFsgL1BERiAvVGV4dCBdIC9Db2xvclNwYWNlIDw8IC9DczEgNyAwIFIgL0NzMiA4IDAgUiA+PiAvRm9udCA8PAovVFQxIDkgMCBSID4+ID4+CmVuZG9iagoxMCAwIG9iago8PCAvTGVuZ3RoIDExIDAgUiAvTiAxIC9BbHRlcm5hdGUgL0RldmljZUdyYXkgL0ZpbHRlciAvRmxhdGVEZWNvZGUgPj4Kc3RyZWFtCngBhVVdaBxVFD67c2cDEgcftA0ttIM/bQnpMolWE4u12026SRO362ZTmyrKdHY2O81kZpyZ3SahT6XgmxYE6augPsaCCLYqNi/2paXFkko1DwoRWowgKH1S8Dsz22R2QTLDnfnuueeee8537rmXqOtv3fPstEo054R+oZybPjl9Su26TWlSqJvw6Ebg5UqlCcaO65j8b38e3qUUS+7sZ1vtY1v25KoZGNC6huZWA2OOKKURZWqG54dEXZcgHzwbeoxvAz85WynngdeAldZcQHqqYDqmbxlqwdcX1JLv1i" +
                "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" +
                "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" +
                "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" +
                "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" +
                "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" +
                "zIFsgMiAwIFIgXSA+PgplbmRvYmoKMTQgMCBvYmoKPDwgL1R5cGUgL0NhdGFsb2cgL1BhZ2VzIDMgMCBSID4+CmVuZG9iago5IDAgb2JqCjw8IC9UeXBlIC9Gb250IC9TdWJ0eXBlIC9UcnVlVHlwZSAvQmFzZUZvbnQgL0NOVFpYVStNZW5" +
                "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" +
                "L0l0YWxpY0FuZ2xlIDAgL0FzY2VudCA5MjggL0Rlc2NlbnQgLTIzNiAvQ2FwSGVpZ2h0IDcyOQovU3RlbVYgOTkgL1hIZWlnaHQgNTQ3…/ZfICj5JcLdi/ATmQZKogDPg0lIDBunI0ZGOB1OB/Lpyce1TbJqCpBThycVs3GyQPZSLKexbMGyFss8LF4sNb2lElu5HPlJ2439G1jKsbRh6cTyPNpx8I6AFxa8P+xD2E4e/G+5PqJ/8aDzERFvGBJR/WLkfwcM3kRCiZpokDMdxhn5MeD9Rn5MSm0mYUpLSF98J5HXaQgtpJvoDWGesEe4C4NgK3woWsQ88RgzszXsMM4WyALeIC5gO5B/FYk/pNxVCJGoZT8NYc8LIknrONeVQYznus51pYeZHCaXw+RYIJLAEogJfMEbVPrvv31S6icvTMlp1EQhO41cOuXb0EEkSYkmGaMXSzuIfhCKAA4Y/YScTs9ASizblWVyWB1UT4fwNfSp9+mgwLFd4oI3D++9++kuheYWpOnEeBhLJrv7kVg" +
                "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" +
                "1RnXWxp1QiNZo2tS7baBjUTYqDqBGONtspI7UEwosSsoMUVevAM5CEO9mmRVEquF/ExwsrxOCTd7OpKnpXxFjfzzO8uPTnz44Oydb7bunLy1kHXu5huMBt59vYvfsNtPZmb4tjfvdblQGjXI23jUayTpg9w5VfFRjer4RqP6DRGPs/ViY3iDscmVYCN9dQkqKZaGxbuMga6uwBXZeYLq/MKI6jShPq0DqDNBUBg0Wy2C0y6YjMSRGU4TJKslPKhYuJS7fkL7u+m7F33yzc3PeOBb6qSWsZv4Zys3bVq5as0atu+gK5Ff4ldLH+d3vvuW36bL6Ab6LF0X37Pw4I4dB//4+z0+RZ8y306xEuNGP7LI3V+tItF2baRBRfZHqurNjjr7O3H1fdrMTZE6GilG6dWSNt8uStbh/Y03u9AkM1G3snI7rtwMyCKWd2DKMeegZ6W749Lj0+3pjvSEZtJMm4VmdbNme3hzRHNkc1RztH4m7rJ3Q4OzB5uc2XpE9M0eOOh+mi1LoNfdwlGfQtuwV159duGWPfTAgfv/VP3GBz98d4eu2jirfca81uK6o8P62oWsJxaXLT57sN/4npUtpY/8eXvr4bhVzwwa6E9MnDIlc2PQditxr2bMIowYLdLd0YxYouv1lvqQJn0bfREiRCIJo0xmzeg43Ju8NTk2KIaDe0ynpqxeHlEd5ixUx790gazCdL9/QFPpiWvX3y/byg1ramr" +
                "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" +
                "JTPMvxBMSaYoSmQ+nHtDywiJbzyzTe7xcfDmR5vTBcyPsKv7yBPEyXopGDxCAHOoUoppRILPQiribnP/IqOjlLka3XEo5eIbu8bCTCqRmej6+99ib/lF6im3/13ItnD8OtF4LyxN+YxQq0iwTyqjsx0mwIFVUkLkZSWbX1dmiLORxlVBGTIWSC" +
                "NNE0wTAxNnJCdIHTeHOcTzt1nM80dUbxARJ9r/0+T2BoAA0leOYPHXrlpnIwoYmg8NPdo9LFdJYupavSQ9JD09Xpmtzw3IjcyNyo3OjcmNzY3LhcWzVUi9WsWqpWVYdUh1arqzXecG+EN9Ib5Y32xnhjvXFem5POpPLJEh5Ff6LMf2vVqgwKOx" +
                "IeHbu64vXswkn3v54zdkzOzp2Oubnjy6B7dMEZfqlnubDymyWVX/SsEFbeWCy3YknJ0NxCWddt/CFxKspCjmFZ7tgfY1ibvokegcNxGL9GKZGsUI5imYqJoV/8GMZcsm0pXJhNRtkbfuofdPmBA3IYu/rV+/Oa6I3VNatqa1fVrF7Xc1xSe4um" +
                "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" +
                "uIAowMDAwMDAzNzk2IDAwMDAwIG4gCjAwMDAwMDg2ODcgMDAwMDAgbiAKMDAwMDAwODcwOCAwMDAwMCBuIAowMDAwMDA4NzI3IDAwMDAwIG4gCjAwMDAwMDg3ODAgMDAwMDAgbiAKMDAwMDAwODc5OSAwMDAwMCBuIAowMDAwMDA4ODE4IDAwMDAwIG4gCjAwMDAwMDg4NDUgMDAwMDAgbiAKMDAwMDAwODg4NyAwMDAwMCBuIAowMDAwMDA4OTA2IDAwMDAwIG4gCnRyYWlsZXIKPDwgL1NpemUgMjYgL1Jvb3QgMTQgMCBSIC9JbmZvIDEgMCBSIC9JRCBbIDxkYjc4M2NhNDM2Mzg4YzI5ZDc5MDQ2NzY3NjUxNjE3OT4KPGRiNzgzY2E0MzYzODhjMjlkNzkwNDY3Njc2NTE2MTc5PiBdID4+CnN0YXJ0eHJlZgo5MTA0CiUlRU9GCg==";
        String content = encodedContent.substring("data:application/pdf;base64," .length());
        return Base64.decodeBase64(content);
    }

    public static byte[] createByteArray() {
        String pathToBinaryData = "/bla-bla/src/main/resources/small.pdf";

        File file = new File(pathToBinaryData);
        if (!file.exists()) {
            System.out.println(" could not be found in folder " + pathToBinaryData);
            return null;
        }

        FileInputStream fin = null;
        try {
            fin = new FileInputStream(file);
        } catch (FileNotFoundException e) {
            e.printStackTrace();
        }

        byte fileContent[] = new byte[(int) file.length()];

        try {
            fin.read(fileContent);
        } catch (IOException e) {
            e.printStackTrace();
        }

        return fileContent;
    }
}

When correctly use Task.Run and when just async-await

Note the guidelines for performing work on a UI thread, collected on my blog:

  • Don't block the UI thread for more than 50ms at a time.
  • You can schedule ~100 continuations on the UI thread per second; 1000 is too much.

There are two techniques you should use:

1) Use ConfigureAwait(false) when you can.

E.g., await MyAsync().ConfigureAwait(false); instead of await MyAsync();.

ConfigureAwait(false) tells the await that you do not need to resume on the current context (in this case, "on the current context" means "on the UI thread"). However, for the rest of that async method (after the ConfigureAwait), you cannot do anything that assumes you're in the current context (e.g., update UI elements).

For more information, see my MSDN article Best Practices in Asynchronous Programming.

2) Use Task.Run to call CPU-bound methods.

You should use Task.Run, but not within any code you want to be reusable (i.e., library code). So you use Task.Run to call the method, not as part of the implementation of the method.

So purely CPU-bound work would look like this:

// Documentation: This method is CPU-bound.
void DoWork();

Which you would call using Task.Run:

await Task.Run(() => DoWork());

Methods that are a mixture of CPU-bound and I/O-bound should have an Async signature with documentation pointing out their CPU-bound nature:

// Documentation: This method is CPU-bound.
Task DoWorkAsync();

Which you would also call using Task.Run (since it is partially CPU-bound):

await Task.Run(() => DoWorkAsync());

How to terminate process from Python using pid?

Using the awesome psutil library it's pretty simple:

p = psutil.Process(pid)
p.terminate()  #or p.kill()

If you don't want to install a new library, you can use the os module:

import os
import signal

os.kill(pid, signal.SIGTERM) #or signal.SIGKILL 

See also the os.kill documentation.


If you are interested in starting the command python StripCore.py if it is not running, and killing it otherwise, you can use psutil to do this reliably.

Something like:

import psutil
from subprocess import Popen

for process in psutil.process_iter():
    if process.cmdline() == ['python', 'StripCore.py']:
        print('Process found. Terminating it.')
        process.terminate()
        break
else:
    print('Process not found: starting it.')
    Popen(['python', 'StripCore.py'])

Sample run:

$python test_strip.py   #test_strip.py contains the code above
Process not found: starting it.
$python test_strip.py 
Process found. Terminating it.
$python test_strip.py 
Process not found: starting it.
$killall python
$python test_strip.py 
Process not found: starting it.
$python test_strip.py 
Process found. Terminating it.
$python test_strip.py 
Process not found: starting it.

Note: In previous psutil versions cmdline was an attribute instead of a method.

jQuery load first 3 elements, click "load more" to display next 5 elements

The expression $(document).ready(function() deprecated in jQuery3.

See working fiddle with jQuery 3 here

Take into account I didn't include the showless button.

Here's the code:

JS

$(function () {
    x=3;
    $('#myList li').slice(0, 3).show();
    $('#loadMore').on('click', function (e) {
        e.preventDefault();
        x = x+5;
        $('#myList li').slice(0, x).slideDown();
    });
});

CSS

#myList li{display:none;
}
#loadMore {
    color:green;
    cursor:pointer;
}
#loadMore:hover {
    color:black;
}

Reading a huge .csv file

You are reading all rows into a list, then processing that list. Don't do that.

Process your rows as you produce them. If you need to filter the data first, use a generator function:

import csv

def getstuff(filename, criterion):
    with open(filename, "rb") as csvfile:
        datareader = csv.reader(csvfile)
        yield next(datareader)  # yield the header row
        count = 0
        for row in datareader:
            if row[3] == criterion:
                yield row
                count += 1
            elif count:
                # done when having read a consecutive series of rows 
                return

I also simplified your filter test; the logic is the same but more concise.

Because you are only matching a single sequence of rows matching the criterion, you could also use:

import csv
from itertools import dropwhile, takewhile

def getstuff(filename, criterion):
    with open(filename, "rb") as csvfile:
        datareader = csv.reader(csvfile)
        yield next(datareader)  # yield the header row
        # first row, plus any subsequent rows that match, then stop
        # reading altogether
        # Python 2: use `for row in takewhile(...): yield row` instead
        # instead of `yield from takewhile(...)`.
        yield from takewhile(
            lambda r: r[3] == criterion,
            dropwhile(lambda r: r[3] != criterion, datareader))
        return

You can now loop over getstuff() directly. Do the same in getdata():

def getdata(filename, criteria):
    for criterion in criteria:
        for row in getstuff(filename, criterion):
            yield row

Now loop directly over getdata() in your code:

for row in getdata(somefilename, sequence_of_criteria):
    # process row

You now only hold one row in memory, instead of your thousands of lines per criterion.

yield makes a function a generator function, which means it won't do any work until you start looping over it.

How to check for palindrome using Python logic

There is also a functional way:

def is_palindrome(word):
  if len(word) == 1: return True
  if word[0] != word[-1]: return False
  return is_palindrome(word[1:-1])

%i or %d to print integer in C using printf()?

They are completely equivalent when used with printf(). Personally, I prefer %d, it's used more often (should I say "it's the idiomatic conversion specifier for int"?).

(One difference between %i and %d is that when used with scanf(), then %d always expects a decimal integer, whereas %i recognizes the 0 and 0x prefixes as octal and hexadecimal, but no sane programmer uses scanf() anyway so this should not be a concern.)

String in function parameter

function("MyString");

is similar to

char *s = "MyString";
function(s);

"MyString" is in both cases a string literal and in both cases the string is unmodifiable.

function("MyString");

passes the address of a string literal to function as an argument.

Parsing JSON objects for HTML table

Loop over each object, appending a table row with the relevant data each iteration.

$(document).ready(function () {
    $.getJSON(url,
    function (json) {
        var tr;
        for (var i = 0; i < json.length; i++) {
            tr = $('<tr/>');
            tr.append("<td>" + json[i].User_Name + "</td>");
            tr.append("<td>" + json[i].score + "</td>");
            tr.append("<td>" + json[i].team + "</td>");
            $('table').append(tr);
        }
    });
});

JSFiddle

How can I send cookies using PHP curl in addition to CURLOPT_COOKIEFILE?

I think the only cookie you need is JSESSIONID=xxx..

Also NEVER share your cookies, becasuse someone may access your personal data that way. Specially when the cookies are session. These cookies will stop working once you logout the site.

Difference between "enqueue" and "dequeue"

Enqueue and Dequeue tend to be operations on a queue, a data structure that does exactly what it sounds like it does.

You enqueue items at one end and dequeue at the other, just like a line of people queuing up for tickets to the latest Taylor Swift concert (I was originally going to say Billy Joel but that would date me severely).

There are variations of queues such as double-ended ones where you can enqueue and dequeue at either end but the vast majority would be the simpler form:

           +---+---+---+
enqueue -> | 3 | 2 | 1 | -> dequeue
           +---+---+---+

That diagram shows a queue where you've enqueued the numbers 1, 2 and 3 in that order, without yet dequeuing any.


By way of example, here's some Python code that shows a simplistic queue in action, with enqueue and dequeue functions. Were it more serious code, it would be implemented as a class but it should be enough to illustrate the workings:

import random

def enqueue(lst, itm):
    lst.append(itm)        # Just add item to end of list.
    return lst             # And return list (for consistency with dequeue).

def dequeue(lst):
    itm = lst[0]           # Grab the first item in list.
    lst = lst[1:]          # Change list to remove first item.
    return (itm, lst)      # Then return item and new list.

# Test harness. Start with empty queue.

myList = []

# Enqueue or dequeue a bit, with latter having probability of 10%.

for _ in range(15):
    if random.randint(0, 9) == 0 and len(myList) > 0:
        (itm, myList) = dequeue(myList)
        print(f"Dequeued {itm} to give {myList}")
    else:
        itm = 10 * random.randint(1, 9)
        myList = enqueue(myList, itm)
        print(f"Enqueued {itm} to give {myList}")

# Now dequeue remainder of list.

print("========")
while len(myList) > 0:
    (itm, myList) = dequeue(myList)
    print(f"Dequeued {itm} to give {myList}")

A sample run of that shows it in operation:

Enqueued 70 to give [70]
Enqueued 20 to give [70, 20]
Enqueued 40 to give [70, 20, 40]
Enqueued 50 to give [70, 20, 40, 50]
Dequeued 70 to give [20, 40, 50]
Enqueued 20 to give [20, 40, 50, 20]
Enqueued 30 to give [20, 40, 50, 20, 30]
Enqueued 20 to give [20, 40, 50, 20, 30, 20]
Enqueued 70 to give [20, 40, 50, 20, 30, 20, 70]
Enqueued 20 to give [20, 40, 50, 20, 30, 20, 70, 20]
Enqueued 20 to give [20, 40, 50, 20, 30, 20, 70, 20, 20]
Dequeued 20 to give [40, 50, 20, 30, 20, 70, 20, 20]
Enqueued 80 to give [40, 50, 20, 30, 20, 70, 20, 20, 80]
Dequeued 40 to give [50, 20, 30, 20, 70, 20, 20, 80]
Enqueued 90 to give [50, 20, 30, 20, 70, 20, 20, 80, 90]
========
Dequeued 50 to give [20, 30, 20, 70, 20, 20, 80, 90]
Dequeued 20 to give [30, 20, 70, 20, 20, 80, 90]
Dequeued 30 to give [20, 70, 20, 20, 80, 90]
Dequeued 20 to give [70, 20, 20, 80, 90]
Dequeued 70 to give [20, 20, 80, 90]
Dequeued 20 to give [20, 80, 90]
Dequeued 20 to give [80, 90]
Dequeued 80 to give [90]
Dequeued 90 to give []

build-impl.xml:1031: The module has not been deployed

  • Close Netbeans.
  • Delete all libraries in the folder "yourprojectfolder"\build\web\WEB-INF\lib
  • Open Netbeans.
  • Clean and Build project.
  • Deploy project.

Can't install any package with node npm

Following on from LAXIT KUMAR's recommendation, we've found that at times the registry URL can get corrupted, not sure how this is happening, however to cure, just reset it:

npm config set registry https://registry.npmjs.org

The 404 disappeared as it was going to the correct location again.

Dependency injection with Jersey 2.0

If you prefer to use Guice and you don't want to declare all the bindings, you can also try this adapter:

guice-bridge-jit-injector

Close Window from ViewModel

You can use Messenger from MVVMLight toolkit. in your ViewModel send a message like this:
Messenger.Default.Send(new NotificationMessage("Close"));
then in your windows code behind, after InitializeComponent, register for that message like this:

Messenger.Default.Register<NotificationMessage>(this, m=>{
    if(m.Notification == "Close") 
    {
        this.Close();
    }
   });

you can find more about MVVMLight toolkit here: MVVMLight toolkit on Codeplex

Notice that there is not a "no code-behind at all rule" in MVVM and you can do registering for messages in a view code-behind.

How to get keyboard input in pygame?

You can get the events from pygame and then watch out for the KEYDOWN event, instead of looking at the keys returned by get_pressed()(which gives you keys that are currently pressed down, whereas the KEYDOWN event shows you which keys were pressed down on that frame).

What's happening with your code right now is that if your game is rendering at 30fps, and you hold down the left arrow key for half a second, you're updating the location 15 times.

events = pygame.event.get()
for event in events:
    if event.type == pygame.KEYDOWN:
        if event.key == pygame.K_LEFT:
            location -= 1
        if event.key == pygame.K_RIGHT:
            location += 1

To support continuous movement while a key is being held down, you would have to establish some sort of limitation, either based on a forced maximum frame rate of the game loop or by a counter which only allows you to move every so many ticks of the loop.

move_ticker = 0
keys=pygame.key.get_pressed()
if keys[K_LEFT]:
    if move_ticker == 0:
        move_ticker = 10
        location -= 1
        if location == -1:
            location = 0
if keys[K_RIGHT]:
    if move_ticker == 0:   
        move_ticker = 10     
        location+=1
        if location == 5:
            location = 4

Then somewhere during the game loop you would do something like this:

if move_ticker > 0:
    move_ticker -= 1

This would only let you move once every 10 frames (so if you move, the ticker gets set to 10, and after 10 frames it will allow you to move again)

The operation couldn’t be completed. (com.facebook.sdk error 2.) ios6

Another potential cause for this error: Attempting to get permission for a Facebook app in sandbox mode when the Facebook user is not listed in the app's admins, developers or testers.

javax.naming.NoInitialContextException - Java

If working on EJB client library:

You need to mention the argument for getting the initial context.

InitialContext ctx = new InitialContext();

If you do not, it will look in the project folder for properties file. Also you can include the properties credentials or values in your class file itself as follows:

Properties props = new Properties();
props.put(Context.INITIAL_CONTEXT_FACTORY, "org.jnp.interfaces.NamingContextFactory");
props.put(Context.URL_PKG_PREFIXES, "org.jboss.ejb.client.naming");
props.put(Context.PROVIDER_URL, "jnp://localhost:1099");

InitialContext ctx = new InitialContext(props);

URL_PKG_PREFIXES: Constant that holds the name of the environment property for specifying the list of package prefixes to use when loading in URL context factories.

The EJB client library is the primary library to invoke remote EJB components.
This library can be used through the InitialContext. To invoke EJB components the library creates an EJB client context via a URL context factory. The only necessary configuration is to parse the value org.jboss.ejb.client.naming for the java.naming.factory.url.pkgs property to instantiate an InitialContext.

Youtube API Limitations

Version 3 of the YouTube Data API has concrete quota numbers listed in the Google API Console where you register for your API Key. You can use 10,000 units per day. Projects that had enabled the YouTube Data API before April 20, 2016, have a default quota of 50M/day.

You can read about what a unit is here: https://developers.google.com/youtube/v3/getting-started#quota

  • A simple read operation that only retrieves the ID of each returned resource has a cost of approximately 1 unit.
  • A write operation has a cost of approximately 50 units.
  • A video upload has a cost of approximately 1600 units.

If you hit the limits, Google will stop returning results until your quota is reset. You can apply for more than 1M requests per day, but you will have to pay for those extra requests.

Also, you can read about why Google has deferred support to StackOverflow on their YouTube blog here: https://youtube-eng.googleblog.com/2012/09/the-youtube-api-on-stack-overflow_14.html

There are a number of active members on the YouTube Developer Relations team here including Jeff Posnick, Jarek Wilkiewicz, and Ibrahim Ulukaya who all have knowledge of Youtube internals...

UPDATE: Increased the quota numbers to reflect current limits on December 10, 2013.

UPDATE: Decreased the quota numbers from 50M to 1M per day to reflect current limits on May 13, 2016.

UPDATE: Decreased the quota numbers from 1M to 10K per day as of January 11, 2019.

Delete many rows from a table using id in Mysql

If you have some 'condition' in your data to figure out the 254 ids, you could use:

delete from tablename
where id in 
(select id from tablename where <your-condition>)

or simply:

delete from tablename where <your-condition>

Simply hard coding the 254 values of id column would be very tough in any case.

Connection to SQL Server Works Sometimes

I had the same issue that automatically resolve after last Microsoft windows update, does anyone experience the same?

Excel VBA Check if directory exists error

This is the cleanest way... BY FAR:

Public Function IsDir(s) As Boolean
    IsDir = CreateObject("Scripting.FileSystemObject").FolderExists(s)
End Function

Excel- compare two cell from different sheet, if true copy value from other cell

In your destination field you want to use VLOOKUP like so:

=VLOOKUP(Sheet1!A1:A100,Sheet2!A1:F100,6,FALSE)

VLOOKUP Arguments:

  1. The set fields you want to lookup.
  2. The table range you want to lookup up your value against. The first column of your defined table should be the column you want compared against your lookup field. The table range should also contain the value you want to display (Column F).
  3. This defines what field you want to display upon a match.
  4. FALSE tells VLOOKUP to do an exact match.

Why I've got no crontab entry on OS X when using vim?

As the previous posts didn't work for me because of some permissions issues, I found that creating a separate crontab file and adding it to the user's crontab with the -u parameter while root worked for me.

sudo crontab -u {USERNAME} ~/{PATH_TO_CRONTAB_FILE}

JavaScript Extending Class

Updated below for ES6

March 2013 and ES5

This MDN document describes extending classes well:

https://developer.mozilla.org/en-US/docs/JavaScript/Introduction_to_Object-Oriented_JavaScript

In particular, here is now they handle it:

// define the Person Class
function Person() {}

Person.prototype.walk = function(){
  alert ('I am walking!');
};
Person.prototype.sayHello = function(){
  alert ('hello');
};

// define the Student class
function Student() {
  // Call the parent constructor
  Person.call(this);
}

// inherit Person
Student.prototype = Object.create(Person.prototype);

// correct the constructor pointer because it points to Person
Student.prototype.constructor = Student;

// replace the sayHello method
Student.prototype.sayHello = function(){
  alert('hi, I am a student');
}

// add sayGoodBye method
Student.prototype.sayGoodBye = function(){
  alert('goodBye');
}

var student1 = new Student();
student1.sayHello();
student1.walk();
student1.sayGoodBye();

// check inheritance
alert(student1 instanceof Person); // true 
alert(student1 instanceof Student); // true

Note that Object.create() is unsupported in some older browsers, including IE8:

Object.create browser support

If you are in the position of needing to support these, the linked MDN document suggests using a polyfill, or the following approximation:

function createObject(proto) {
    function ctor() { }
    ctor.prototype = proto;
    return new ctor();
}

Using this like Student.prototype = createObject(Person.prototype) is preferable to using new Person() in that it avoids calling the parent's constructor function when inheriting the prototype, and only calls the parent constructor when the inheritor's constructor is being called.

May 2017 and ES6

Thankfully, the JavaScript designers have heard our pleas for help and have adopted a more suitable way of approaching this issue.

MDN has another great example on ES6 class inheritance, but I'll show the exact same set of classes as above reproduced in ES6:

class Person {
    sayHello() {
        alert('hello');
    }

    walk() {
        alert('I am walking!');
    }
}

class Student extends Person {
    sayGoodBye() {
        alert('goodBye');
    }

    sayHello() {
        alert('hi, I am a student');
    }
}

var student1 = new Student();
student1.sayHello();
student1.walk();
student1.sayGoodBye();

// check inheritance
alert(student1 instanceof Person); // true 
alert(student1 instanceof Student); // true

Clean and understandable, just like we all want. Keep in mind, that while ES6 is pretty common, it's not supported everywhere:

ES6 browser support

Difference between Encapsulation and Abstraction

Just a few more points to make thing clear,

One must not confuse data abstraction and the abstract class. They are different.

Generally we say abstract class or method is to basically hide something. But no.. That is wrong. What is the word abstract means ? Google search says the English word abstraction means

"Existing in thought or as an idea but not having a physical or concrete existence."

And thats right in case of abstract class too. It is not hiding the content of the method but the method's content is already empty (not having a physical or concrete existence) but it determines how a method should be (existing in thought or as an idea) or a method should be in the calss.

So when do you actually use abstract methods ?

  • When a method from base class will differ in each child class that extends it.
  • And so you want to make sure the child class have this function implemented.
  • This also ensures that method, to have compulsory signature like, it must have n number of parameters.

So about abstract class! - An Abstract class cannot be instantiated only extended! But why ?

  • A class with abstract method must be prevented from creating its own instance because the abstract methods in it, are not having any meaningful implementation.
  • You can even make a class abstract, if for some reason you find that it is meaning less to have a instance of your that class.

An Abstract class help us avoid creating new instance of it!

An abstract method in a class forces the child class to implement that function for sure with the provided signature!

Can we execute a java program without a main() method?

Since you tagged Java-ee as well - then YES it is possible.

and in core java as well it is possible using static blocks

and check this How can you run a Java program without main method?

Edit:
as already pointed out in other answers - it does not support from Java 7

Change onClick attribute with javascript

You are not actually changing the function.

onClick is assigned to a function (Which is a reference to something, a function pointer in this case). The values passed to it don't matter and cannot be utilised in any manner.

Another problem is your variable color seems out of nowhere.

Ideally, inside the function you should put this logic and let it figure out what to write. (on/off etc etc)

JPA: How to get entity based on field value other than ID?

I've written a library that helps do precisely this. It allows search by object simply by initializing only the fields you want to filter by: https://github.com/kg6zvp/GenericEntityEJB

How to get page content using cURL?

For a realistic approach that emulates the most human behavior, you may want to add a referer in your curl options. You may also want to add a follow_location to your curl options. Trust me, whoever said that cURLING Google results is impossible, is a complete dolt and should throw his/her computer against the wall in hopes of never returning to the internetz again. Everything that you can do "IRL" with your own browser can all be emulated using PHP cURL or libCURL in Python. You just need to do more cURLS to get buff. Then you will see what I mean. :)

  $url = "http://www.google.com/search?q=".$strSearch."&hl=en&start=0&sa=N";
  $ch = curl_init();
  curl_setopt($ch, CURLOPT_REFERER, 'http://www.example.com/1');
  curl_setopt($ch, CURLOPT_HEADER, 0);
  curl_setopt($ch, CURLOPT_VERBOSE, 0);
  curl_setopt($ch, CURLOPT_RETURNTRANSFER, true);
  curl_setopt($ch, CURLOPT_USERAGENT, "Mozilla/4.0 (compatible;)");
  curl_setopt($ch, CURLOPT_URL, urlencode($url));
  $response = curl_exec($ch);
  curl_close($ch);

Simple example for Intent and Bundle

Try this: if you need pass values between the activities you use this...

This is code for Main_Activity put the values to intent

 String name="aaaa";
 Intent intent=new Intent(Main_Activity.this,Other_Activity.class);
 intent.putExtra("name", name);
 startActivity(intent);

This code for Other_Activity and get the values form intent

    Bundle b = new Bundle();
    b = getIntent().getExtras();
    String name = b.getString("name");

How to Create a script via batch file that will uninstall a program if it was installed on windows 7 64-bit or 32-bit

In my experience, to use wmic in a script, you need to get the nested quoting right:

wmic product where "name = 'Windows Azure Authoring Tools - v2.3'" call uninstall /nointeractive 

quoting both the query and the name. But wmic will only uninstall things installed via windows installer.

Parsing XML with namespace in Python via 'ElementTree'

ElementTree is not too smart about namespaces. You need to give the .find(), findall() and iterfind() methods an explicit namespace dictionary. This is not documented very well:

namespaces = {'owl': 'http://www.w3.org/2002/07/owl#'} # add more as needed

root.findall('owl:Class', namespaces)

Prefixes are only looked up in the namespaces parameter you pass in. This means you can use any namespace prefix you like; the API splits off the owl: part, looks up the corresponding namespace URL in the namespaces dictionary, then changes the search to look for the XPath expression {http://www.w3.org/2002/07/owl}Class instead. You can use the same syntax yourself too of course:

root.findall('{http://www.w3.org/2002/07/owl#}Class')

If you can switch to the lxml library things are better; that library supports the same ElementTree API, but collects namespaces for you in a .nsmap attribute on elements.

Clearing a text field on button click

Something like this will add a button and let you use it to clear the values

<div>           
<input type="text" id="textfield1" size="5"/>         
</div>

<div>          
<input type="text" id="textfield2" size="5"/>           
</div>

<div>
    <input type="button" onclick="clearFields()" value="Clear">
</div>


<script type="text/javascript">
function clearFields() {
    document.getElementById("textfield1").value=""
    document.getElementById("textfield2").value=""
}
</script>

Select the top N values by group

I prefer @Ista solution, cause needs no extra package and is simple.
A modification of the data.table solution also solve my problem, and is more general.
My data.frame is

> str(df)
'data.frame':   579 obs. of  11 variables:
 $ trees     : num  2000 5000 1000 2000 1000 1000 2000 5000 5000 1000 ...
 $ interDepth: num  2 3 5 2 3 4 4 2 3 5 ...
 $ minObs    : num  6 4 1 4 10 6 10 10 6 6 ...
 $ shrinkage : num  0.01 0.001 0.01 0.005 0.01 0.01 0.001 0.005 0.005 0.001     ...
 $ G1        : num  0 2 2 2 2 2 8 8 8 8 ...
 $ G2        : logi  FALSE FALSE FALSE FALSE FALSE FALSE ...
 $ qx        : num  0.44 0.43 0.419 0.439 0.43 ...
 $ efet      : num  43.1 40.6 39.9 39.2 38.6 ...
 $ prec      : num  0.606 0.593 0.587 0.582 0.574 0.578 0.576 0.579 0.588 0.585 ...
 $ sens      : num  0.575 0.57 0.573 0.575 0.587 0.574 0.576 0.566 0.542 0.545 ...
 $ acu       : num  0.631 0.645 0.647 0.648 0.655 0.647 0.619 0.611 0.591 0.594 ...

The data.table solution needs order on i to do the job:

> require(data.table)
> dt1 <- data.table(df)
> dt2 = dt1[order(-efet, G1, G2), head(.SD, 3), by = .(G1, G2)]
> dt2
    G1    G2 trees interDepth minObs shrinkage        qx   efet  prec  sens   acu
 1:  0 FALSE  2000          2      6     0.010 0.4395953 43.066 0.606 0.575 0.631
 2:  0 FALSE  2000          5      1     0.005 0.4294718 37.554 0.583 0.548 0.607
 3:  0 FALSE  5000          2      6     0.005 0.4395753 36.981 0.575 0.559 0.616
 4:  2 FALSE  5000          3      4     0.001 0.4296346 40.624 0.593 0.570 0.645
 5:  2 FALSE  1000          5      1     0.010 0.4186802 39.915 0.587 0.573 0.647
 6:  2 FALSE  2000          2      4     0.005 0.4390503 39.164 0.582 0.575 0.648
 7:  8 FALSE  2000          4     10     0.001 0.4511349 38.240 0.576 0.576 0.619
 8:  8 FALSE  5000          2     10     0.005 0.4469665 38.064 0.579 0.566 0.611
 9:  8 FALSE  5000          3      6     0.005 0.4426952 37.888 0.588 0.542 0.591
10:  2  TRUE  5000          3      4     0.001 0.3812878 21.057 0.510 0.479 0.615
11:  2  TRUE  2000          3     10     0.005 0.3790536 20.127 0.507 0.470 0.608
12:  2  TRUE  1000          5      4     0.001 0.3690911 18.981 0.500 0.475 0.611
13:  8  TRUE  5000          6     10     0.010 0.2865042 16.870 0.497 0.435 0.635
14:  0  TRUE  2000          6      4     0.010 0.3192862  9.779 0.460 0.433 0.621  

By some reason, it does not order the way pointed (probably because ordering by the groups). So, another ordering is done.

> dt2[order(G1, G2)]
    G1    G2 trees interDepth minObs shrinkage        qx   efet  prec  sens   acu
 1:  0 FALSE  2000          2      6     0.010 0.4395953 43.066 0.606 0.575 0.631
 2:  0 FALSE  2000          5      1     0.005 0.4294718 37.554 0.583 0.548 0.607
 3:  0 FALSE  5000          2      6     0.005 0.4395753 36.981 0.575 0.559 0.616
 4:  0  TRUE  2000          6      4     0.010 0.3192862  9.779 0.460 0.433 0.621
 5:  2 FALSE  5000          3      4     0.001 0.4296346 40.624 0.593 0.570 0.645
 6:  2 FALSE  1000          5      1     0.010 0.4186802 39.915 0.587 0.573 0.647
 7:  2 FALSE  2000          2      4     0.005 0.4390503 39.164 0.582 0.575 0.648
 8:  2  TRUE  5000          3      4     0.001 0.3812878 21.057 0.510 0.479 0.615
 9:  2  TRUE  2000          3     10     0.005 0.3790536 20.127 0.507 0.470 0.608
10:  2  TRUE  1000          5      4     0.001 0.3690911 18.981 0.500 0.475 0.611
11:  8 FALSE  2000          4     10     0.001 0.4511349 38.240 0.576 0.576 0.619
12:  8 FALSE  5000          2     10     0.005 0.4469665 38.064 0.579 0.566 0.611
13:  8 FALSE  5000          3      6     0.005 0.4426952 37.888 0.588 0.542 0.591
14:  8  TRUE  5000          6     10     0.010 0.2865042 16.870 0.497 0.435 0.635

Background service with location listener in android

First you need to create a Service. In that Service, create a class extending LocationListener. For this, use the following code snippet of Service:

public class LocationService extends Service {
public static final String BROADCAST_ACTION = "Hello World";
private static final int TWO_MINUTES = 1000 * 60 * 2;
public LocationManager locationManager;
public MyLocationListener listener;
public Location previousBestLocation = null;

Intent intent;
int counter = 0;

@Override
public void onCreate() {
    super.onCreate();
    intent = new Intent(BROADCAST_ACTION);
}

@Override
public void onStart(Intent intent, int startId) {
    locationManager = (LocationManager) getSystemService(Context.LOCATION_SERVICE);
    listener = new MyLocationListener();
    if (ActivityCompat.checkSelfPermission(this, Manifest.permission.ACCESS_FINE_LOCATION) != PackageManager.PERMISSION_GRANTED && ActivityCompat.checkSelfPermission(this, Manifest.permission.ACCESS_COARSE_LOCATION) != PackageManager.PERMISSION_GRANTED) {
        return;
    }
    locationManager.requestLocationUpdates(LocationManager.NETWORK_PROVIDER, 4000, 0, (LocationListener) listener);
    locationManager.requestLocationUpdates(LocationManager.GPS_PROVIDER, 4000, 0, listener);
}

@Override
public IBinder onBind(Intent intent)
{
    return null;
}

protected boolean isBetterLocation(Location location, Location currentBestLocation) {
    if (currentBestLocation == null) {
        // A new location is always better than no location
        return true;
    }

    // Check whether the new location fix is newer or older
    long timeDelta = location.getTime() - currentBestLocation.getTime();
    boolean isSignificantlyNewer = timeDelta > TWO_MINUTES;
    boolean isSignificantlyOlder = timeDelta < -TWO_MINUTES;
    boolean isNewer = timeDelta > 0;

    // If it's been more than two minutes since the current location, use the new location
    // because the user has likely moved
    if (isSignificantlyNewer) {
        return true;
        // If the new location is more than two minutes older, it must be worse
    } else if (isSignificantlyOlder) {
        return false;
    }

    // Check whether the new location fix is more or less accurate
    int accuracyDelta = (int) (location.getAccuracy() - currentBestLocation.getAccuracy());
    boolean isLessAccurate = accuracyDelta > 0;
    boolean isMoreAccurate = accuracyDelta < 0;
    boolean isSignificantlyLessAccurate = accuracyDelta > 200;

    // Check if the old and new location are from the same provider
    boolean isFromSameProvider = isSameProvider(location.getProvider(),
            currentBestLocation.getProvider());

    // Determine location quality using a combination of timeliness and accuracy
    if (isMoreAccurate) {
        return true;
    } else if (isNewer && !isLessAccurate) {
        return true;
    } else if (isNewer && !isSignificantlyLessAccurate && isFromSameProvider) {
        return true;
    }
    return false;
}



/** Checks whether two providers are the same */
private boolean isSameProvider(String provider1, String provider2) {
    if (provider1 == null) {
        return provider2 == null;
    }
    return provider1.equals(provider2);
}



@Override
public void onDestroy() {
    // handler.removeCallbacks(sendUpdatesToUI);     
    super.onDestroy();
    Log.v("STOP_SERVICE", "DONE");
    locationManager.removeUpdates(listener);
}

public static Thread performOnBackgroundThread(final Runnable runnable) {
    final Thread t = new Thread() {
        @Override
        public void run() {
            try {
                runnable.run();
            } finally {

            }
        }
    };
    t.start();
    return t;
}
public class MyLocationListener implements LocationListener
{

    public void onLocationChanged(final Location loc)
    {
        Log.i("*****", "Location changed");
        if(isBetterLocation(loc, previousBestLocation)) {
            loc.getLatitude();
            loc.getLongitude();
            intent.putExtra("Latitude", loc.getLatitude());
            intent.putExtra("Longitude", loc.getLongitude());
            intent.putExtra("Provider", loc.getProvider());
            sendBroadcast(intent);

        }
    }

    @Override
    public void onStatusChanged(String provider, int status, Bundle extras) {

    }

    public void onProviderDisabled(String provider)
    {
        Toast.makeText( getApplicationContext(), "Gps Disabled", Toast.LENGTH_SHORT ).show();
    }


    public void onProviderEnabled(String provider)
    {
        Toast.makeText( getApplicationContext(), "Gps Enabled", Toast.LENGTH_SHORT).show();
    }
}

Add this Service any where in your project, the way you want! :)

Iterator Loop vs index loop

Iterators make your code more generic.
Every standard library container provides an iterator hence if you change your container class in future the loop wont be affected.

How do I determine whether my calculation of pi is accurate?

The Taylor series is one way to approximate pi. As noted it converges slowly.

The partial sums of the Taylor series can be shown to be within some multiplier of the next term away from the true value of pi.

Other means of approximating pi have similar ways to calculate the max error.

We know this because we can prove it mathematically.

Excel VBA code to copy a specific string to clipboard

The simplest (Non Win32) way is to add a UserForm to your VBA project (if you don't already have one) or alternatively add a reference to Microsoft Forms 2 Object Library, then from a sheet/module you can simply:

With New MSForms.DataObject
    .SetText "http://zombo.com"
    .PutInClipboard
End With

Text on image mouseover?

And if you come from even further in the future you can use the title property on div tags now to provide tooltips:

<div title="Tooltip text">Hover over me</div>

Let's just hope you're not using a browser from the past.

_x000D_
_x000D_
<div title="Tooltip text">Hover over me</div>
_x000D_
_x000D_
_x000D_

TypeScript and field initializers

Update

Since writing this answer, better ways have come up. Please see the other answers below that have more votes and a better answer. I cannot remove this answer since it's marked as accepted.


Old answer

There is an issue on the TypeScript codeplex that describes this: Support for object initializers.

As stated, you can already do this by using interfaces in TypeScript instead of classes:

interface Name {
    first: string;
    last: string;
}
class Person {
    name: Name;
    age: number;
}

var bob: Person = {
    name: {
        first: "Bob",
        last: "Smith",
    },
    age: 35,
};

/lib/ld-linux.so.2: bad ELF interpreter: No such file or directory

These are the installation i had to run in order to make it work on fedora 22 :-

glibc-2.21-7.fc22.i686

alsa-lib-1.0.29-1.fc22.i686

qt3-3.3.8b-64.fc22.i686

libusb-1:0.1.5-5.fc22.i686

UICollectionView spacing margins

For adding margins to specified cells, you can use this custom flow layout. https://github.com/voyages-sncf-technologies/VSCollectionViewCellInsetFlowLayout/

extension ViewController : VSCollectionViewDelegateCellInsetFlowLayout 
{
    func collectionView(_ collectionView: UICollectionView, layout collectionViewLayout: UICollectionViewLayout, insetForItemAt indexPath: IndexPath) -> UIEdgeInsets {
        if indexPath.item == 0 {
            return UIEdgeInsets(top: 0, left: 0, bottom: 10, right: 0)
        }
        return UIEdgeInsets.zero
    }
}

Found 'OR 1=1/* sql injection in my newsletter database

'OR 1=1 is an attempt to make a query succeed no matter what
The /* is an attempt to start a multiline comment so the rest of the query is ignored.

An example would be

SELECT userid 
FROM users 
WHERE username = ''OR 1=1/*' 
    AND password = ''
    AND domain = ''

As you can see if you were to populate the username field without escaping the ' no matter what credentials the user passes in the query would return all userids in the system likely granting access to the attacker (possibly admin access if admin is your first user). You will also notice the remainder of the query would be commented out because of the /* including the real '.

The fact that you can see the value in your database means that it was escaped and that particular attack did not succeed. However, you should investigate if any other attempts were made.

What does the 'export' command do?

export in sh and related shells (such as bash), marks an environment variable to be exported to child-processes, so that the child inherits them.

export is defined in POSIX:

The shell shall give the export attribute to the variables corresponding to the specified names, which shall cause them to be in the environment of subsequently executed commands. If the name of a variable is followed by = word, then the value of that variable shall be set to word.

setValue:forUndefinedKey: this class is not key value coding-compliant for the key

This error is something else!

Here is how i Fixed it. I'm using xcode Version 6.1.1 and using swift. I got this error every time my app tried to perform a segue to jump to the next screen. Here what I did.

  1. Checked that the button was connected to the right action.(This wasn't the problem, but still good to check)
  2. Check that the button does not have any additional actions or outlets that you may have created by mistake. (This wasn't the problem, but still good to check)
  3. Check the logs and make sure that all the buttons in the NEXT SCREEN have the correct actions, and if there are any segues, make sure that they have a unique identifier. (This was the problem)
    • One of the segues did not have a unique identifier
    • One of the buttons had an action and two outlets that I created by mistake.

Delete any additional outlets and make sure that you the segues to the next screen have unique identifiers.

Cheers,

add controls vertically instead of horizontally using flow layout

I hope what you are trying to achieve is like this. For this please use Box layout.

package com.kcing.kailas.sample.client;

import javax.swing.BoxLayout;
import javax.swing.JCheckBox;
import javax.swing.JFrame;
import javax.swing.JPanel;
import javax.swing.SwingUtilities;
import javax.swing.UIManager;
import javax.swing.WindowConstants;

public class Testing extends JFrame {

    private JPanel jContentPane = null;

    public Testing() {
        super();
        initialize();
    }

    private void initialize() {
        this.setSize(300, 200);
        this.setContentPane(getJContentPane());
        this.setTitle("JFrame");
    }

    private JPanel getJContentPane() {
        if (jContentPane == null) {
            jContentPane = new JPanel();
            jContentPane.setLayout(null);

            JPanel panel = new JPanel();

            panel.setBounds(61, 11, 81, 140);
            panel.setLayout(new BoxLayout(panel, BoxLayout.Y_AXIS));
            jContentPane.add(panel);

            JCheckBox c1 = new JCheckBox("Check1");
            panel.add(c1);
            c1 = new JCheckBox("Check2");
            panel.add(c1);
            c1 = new JCheckBox("Check3");
            panel.add(c1);
            c1 = new JCheckBox("Check4");
            panel.add(c1);
        }
        return jContentPane;
    }

    public static void main(String[] args) throws Exception {
        Testing frame = new Testing();
        frame.setVisible(true);
        frame.setDefaultCloseOperation(WindowConstants.DISPOSE_ON_CLOSE);
    }
}

Using CSS in Laravel views?

To my opinion the best option to route to css & js use the following code:

<link rel="stylesheet" type="text/css" href="{{ URL::to('route/to/css') }}">

So if you have css file called main.css inside of css folder in public folder it should be the following:

<link rel="stylesheet" type="text/css" href="{{ URL::to('css/main.css') }}">

What does "&" at the end of a linux command mean?

I don’t know for sure but I’m reading a book right now and what I am getting is that a program need to handle its signal ( as when I press CTRL-C). Now a program can use SIG_IGN to ignore all signals or SIG_DFL to restore the default action.

Now if you do $ command & then this process running as background process simply ignores all signals that will occur. For foreground processes these signals are not ignored.

How to use css style in php

Cascading Style Sheets (CSS) is a style sheet language used for describing the presentation semantics (the look and formatting) of a document written in a markup language. more info : http://en.wikipedia.org/wiki/Cascading_Style_Sheets CSS is not a programming language, and does not have the tools that come with a server side language like PHP. However, we can use Server-side languages to generate style sheets.

<html>
<head>
<title>...</title>
<style type="text/css">
table {
margin: 8px;
}

th {
font-family: Arial, Helvetica, sans-serif;
font-size: .7em;
background: #666;
color: #FFF;
padding: 2px 6px;
border-collapse: separate;
border: 1px solid #000;
}

td {
font-family: Arial, Helvetica, sans-serif;
font-size: .7em;
border: 1px solid #DDD;
}
</style>
</head>
<body>
<?php>
echo "<table>";
echo "<tr><th>ID</th><th>hashtag</th></tr>";
while($row = mysql_fetch_row($result))
{
echo "<tr onmouseover=\"hilite(this)\" onmouseout=\"lowlite(this)\"><td>$row[0]</td>                <td>$row[1]</td></tr>\n";
}
echo "</table>";
?>
</body>
</html>

PHP Curl And Cookies

You can define different cookies for every user with CURLOPT_COOKIEFILE and CURLOPT_COOKIEJAR. Make different file for every user so each one would have it's own cookie-based session on remote server.

Regular expression for floating point numbers

In c notation, float number can occur in following shapes:

  1. 123
  2. 123.
  3. 123.24
  4. .24
  5. 2e-2 = 2 * 10 pow -2 = 2 * 0.1
  6. 4E+4 = 4 * 10 pow 4 = 4 * 10 000

For creating float regular expresion, I will first create "int regular expresion variable":

(([1-9][0-9]*)|0) will be int

Now, I will write small chunks of float regular expresion - solution is to concat those chunks with or simbol "|".

Chunks:

- (([+-]?{int}) satysfies case 1
- (([+-]?{int})"."[0-9]*)  satysfies cases 2 and 3
- ("."[0-9]*) satysfies case 4
- ([+-]?{int}[eE][+-]?{int}) satysfies cases 5 and 6

Final solution (concanating small chunks):

(([+-]?{int})|(([+-]?{int})"."[0-9]*)|("."[0-9]*)|([+-]?{int}[eE][+-]?{int})

Python threading.timer - repeat function every 'n' seconds

The best way is to start the timer thread once. Inside your timer thread you'd code the following

class MyThread(Thread):
    def __init__(self, event):
        Thread.__init__(self)
        self.stopped = event

    def run(self):
        while not self.stopped.wait(0.5):
            print("my thread")
            # call a function

In the code that started the timer, you can then set the stopped event to stop the timer.

stopFlag = Event()
thread = MyThread(stopFlag)
thread.start()
# this will stop the timer
stopFlag.set()

Error :Request header field Content-Type is not allowed by Access-Control-Allow-Headers

I know it's an old thread I worked with above answer and had to add:

header('Access-Control-Allow-Methods: GET, POST, PUT');

So my header looks like:

header('Access-Control-Allow-Origin: *');
header("Access-Control-Allow-Headers: Origin, X-Requested-With, Content-Type, Accept");
header('Access-Control-Allow-Methods: GET, POST, PUT');

And the problem was fixed.

Connection reset by peer: mod_fcgid: error reading data from FastCGI server

if you want to install a PHP version < 5.3.0, you must replace

--enable-cgi

with:

--enable-fastcgi

in your ./configure statement, excerpt from the php.net doc:

--enable-fastcgi

If this is enabled, the CGI module will be built with support for FastCGI also. Available since PHP 4.3.0

As of PHP 5.3.0 this argument no longer exists and is enabled by --enable-cgi instead. After the compilation the ./php-cgi -v should look like this:

PHP 5.2.17 (cgi-fcgi) (built: Jul  9 2013 18:28:12)
Copyright (c) 1997-2010 The PHP Group
Zend Engine v2.2.0, Copyright (c) 1998-2010 Zend Technologies

NOTICE THE (cgi-fcgi)

isset PHP isset($_GET['something']) ? $_GET['something'] : ''

It's commonly referred to as 'shorthand' or the Ternary Operator.

$test = isset($_GET['something']) ? $_GET['something'] : '';

means

if(isset($_GET['something'])) {
    $test = $_GET['something'];
} else {
    $test = '';
}

To break it down:

$test = ... // assign variable
isset(...) // test
? ... // if test is true, do ... (equivalent to if)
: ... // otherwise... (equivalent to else)

Or...

// test --v
if(isset(...)) { // if test is true, do ... (equivalent to ?)
    $test = // assign variable
} else { // otherwise... (equivalent to :)

android activity has leaked window com.android.internal.policy.impl.phonewindow$decorview Issue

Change this dialog.cancel(); to dialog.dismiss();

The solution is to call dismiss() on the Dialog you created in NetErrorPage.java:114 before exiting the Activity, e.g. in onPause().

Views have a reference to their parent Context (taken from constructor argument). If you leave an Activity without destroying Dialogs and other dynamically created Views, they still hold this reference to your Activity (if you created with this as Context: like new ProgressDialog(this)), so it cannot be collected by the GC, causing a memory leak.

How to read .pem file to get private and public key

Read public key from pem (PK or Cert). Depends on Bouncycastle.

private static PublicKey getPublicKeyFromPEM(Reader reader) throws IOException {

    PublicKey key;

    try (PEMParser pem = new PEMParser(reader)) {
        JcaPEMKeyConverter jcaPEMKeyConverter = new JcaPEMKeyConverter();
        Object pemContent = pem.readObject();
        if (pemContent instanceof PEMKeyPair) {
            PEMKeyPair pemKeyPair = (PEMKeyPair) pemContent;
            KeyPair keyPair = jcaPEMKeyConverter.getKeyPair(pemKeyPair);
            key = keyPair.getPublic();
        } else if (pemContent instanceof SubjectPublicKeyInfo) {
            SubjectPublicKeyInfo keyInfo = (SubjectPublicKeyInfo) pemContent;
            key = jcaPEMKeyConverter.getPublicKey(keyInfo);
        } else if (pemContent instanceof X509CertificateHolder) {
            X509CertificateHolder cert = (X509CertificateHolder) pemContent;
            key = jcaPEMKeyConverter.getPublicKey(cert.getSubjectPublicKeyInfo());
        } else {
            throw new IllegalArgumentException("Unsupported public key format '" +
                pemContent.getClass().getSimpleName() + '"');
        }
    }

    return key;
}

Read private key from PEM:

private static PrivateKey getPrivateKeyFromPEM(Reader reader) throws IOException {

    PrivateKey key;

    try (PEMParser pem = new PEMParser(reader)) {
        JcaPEMKeyConverter jcaPEMKeyConverter = new JcaPEMKeyConverter();
        Object pemContent = pem.readObject();
        if (pemContent instanceof PEMKeyPair) {
            PEMKeyPair pemKeyPair = (PEMKeyPair) pemContent;
            KeyPair keyPair = jcaPEMKeyConverter.getKeyPair(pemKeyPair);
            key = keyPair.getPrivate();
        } else if (pemContent instanceof PrivateKeyInfo) {
            PrivateKeyInfo privateKeyInfo = (PrivateKeyInfo) pemContent;
            key = jcaPEMKeyConverter.getPrivateKey(privateKeyInfo);
        } else {
            throw new IllegalArgumentException("Unsupported private key format '" +
                pemContent.getClass().getSimpleName() + '"');
        }
    }

    return key;
}

Dealing with "Xerces hell" in Java/Maven?

I know this doesn't answer the question exactly, but for ppl coming in from google that happen to use Gradle for their dependency management:

I managed to get rid of all xerces/Java8 issues with Gradle like this:

configurations {
    all*.exclude group: 'xml-apis'
    all*.exclude group: 'xerces'
}

Add CSS3 transition expand/collapse

This is my solution that adjusts the height automatically:

_x000D_
_x000D_
function growDiv() {_x000D_
  var growDiv = document.getElementById('grow');_x000D_
  if (growDiv.clientHeight) {_x000D_
    growDiv.style.height = 0;_x000D_
  } else {_x000D_
    var wrapper = document.querySelector('.measuringWrapper');_x000D_
    growDiv.style.height = wrapper.clientHeight + "px";_x000D_
  }_x000D_
  document.getElementById("more-button").value = document.getElementById("more-button").value == 'Read more' ? 'Read less' : 'Read more';_x000D_
}
_x000D_
#more-button {_x000D_
  border-style: none;_x000D_
  background: none;_x000D_
  font: 16px Serif;_x000D_
  color: blue;_x000D_
  margin: 0 0 10px 0;_x000D_
}_x000D_
_x000D_
#grow input:checked {_x000D_
  color: red;_x000D_
}_x000D_
_x000D_
#more-button:hover {_x000D_
  color: black;_x000D_
}_x000D_
_x000D_
#grow {_x000D_
  -moz-transition: height .5s;_x000D_
  -ms-transition: height .5s;_x000D_
  -o-transition: height .5s;_x000D_
  -webkit-transition: height .5s;_x000D_
  transition: height .5s;_x000D_
  height: 0;_x000D_
  overflow: hidden;_x000D_
}
_x000D_
<input type="button" onclick="growDiv()" value="Read more" id="more-button">_x000D_
_x000D_
<div id='grow'>_x000D_
  <div class='measuringWrapper'>_x000D_
    <div class="text">Here is some more text: Lorem ipsum dolor sit amet, consectetur adipiscing elit. Vestibulum vitae urna nulla. Vivamus a purus mi. In hac habitasse platea dictumst. In ac tempor quam. Vestibulum eleifend vehicula ligula, et cursus nisl gravida sit_x000D_
      amet. Pellentesque habitant morbi tristique senectus et netus et malesuada fames ac turpis egestas.</div>_x000D_
  </div>_x000D_
</div>
_x000D_
_x000D_
_x000D_

I used the workaround that r3bel posted: Can you use CSS3 to transition from height:0 to the variable height of content?

Call a child class method from a parent class object

A parent class should not have knowledge of child classes. You can implement a method calculate() and override it in every subclass:

class Person {
    String name;
    void getName(){...}
    void calculate();
}

and then

class Student extends Person{
    String class;
    void getClass(){...}

    @Override
    void calculate() {
        // do something with a Student
    }
}

and

class Teacher extends Person{
    String experience;
    void getExperience(){...}

    @Override
    void calculate() {
        // do something with a Student
    }

}

By the way. Your statement about abstract classes is confusing. You can call methods defined in an abstract class, but of course only of instances of subclasses.

In your example you can make Person abstract and the use getName() on instanced of Student and Teacher.

How to use pip with Python 3.x alongside Python 2.x

This worked for me on OS X: (I say this because sometimes is a pain that mac has "its own" version of every open source tool, and you cannot remove it because "its improvements" make it unique for other apple stuff to work, and if you remove it things start falling appart)

I followed the steps provided by @Lennart Regebro to get pip for python 3, nevertheless pip for python 2 was still first on the path, so... what I did is to create a symbolic link to python 3 inside /usr/bin (in deed I did the same to have my 2 pythons running in peace):

ln -s /Library/Frameworks/Python.framework/Versions/3.4/bin/pip /usr/bin/pip3

Notice that I added a 3 at the end, so basically what you have to do is to use pip3 instead of just pip.

The post is old but I hope this helps someone someday. this should theoretically work for any LINUX system.

postgresql COUNT(DISTINCT ...) very slow

-- My default settings (this is basically a single-session machine, so work_mem is pretty high)
SET effective_cache_size='2048MB';
SET work_mem='16MB';

\echo original
EXPLAIN ANALYZE
SELECT
        COUNT (distinct val) as aantal
FROM one
        ;

\echo group by+count(*)
EXPLAIN ANALYZE
SELECT
        distinct val
       -- , COUNT(*)
FROM one
GROUP BY val;

\echo with CTE
EXPLAIN ANALYZE
WITH agg AS (
    SELECT distinct val
    FROM one
    GROUP BY val
    )
SELECT COUNT (*) as aantal
FROM agg
        ;

Results:

original                                                      QUERY PLAN                                                      
----------------------------------------------------------------------------------------------------------------------
 Aggregate  (cost=36448.06..36448.07 rows=1 width=4) (actual time=1766.472..1766.472 rows=1 loops=1)
   ->  Seq Scan on one  (cost=0.00..32698.45 rows=1499845 width=4) (actual time=31.371..185.914 rows=1499845 loops=1)
 Total runtime: 1766.642 ms
(3 rows)

group by+count(*)
                                                         QUERY PLAN                                                         
----------------------------------------------------------------------------------------------------------------------------
 HashAggregate  (cost=36464.31..36477.31 rows=1300 width=4) (actual time=412.470..412.598 rows=1300 loops=1)
   ->  HashAggregate  (cost=36448.06..36461.06 rows=1300 width=4) (actual time=412.066..412.203 rows=1300 loops=1)
         ->  Seq Scan on one  (cost=0.00..32698.45 rows=1499845 width=4) (actual time=26.134..166.846 rows=1499845 loops=1)
 Total runtime: 412.686 ms
(4 rows)

with CTE
                                                             QUERY PLAN                                                             
------------------------------------------------------------------------------------------------------------------------------------
 Aggregate  (cost=36506.56..36506.57 rows=1 width=0) (actual time=408.239..408.239 rows=1 loops=1)
   CTE agg
     ->  HashAggregate  (cost=36464.31..36477.31 rows=1300 width=4) (actual time=407.704..407.847 rows=1300 loops=1)
           ->  HashAggregate  (cost=36448.06..36461.06 rows=1300 width=4) (actual time=407.320..407.467 rows=1300 loops=1)
                 ->  Seq Scan on one  (cost=0.00..32698.45 rows=1499845 width=4) (actual time=24.321..165.256 rows=1499845 loops=1)
       ->  CTE Scan on agg  (cost=0.00..26.00 rows=1300 width=0) (actual time=407.707..408.154 rows=1300 loops=1)
     Total runtime: 408.300 ms
    (7 rows)

The same plan as for the CTE could probably also be produced by other methods (window functions)

ToggleClass animate jQuery?

I attempted to use the toggleClass method to hide an item on my site (using visibility:hidden as opposed to display:none) with a slight animation, but for some reason the animation would not work (possibly due to an older version of jQuery UI).

The class was removed and added correctly, but the duration I added did not seem to make any difference - the item was simply added or removed with no effect.

So to resolve this I used a second class in my toggle method and applied a CSS transition instead:

The CSS:

.hidden{
    visibility:hidden;
    opacity: 0;
    -moz-transition: opacity 1s, visibility 1.3s;
    -webkit-transition: opacity 1s, visibility 1.3s;
    -o-transition: opacity 1s, visibility 1.3s;
    transition: opacity 1s, visibility 1.3s;
}
.shown{
    visibility:visible;
    opacity: 1;
    -moz-transition: opacity 1s, visibility 1.3s;
    -webkit-transition: opacity 1s, visibility 1.3s;
    -o-transition: opacity 1s, visibility 1.3s;
    transition: opacity 1s, visibility 1.3s;
}

The JS:

    function showOrHide() {
        $('#element').toggleClass("hidden shown");
    }

Thanks @tomas.satinsky for the awesome (and super simple) answer on this post.

Mockito, JUnit and Spring

Here's my short summary.

If you want to write a unit test, don't use a Spring applicationContext because you don't want any real dependencies injected in the class you are unit testing. Instead use mocks, either with the @RunWith(MockitoJUnitRunner.class) annotation on top of the class, or with MockitoAnnotations.initMocks(this) in the @Before method.

If you want to write an integration test, use:

@RunWith(SpringJUnit4ClassRunner.class)
@ContextConfiguration("yourTestApplicationContext.xml")

To set up your application context with an in-memory database for example. Normally you don't use mocks in integration tests, but you could do it by using the MockitoAnnotations.initMocks(this) approach described above.

PHP Curl UTF-8 Charset

You Can use this header

   header('Content-type: text/html; charset=UTF-8');

and after decoding the string

 $page = utf8_decode(curl_exec($ch));

It worked for me

Inserting image into IPython notebook markdown

minrk's answer is right.

However, I found that the images appeared broken in Print View (on my Windows machine running the Anaconda distribution of IPython version 0.13.2 in a Chrome browser)

The workaround for this was to use <img src="../files/image.png"> instead.

This made the image appear correctly in both Print View and the normal iPython editing view.

UPDATE: as of my upgrade to iPython v1.1.0 there is no more need for this workaround since the print view no longer exists. In fact, you must avoid this workaround since it prevents the nbconvert tool from finding the files.

Twitter Bootstrap modal on mobile devices

My solution...

Ver en jsfiddle

//Fix modal mobile Boostrap 3
function Show(id){
    //Fix CSS
    $(".modal-footer").css({"padding":"19px 20px 20px","margin-top":"15px","text-align":"right","border-top":"1px solid #e5e5e5"});
    $(".modal-body").css("overflow-y","auto");
    //Fix .modal-body height
    $('#'+id).on('shown.bs.modal',function(){
        $("#"+id+">.modal-dialog>.modal-content>.modal-body").css("height","auto");
        h1=$("#"+id+">.modal-dialog").height();
        h2=$(window).height();
        h3=$("#"+id+">.modal-dialog>.modal-content>.modal-body").height();
        h4=h2-(h1-h3);      
        if($(window).width()>=768){
            if(h1>h2){
                $("#"+id+">.modal-dialog>.modal-content>.modal-body").height(h4);
            }
            $("#"+id+">.modal-dialog").css("margin","30px auto");
            $("#"+id+">.modal-dialog>.modal-content").css("border","1px solid rgba(0,0,0,0.2)");
            $("#"+id+">.modal-dialog>.modal-content").css("border-radius",6);               
            if($("#"+id+">.modal-dialog").height()+30>h2){
                $("#"+id+">.modal-dialog").css("margin-top","0px");
                $("#"+id+">.modal-dialog").css("margin-bottom","0px");
            }
        }
        else{
            //Fix full-screen in mobiles
            $("#"+id+">.modal-dialog>.modal-content>.modal-body").height(h4);
            $("#"+id+">.modal-dialog").css("margin",0);
            $("#"+id+">.modal-dialog>.modal-content").css("border",0);
            $("#"+id+">.modal-dialog>.modal-content").css("border-radius",0);   
        }
        //Aply changes on screen resize (example: mobile orientation)
        window.onresize=function(){
            $("#"+id+">.modal-dialog>.modal-content>.modal-body").css("height","auto");
            h1=$("#"+id+">.modal-dialog").height();
            h2=$(window).height();
            h3=$("#"+id+">.modal-dialog>.modal-content>.modal-body").height();
            h4=h2-(h1-h3);
            if($(window).width()>=768){
                if(h1>h2){
                    $("#"+id+">.modal-dialog>.modal-content>.modal-body").height(h4);
                }
                $("#"+id+">.modal-dialog").css("margin","30px auto");
                $("#"+id+">.modal-dialog>.modal-content").css("border","1px solid rgba(0,0,0,0.2)");
                $("#"+id+">.modal-dialog>.modal-content").css("border-radius",6);               
                if($("#"+id+">.modal-dialog").height()+30>h2){
                    $("#"+id+">.modal-dialog").css("margin-top","0px");
                    $("#"+id+">.modal-dialog").css("margin-bottom","0px");
                }
            }
            else{
                //Fix full-screen in mobiles
                $("#"+id+">.modal-dialog>.modal-content>.modal-body").height(h4);
                $("#"+id+">.modal-dialog").css("margin",0);
                $("#"+id+">.modal-dialog>.modal-content").css("border",0);
                $("#"+id+">.modal-dialog>.modal-content").css("border-radius",0);   
            }
        };
    });  
    //Free event listener
    $('#'+id).on('hide.bs.modal',function(){
        window.onresize=function(){};
    });  
    //Mobile haven't scrollbar, so this is touch event scrollbar implementation
    var y1=0;
    var y2=0;
    var div=$("#"+id+">.modal-dialog>.modal-content>.modal-body")[0];
    div.addEventListener("touchstart",function(event){
        y1=event.touches[0].clientY;
    });
    div.addEventListener("touchmove",function(event){
        event.preventDefault();
        y2=event.touches[0].clientY;
        var limite=div.scrollHeight-div.clientHeight;
        var diff=div.scrollTop+y1-y2;
        if(diff<0)diff=0;
        if(diff>limite)diff=limite;
        div.scrollTop=diff;
        y1=y2;
    });
    //Fix position modal, scroll to top.    
    $('html, body').scrollTop(0);
    //Show
    $("#"+id).modal('show');
}

Tooltips with Twitter Bootstrap

Add this into your header

<script>
    //tooltip
    $(function() {
        var tooltips = $( "[title]" ).tooltip();
        $(document)(function() {
            tooltips.tooltip( "open" );
        });
    });
</script>

Then just add the attribute title="your tooltip" to any element

Design Documents (High Level and Low Level Design Documents)

High-Level Design (HLD) involves decomposing a system into modules, and representing the interfaces & invocation relationships among modules. An HLD is referred to as software architecture.

LLD, also known as a detailed design, is used to design internals of the individual modules identified during HLD i.e. data structures and algorithms of the modules are designed and documented.

Now, HLD and LLD are actually used in traditional Approach (Function-Oriented Software Design) whereas, in OOAD, the system is seen as a set of objects interacting with each other.

As per the above definitions, a high-level design document will usually include a high-level architecture diagram depicting the components, interfaces, and networks that need to be further specified or developed. The document may also depict or otherwise refer to work flows and/or data flows between component systems.

Class diagrams with all the methods and relations between classes come under LLD. Program specs are covered under LLD. LLD describes each and every module in an elaborate manner so that the programmer can directly code the program based on it. There will be at least 1 document for each module. The LLD will contain - a detailed functional logic of the module in pseudo code - database tables with all elements including their type and size - all interface details with complete API references(both requests and responses) - all dependency issues - error message listings - complete inputs and outputs for a module.

Node.js for() loop returning the same values at each loop

I would suggest doing this in a more functional style :P

function CreateMessageboard(BoardMessages) {
  var htmlMessageboardString = BoardMessages
   .map(function(BoardMessage) {
     return MessageToHTMLString(BoardMessage);
   })
   .join('');
}

Try this

git diff file against its last change

One of the ways to use git diff is:

git diff <commit> <path>

And a common way to refer one commit of the last commit is as a relative path to the actual HEAD. You can reference previous commits as HEAD^ (in your example this will be 123abc) or HEAD^^ (456def in your example), etc ...

So the answer to your question is:

git diff HEAD^^ myfile

Optimal way to DELETE specified rows from Oracle

In advance of my questions being answered, this is how I'd go about it:

Minimize the number of statements and the work they do issued in relative terms.

All scenarios assume you have a table of IDs (PURGE_IDS) to delete from TABLE_1, TABLE_2, etc.

Consider Using CREATE TABLE AS SELECT for really large deletes

If there's no concurrent activity, and you're deleting 30+ % of the rows in one or more of the tables, don't delete; perform a create table as select with the rows you wish to keep, and swap the new table out for the old table. INSERT /*+ APPEND */ ... NOLOGGING is surprisingly cheap if you can afford it. Even if you do have some concurrent activity, you may be able to use Online Table Redefinition to rebuild the table in-place.

Don't run DELETE statements you know won't delete any rows

If an ID value exists in at most one of the six tables, then keep track of which IDs you've deleted - and don't try to delete those IDs from any of the other tables.

CREATE TABLE TABLE1_PURGE NOLOGGING
AS 
SELECT ID FROM PURGE_IDS INNER JOIN TABLE_1 ON PURGE_IDS.ID = TABLE_1.ID;

DELETE FROM TABLE1 WHERE ID IN (SELECT ID FROM TABLE1_PURGE);

DELETE FROM PURGE_IDS WHERE ID IN (SELECT ID FROM TABLE1_PURGE);

DROP TABLE TABLE1_PURGE;

and repeat.

Manage Concurrency if you have to

Another way is to use PL/SQL looping over the tables, issuing a rowcount-limited delete statement. This is most likely appropriate if there's significant insert/update/delete concurrent load against the tables you're running the deletes against.

declare
  l_sql varchar2(4000);
begin
  for i in (select table_name from all_tables 
             where table_name in ('TABLE_1', 'TABLE_2', ...)
             order by table_name);
  loop
    l_sql := 'delete from ' || i.table_name || 
             ' where id in (select id from purge_ids) ' || 
             '   and rownum <= 1000000';
    loop
      commit;
      execute immediate l_sql;
      exit when sql%rowcount <> 1000000;  -- if we delete less than 1,000,000
    end loop;                             -- no more rows need to be deleted!
  end loop;
  commit;
end;

How to resolve cURL Error (7): couldn't connect to host?

“CURL ERROR 7 Failed to connect to Permission denied” error is caused, when for any reason curl request is blocked by some firewall or similar thing.

you will face this issue when ever the curl request is not with standard ports.

for example if you do curl to some URL which is on port 1234, you will face this issue where as URL with port 80 will give you results easily.

Most commonly this error has been seen on CentOS and any other OS with ‘SElinux’.

you need to either disable or change ’SElinux’ to permissive

have a look on this one

http://www.akashif.co.uk/php/curl-error-7-failed-to-connect-to-permission-denied

Hope this helps

using wildcards in LDAP search filters/queries

This should work, at least according to the Search Filter Syntax article on MSDN network.

The "hang-up" you have noticed is probably just a delay. Try running the same query with narrower scope (for example the specific OU where the test object is located), as it may take very long time for processing if you run it against all AD objects.

You may also try separating the filter into two parts:

(|(displayName=*searchstring)(displayName=searchstring*))

How to remove origin from git repository

Fairly straightforward:

git remote rm origin

As for the filter-branch question - just add --prune-empty to your filter branch command and it'll remove any revision that doesn't actually contain any changes in your resulting repo:

git filter-branch --prune-empty --subdirectory-filter path/to/subtree HEAD

How do I add a Maven dependency in Eclipse?

I have faced same problem with maven dependencies, eg: unfortunetly your maven dependencies deleted from your buildpath,then you people get lot of exceptions,if you follow below process you can easily resolve this issue.

 Right click on project >> maven >> updateProject >> selectProject >> OK

How does numpy.histogram() work?

Another useful thing to do with numpy.histogram is to plot the output as the x and y coordinates on a linegraph. For example:

arr = np.random.randint(1, 51, 500)
y, x = np.histogram(arr, bins=np.arange(51))
fig, ax = plt.subplots()
ax.plot(x[:-1], y)
fig.show()

enter image description here

This can be a useful way to visualize histograms where you would like a higher level of granularity without bars everywhere. Very useful in image histograms for identifying extreme pixel values.

PHP Session Destroy on Log Out Button

// logout

if(isset($_GET['logout'])) {
    session_destroy();
    unset($_SESSION['username']);
    header('location:login.php');
}

?>

How to use PrimeFaces p:fileUpload? Listener method is never invoked or UploadedFile is null / throws an error / not usable

I had same issue with primefaces 5.3 and I went through all the points described by BalusC with no result. I followed his advice of debugging FileUploadRenderer#decode() and I discovered that my web.xml was unproperly set

<context-param>
  <param-name>primefaces.UPLOADER</param-name>
  <param-value>auto|native|commons</param-value>
</context-param>

The param-value must be 1 of these 3 values but not all of them!! The whole context-param section can be removed and the default will be auto

What are the undocumented features and limitations of the Windows FINDSTR command?

The findstr command sets the ErrorLevel (or exit code) to one of the following values, given that there are no invalid or incompatible switches and no search string exceeds the applicable length limit:

  • 0 when at least a single match is encountered in one line throughout all specified files;
  • 1 otherwise;

A line is considered to contain a match when:

  • no /V option is given and the search expression occurs at least once;
  • the /V option is given and the search expression does not occur;

This means that the /V option also changes the returned ErrorLevel, but it does not just revert it!

For example, when you have got a file test.txt with two lines, one of which contains the string text but the other one does not, both findstr "text" "test.txt" and findstr /V "text" "test.txt" return an ErrorLevel of 0.

Basically you can say: if findstr returns at least a line, ErrorLevel is set to 0, else to 1.

Note that the /M option does not affect the ErrorLevel value, it just alters the output.

(Just for the sake of completeness: the find command behaves exactly the same way with respect to the /V option and ErrorLevel; the /C option does not affect ErrorLevel.)

How do I detect whether 32-bit Java is installed on x64 Windows, only looking at the filesystem and registry?

I tried both the 32-bit and 64-bit installers of both Oracle and IBM Java on Windows, and the presence of C:\Windows\SysWOW64\java.exe seems to be a reliable way to determine that 32-bit Java is available. I haven't tested older versions of these installers, but this at least looks like it should be a reliable way to test, for the most recent versions of Java.

How to move Jenkins from one PC to another

Sometimes we may not have access to a Jenkins machine to copy a folder directly into another Jenkins instance. So I wrote a menu driven utility which uses Jenkins REST API calls to install plugins and jobs from one Jenkins instance to another.

For plugin migration:

  1. GET request: {SOURCE_JENKINS_SERVER}/pluginManager/api/json?depth=1 will get you the list of plugins installed with their version.
  2. You can send a POST request with the following parameters to install these plugins.

    final_url=`{DESTINATION_JENKINS_SERVER}/pluginManager/installNecessaryPlugins`
    
    data=`<jenkins><install plugin="{PLUGIN_NAME}@latest"/></jenkins>` (where, latest will fetch the latest version of the plugin_name)
    
    auth=`(destination_jenkins_username, destination_jenkins_password)`
    
    header=`{crumb_field:crumb_value,"Content-Type":"application/xml”}` (where crumb_field=Jenkins-Crumb and get crumb value using API call {DESTINATION_JENKINS_SERVER}/crumbIssuer/api/json
    

For job migration:

  1. You can get the list of jobs installed on {SOURCE_JENKINS_URL} using a REST call, {SOURCE_JENKINS_URL}/view/All/api/json
  2. Then you can get each job config.xml file from the jobs on {SOURCE_JENKINS_URL} using the job URL {SOURCE_JENKINS_URL}/job/{JOB_NAME}.
  3. Use this config.xml file to POST the content of the XML file on {DESTINATION_JENKINS_URL} and that will create a job on {DESTINATION_JENKINS_URL}.

I have created a menu-driven utility in Python which asks the user to start plugin or Jenkins migration and uses Jenkins REST API calls to do it.

You can refer the JenkinsMigration.docx from this URL

Error message "unreported exception java.io.IOException; must be caught or declared to be thrown"

Exceptions bubble up the stack. If a caller calls a method that throws a checked exception, like IOException, it must also either catch the exception, or itself throw it.

In the case of the first block:

filecontent()
{
    setGUI();
    setRegister();
    showfile();
    setTitle("FileData");
    setVisible(true);
    setSize(300, 300);

    /*
        addWindowListener(new WindowAdapter()
        {
            public void windowClosing(WindowEvent we)
            {
                System.exit(0);
            }
        });
    */
}

You would have to include a try catch block:

filecontent()
{
    setGUI();
    setRegister();
    try {
        showfile();
    }
    catch (IOException e) {
        // Do something here
    }
    setTitle("FileData");
    setVisible(true);
    setSize(300, 300);

    /*
        addWindowListener(new WindowAdapter()
        {
            public void windowClosing(WindowEvent we)
            {
                System.exit(0);
            }
        });
    */
}

In the case of the second:

public void actionPerformed(ActionEvent ae)
{
    if (ae.getSource() == submit)
    {
        showfile();
    }
}

You cannot throw IOException from this method as its signature is determined by the interface, so you must catch the exception within:

public void actionPerformed(ActionEvent ae)
{
    if(ae.getSource()==submit)
    {
        try {
            showfile();
        }
        catch (IOException e) {
            // Do something here
        }
    }
}

Remember, the showFile() method is throwing the exception; that's what the "throws" keyword indicates that the method may throw that exception. If the showFile() method is throwing, then whatever code calls that method must catch, or themselves throw the exception explicitly by including the same throws IOException addition to the method signature, if it's permitted.

If the method is overriding a method signature defined in an interface or superclass that does not also declare that the method may throw that exception, you cannot declare it to throw an exception.

PHP-FPM doesn't write to error log

in my case I show that the error log was going to /var/log/php-fpm/www-error.log . so I commented this line in /etc/php-fpm.d/www.conf

php_flag[display_errors]   is commented
php_flag[display_errors] = on  log will be at /var/log/php-fpm/www-error.log

and as said above I also uncommented this line

catch_workers_output = yes

Now I can see logs in the file specified by nginx.

How can I make my website's background transparent without making the content (images & text) transparent too?

Just include following in your code

<body background="C:\Users\Desktop\images.jpg"> 

if you want to specify the size and opacity you can use following

<p><img style="opacity:0.9;" src="C:\Users\Desktop\images.jpg" width="300" height="231" alt="Image" /></p> 

Python: Generate random number between x and y which is a multiple of 5

>>> import random
>>> random.randrange(5,60,5)

should work in any Python >= 2.

Factorial using Recursion in Java

The key point that you missing here is that the variable "result" is a stack variable, and as such it does not get "replaced". To elaborate, every time fact is called, a NEW variable called "result" is created internally in the interpreter and linked to that invocation of the methods. This is in contrast of object fields which linked to the instance of the object and not a specific method call

What are the differences between using the terminal on a mac vs linux?

If you did a new or clean install of OS X version 10.3 or more recent, the default user terminal shell is bash.

Bash is essentially an enhanced and GNU freeware version of the original Bourne shell, sh. If you have previous experience with bash (often the default on GNU/Linux installations), this makes the OS X command-line experience familiar, otherwise consider switching your shell either to tcsh or to zsh, as some find these more user-friendly.

If you upgraded from or use OS X version 10.2.x, 10.1.x or 10.0.x, the default user shell is tcsh, an enhanced version of csh('c-shell'). Early implementations were a bit buggy and the programming syntax a bit weird so it developed a bad rap.

There are still some fundamental differences between mac and linux as Gordon Davisson so aptly lists, for example no useradd on Mac and ifconfig works differently.

The following table is useful for knowing the various unix shells.

sh      The original Bourne shell   Present on every unix system 
ksh     Original Korn shell         Richer shell programming environment than sh 
csh     Original C-shell            C-like syntax; early versions buggy 
tcsh    Enhanced C-shell            User-friendly and less buggy csh implementation 
bash    GNU Bourne-again shell      Enhanced and free sh implementation 
zsh     Z shell                     Enhanced, user-friendly ksh-like shell

You may also find these guides helpful:

http://homepage.mac.com/rgriff/files/TerminalBasics.pdf

http://guides.macrumors.com/Terminal
http://www.ofb.biz/safari/article/476.html

On a final note, I am on Linux (Ubuntu 11) and Mac osX so I use bash and the thing I like the most is customizing the .bashrc (source'd from .bash_profile on OSX) file with aliases, some examples below. I now placed all my aliases in a separate .bash_aliases file and include it with:

if [ -f ~/.bash_aliases ]; then
    . ~/.bash_aliases
fi

in the .bashrc or .bash_profile file.

Note that this is an example of a mac-linux difference because on a Mac you can't have the --color=auto. The first time I did this (without knowing) I redefined ls to be invalid which was a bit alarming until I removed --auto-color !

You may also find https://unix.stackexchange.com/q/127799/10043 useful

# ~/.bash_aliases
# ls variants
#alias l='ls -CF' 
alias la='ls -A' 
alias l='ls -alFtr' 
alias lsd='ls -d .*' 
# Various
alias h='history | tail'
alias hg='history | grep'
alias mv='mv -i' 
alias zap='rm -i'
# One letter quickies:
alias p='pwd'
alias x='exit'
alias {ack,ak}='ack-grep'
# Directories
alias s='cd ..'
alias play='cd ~/play/'
# Rails
alias src='script/rails console'
alias srs='script/rails server'
alias raked='rake db:drop db:create db:migrate db:seed' 
alias rvm-restart='source '\''/home/durrantm/.rvm/scripts/rvm'\'''
alias rrg='rake routes | grep '
alias rspecd='rspec --drb '
#
# DropBox - syncd
WORKBASE="~/Dropbox/97_2012/work"
alias work="cd $WORKBASE"
alias code="cd $WORKBASE/ror/code"
#
# DropNot - NOT syncd !
WORKBASE_GIT="~/Dropnot"
alias {dropnot,not}="cd $WORKBASE_GIT"
alias {webs,ww}="cd $WORKBASE_GIT/webs"
alias {setups,docs}="cd $WORKBASE_GIT/setups_and_docs"
alias {linker,lnk}="cd $WORKBASE_GIT/webs/rails_v3/linker"
#
# git
alias {gsta,gst}='git status' 
# Warning: gst conflicts with gnu-smalltalk (when used).
alias {gbra,gb}='git branch'
alias {gco,go}='git checkout'
alias {gcob,gob}='git checkout -b '
alias {gadd,ga}='git add '
alias {gcom,gc}='git commit'
alias {gpul,gl}='git pull '
alias {gpus,gh}='git push '
alias glom='git pull origin master'
alias ghom='git push origin master'
alias gg='git grep '
#
# vim
alias v='vim'
#
# tmux
alias {ton,tn}='tmux set -g mode-mouse on'
alias {tof,tf}='tmux set -g mode-mouse off'
#
# dmc
alias {dmc,dm}='cd ~/Dropnot/webs/rails_v3/dmc/'
alias wf='cd ~/Dropnot/webs/rails_v3/dmc/dmWorkflow'
alias ws='cd ~/Dropnot/webs/rails_v3/dmc/dmStaffing'

How to remove carriage returns and new lines in Postgresql?

select regexp_replace(field, E'[\\n\\r\\u2028]+', ' ', 'g' )

I had the same problem in my postgres d/b, but the newline in question wasn't the traditional ascii CRLF, it was a unicode line separator, character U2028. The above code snippet will capture that unicode variation as well.

Update... although I've only ever encountered the aforementioned characters "in the wild", to follow lmichelbacher's advice to translate even more unicode newline-like characters, use this:

select regexp_replace(field, E'[\\n\\r\\f\\u000B\\u0085\\u2028\\u2029]+', ' ', 'g' )

Pass object to javascript function

Answering normajeans' question about setting default value. Create a defaults object with same properties and merge with the arguments object

If using ES6:

    function yourFunction(args){
        let defaults = {opt1: true, opt2: 'something'};
        let params = {...defaults, ...args}; // right-most object overwrites 
        console.log(params.opt1);
    }

Older Browsers using Object.assign(target, source):

    function yourFunction(args){
        var defaults = {opt1: true, opt2: 'something'};
        var params = Object.assign(defaults, args) // args overwrites as it is source
        console.log(params.opt1);
    }

Using Jquery AJAX function with datatype HTML

Here is a version that uses dataType html, but this is far less explicit, because i am returning an empty string to indicate an error.

Ajax call:

$.ajax({
  type : 'POST',
  url : 'post.php',
  dataType : 'html',
  data: {
      email : $('#email').val()
  },
  success : function(data){
      $('#waiting').hide(500);
      $('#message').removeClass().addClass((data == '') ? 'error' : 'success')
     .html(data).show(500);
      if (data == '') {
          $('#message').html("Format your email correcly");
          $('#demoForm').show(500);
      }
  },
  error : function(XMLHttpRequest, textStatus, errorThrown) {
      $('#waiting').hide(500);
      $('#message').removeClass().addClass('error')
      .text('There was an error.').show(500);
      $('#demoForm').show(500);
  }

});

post.php

<?php
sleep(1);

function processEmail($email) {
    if (preg_match("#^[a-zA-Z0-9_.-]+@[a-zA-Z0-9-]+.[a-zA-Z0-9-.]+$#", $email)) {
        // your logic here (ex: add into database)
        return true;
    }
    return false;
}

if (processEmail($_POST['email'])) {
    echo "<span>Your email is <strong>{$_POST['email']}</strong></span>";
}

How do you make Git work with IntelliJ?

git.exe is common for any git based applications like GitHub, Bitbucket etc. Some times it is possible that you have already installed another git based application so git.exe will be present in the bin folder of that application.

For example if you installed bitbucket before github in your PC, you will find git.exe in C:\Users\{username}\AppData\Local\Atlassian\SourceTree\git_local\bin instead of C:\Users\{username}\AppData\Local\GitHub\PortableGit.....\bin.

ld.exe: cannot open output file ... : Permission denied

Got the same issue. Read this. Disabled the antivirus software (mcaffee). Et voila

Confirmed by the antivirus log:

Blocked by Access Protection rule d:\mingw64\x86_64-w64-mingw32\bin\ld.exe d:\workspace\cpp\bar\foo.exe User-defined Rules:ctx3 Action blocked : Create

How to convert WebResponse.GetResponseStream return into a string?

As @Heinzi mentioned the character set of the response should be used.

var encoding = response.CharacterSet == ""
    ? Encoding.UTF8
    : Encoding.GetEncoding(response.CharacterSet);

using (var stream = response.GetResponseStream())
{
    var reader = new StreamReader(stream, encoding);
    var responseString = reader.ReadToEnd();
}

Java check to see if a variable has been initialized

Assuming you're interested in whether the variable has been explicitly assigned a value or not, the answer is "not really". There's absolutely no difference between a field (instance variable or class variable) which hasn't been explicitly assigned at all yet, and one which has been assigned its default value - 0, false, null etc.

Now if you know that once assigned, the value will never reassigned a value of null, you can use:

if (box != null) {
    box.removeFromCanvas();
}

(and that also avoids a possible NullPointerException) but you need to be aware that "a field with a value of null" isn't the same as "a field which hasn't been explicitly assigned a value". Null is a perfectly valid variable value (for non-primitive variables, of course). Indeed, you may even want to change the above code to:

if (box != null) {
    box.removeFromCanvas();
    // Forget about the box - we don't want to try to remove it again
    box = null;
}

The difference is also visible for local variables, which can't be read before they've been "definitely assigned" - but one of the values which they can be definitely assigned is null (for reference type variables):

// Won't compile
String x;
System.out.println(x);

// Will compile, prints null
String y = null;
System.out.println(y);

Android webview slow

I think the following works best:

if (Build.VERSION.SDK_INT >= Build.VERSION_CODES.KITKAT) {
    webView.setLayerType(View.LAYER_TYPE_HARDWARE, null);
} else {
    webView.setLayerType(View.LAYER_TYPE_SOFTWARE, null);
}

Android 19 has Chromium engine for WebView. I guess it works better with hardware acceleration.

Change image source with JavaScript

The problem was that you were using .src without needing it and you also needed to differentiate which img you wanted to change.

function changeImage(a, imgid) {
    document.getElementById(imgid).src=a;
}
<div id="main_img">
    <img id="img" src="1772031_29_b.jpg">
</div>
<div id="thumb_img">
    <img id="1" src='1772031_29_t.jpg'  onclick='changeImage("1772031_29_b.jpg", "1");'>
    <img id="2" src='1772031_55_t.jpg'  onclick='changeImage("1772031_55_b.jpg", "2");'>
    <img id="3" src='1772031_53_t.jpg'  onclick='changeImage("1772031_53_b.jpg", "3");'>
</div>

How do I import from Excel to a DataSet using Microsoft.Office.Interop.Excel?

object[,] valueArray = (object[,])excelRange.get_Value(XlRangeValueDataType.xlRangeValueDefault);

//Get the column names
for (int k = 0; k < valueArray.GetLength(1); )
{
    //add columns to the data table.
    dt.Columns.Add((string)valueArray[1,++k]);
}

//Load data into data table
object[] singleDValue = new object[valueArray.GetLength(1)];
//value array first row contains column names. so loop starts from 1 instead of 0
for (int i = 1; i < valueArray.GetLength(0); i++)
{
    Console.WriteLine(valueArray.GetLength(0) + ":" + valueArray.GetLength(1));
    for (int k = 0; k < valueArray.GetLength(1); )
    {
        singleDValue[k] = valueArray[i+1, ++k];
    }
    dt.LoadDataRow(singleDValue, System.Data.LoadOption.PreserveChanges);
}

How to default to other directory instead of home directory

(Please read warning below)

Really simple way to do this in Windows (works with git bash, possibly others) is to create an environmental variable called HOME that points to your desired home directory.

  1. Right click on my computer, and choose properties
  2. Choose advanced system settings (location varies by Windows version)
  3. Within system properties, choose the advanced tab
  4. On the advanced tab, choose Environmental Variables (bottom button)
  5. Under "system variable" check to see if you already have a variable called HOME. If so, edit that variable by highlighting the variable name and clicking edit. Make the new variable name the desired path.
  6. If HOME does not already exist, click "new" under system variables and create a new variable called HOME whose value is desired path.

Environmental Variable

NOTE: This may change the way other things work. For example, for me it changes where my .ssh config files live. In my case, I wanted my home to be U:\, because that's my main place that I put project work and application settings (i.e. it really is my "home" directory).

EDIT June 23, 2017: This answer continues to get occasional upvotes, and I want to warn people that although this may "work", I agree with @AnthonyRaymond that it's not recommended. This is more of a temporary fix or a fix if you don't care if other things break. Changing your home won't cause active damage (like deleting your hard drive) but it's likely to cause insidious annoyances later. When you do start to have annoying problems down the road, you probably won't remember this change... so you're likely to be scratching your head later on!

Speed comparison with Project Euler: C vs Python vs Erlang vs Haskell

Trying GO:

package main

import "fmt"
import "math"

func main() {
    var n, m, c int
    for i := 1; ; i++ {
        n, m, c = i * (i + 1) / 2, int(math.Sqrt(float64(n))), 0
        for f := 1; f < m; f++ {
            if n % f == 0 { c++ }
    }
    c *= 2
    if m * m == n { c ++ }
    if c > 1001 {
        fmt.Println(n)
        break
        }
    }
}

I get:

original c version: 9.1690 100%
go: 8.2520 111%

But using:

package main

import (
    "math"
    "fmt"
 )

// Sieve of Eratosthenes
func PrimesBelow(limit int) []int {
    switch {
        case limit < 2:
            return []int{}
        case limit == 2:
            return []int{2}
    }
    sievebound := (limit - 1) / 2
    sieve := make([]bool, sievebound+1)
    crosslimit := int(math.Sqrt(float64(limit))-1) / 2
    for i := 1; i <= crosslimit; i++ {
        if !sieve[i] {
            for j := 2 * i * (i + 1); j <= sievebound; j += 2*i + 1 {
                sieve[j] = true
            }
        }
    }
    plimit := int(1.3*float64(limit)) / int(math.Log(float64(limit)))
    primes := make([]int, plimit)
    p := 1
    primes[0] = 2
    for i := 1; i <= sievebound; i++ {
        if !sieve[i] {
            primes[p] = 2*i + 1
            p++
            if p >= plimit {
                break
            }
        }
    }
    last := len(primes) - 1
    for i := last; i > 0; i-- {
        if primes[i] != 0 {
            break
        }
        last = i
    }
    return primes[0:last]
}



func main() {
    fmt.Println(p12())
}
// Requires PrimesBelow from utils.go
func p12() int {
    n, dn, cnt := 3, 2, 0
    primearray := PrimesBelow(1000000)
    for cnt <= 1001 {
        n++
        n1 := n
        if n1%2 == 0 {
            n1 /= 2
        }
        dn1 := 1
        for i := 0; i < len(primearray); i++ {
            if primearray[i]*primearray[i] > n1 {
                dn1 *= 2
                break
            }
            exponent := 1
            for n1%primearray[i] == 0 {
                exponent++
                n1 /= primearray[i]
            }
            if exponent > 1 {
                dn1 *= exponent
            }
            if n1 == 1 {
                break
            }
        }
        cnt = dn * dn1
        dn = dn1
    }
    return n * (n - 1) / 2
}

I get:

original c version: 9.1690 100%
thaumkid's c version: 0.1060 8650%
first go version: 8.2520 111%
second go version: 0.0230 39865%

I also tried Python3.6 and pypy3.3-5.5-alpha:

original c version: 8.629 100%
thaumkid's c version: 0.109 7916%
Python3.6: 54.795 16%
pypy3.3-5.5-alpha: 13.291 65%

and then with following code I got:

original c version: 8.629 100%
thaumkid's c version: 0.109 8650%
Python3.6: 1.489 580%
pypy3.3-5.5-alpha: 0.582 1483%

def D(N):
    if N == 1: return 1
    sqrtN = int(N ** 0.5)
    nf = 1
    for d in range(2, sqrtN + 1):
        if N % d == 0:
            nf = nf + 1
    return 2 * nf - (1 if sqrtN**2 == N else 0)

L = 1000
Dt, n = 0, 0

while Dt <= L:
    t = n * (n + 1) // 2
    Dt = D(n/2)*D(n+1) if n%2 == 0 else D(n)*D((n+1)/2)
    n = n + 1

print (t)

How can I simulate a print statement in MySQL?

to take output in MySQL you can use if statement SYNTAX:

if(condition,if_true,if_false)

the if_true and if_false can be used to verify and to show output as there is no print statement in the MySQL

embedding image in html email

It may be of interest that both Outlook and Outlook Express can generate these multipart image email formats, if you insert the image files using the Insert / Picture menu function.

Obviously the email type must be set to HTML (not plain text).

Any other method (e.g. drag/drop, or any command-line invocation) results in the image(s) being sent as an attachment.

If you then send such an email to yourself, you can see how it is formatted! :)

FWIW, I am looking for a standalone windows executable which does inline images from the command line mode, but there seem to be none. It's a path which many have gone up... One can do it with say Outlook Express, by passing it an appropriately formatted .eml file.

Basic Apache commands for a local Windows machine

Going back to absolute basics here. The answers on this page and a little googling have brought me to the following resolution to my issue. Steps to restart the apache service with Xampp installed:-

  1. Click the start button and type CMD (if on Windows Vista or later and Apache is installed as a service make sure this is an elevated command prompt)
  2. In the command window that appears type cd C:\xampp\apache\bin (the default installation path for Xampp)
  3. Then type httpd -k restart

I hope that this is of use to others just starting out with running a local Apache server.

Inversion of Control vs Dependency Injection

//ICO , DI ,10 years back , this was they way:

public class  AuditDAOImpl implements Audit{

    //dependency
    AuditDAO auditDAO = null;
        //Control of the AuditDAO is with AuditDAOImpl because its creating the object
    public AuditDAOImpl () {
        this.auditDAO = new AuditDAO ();
    }
}

Now with Spring 3,4 or latest its like below

public class  AuditDAOImpl implements Audit{

    //dependency

     //Now control is shifted to Spring. Container find the object and provide it. 
    @Autowired
    AuditDAO auditDAO = null;

}

Overall the control is inverted from old concept of coupled code to the frameworks like Spring which makes the object available. So that's IOC as far as I know and Dependency injection as you know when we inject the dependent object into another object using Constructor or setters . Inject basically means passing it as an argument. In spring we have XML & annotation based configuration where we define bean object and pass the dependent object with Constructor or setter injection style.

Failed to install *.apk on device 'emulator-5554': EOF

I was facing the same problem but i tried changing the ADB connection timeout. I think it defaults that to 5000ms and I changed mine to 10000ms to get rid of that problem. If you are in Eclipse, you can do this by going through Window -> Preferences and then it is in DDMS under Android.

Using grep to search for hex strings in a file

grep has a -P switch allowing to use perl regexp syntax the perl regex allows to look at bytes, using \x.. syntax.

so you can look for a given hex string in a file with: grep -aP "\xdf"

but the outpt won't be very useful; indeed better do a regexp on the hexdump output;

The grep -P can be useful however to just find files matrching a given binary pattern. Or to do a binary query of a pattern that actually happens in text (see for example How to regexp CJK ideographs (in utf-8) )

MySQL Trigger after update only if row has changed

You can do this by comparing each field using the NULL-safe equals operator <=> and then negating the result using NOT.

The complete trigger would become:

DROP TRIGGER IF EXISTS `my_trigger_name`;

DELIMITER $$

CREATE TRIGGER `my_trigger_name` AFTER UPDATE ON `my_table_name` FOR EACH ROW 
    BEGIN
        /*Add any fields you want to compare here*/
        IF !(OLD.a <=> NEW.a AND OLD.b <=> NEW.b) THEN
            INSERT INTO `my_other_table` (
                `a`,
                 `b`
            ) VALUES (
                NEW.`a`,
                NEW.`b`
            );
        END IF;
    END;$$

DELIMITER ;

(Based on a different answer of mine.)

How to send POST request in JSON using HTTPClient in Android?

In this answer I am using an example posted by Justin Grammens.

About JSON

JSON stands for JavaScript Object Notation. In JavaScript properties can be referenced both like this object1.name and like this object['name'];. The example from the article uses this bit of JSON.

The Parts
A fan object with email as a key and [email protected] as a value

{
  fan:
    {
      email : '[email protected]'
    }
}

So the object equivalent would be fan.email; or fan['email'];. Both would have the same value of '[email protected]'.

About HttpClient Request

The following is what our author used to make a HttpClient Request. I do not claim to be an expert at all this so if anyone has a better way to word some of the terminology feel free.

public static HttpResponse makeRequest(String path, Map params) throws Exception 
{
    //instantiates httpclient to make request
    DefaultHttpClient httpclient = new DefaultHttpClient();

    //url with the post data
    HttpPost httpost = new HttpPost(path);

    //convert parameters into JSON object
    JSONObject holder = getJsonObjectFromMap(params);

    //passes the results to a string builder/entity
    StringEntity se = new StringEntity(holder.toString());

    //sets the post request as the resulting string
    httpost.setEntity(se);
    //sets a request header so the page receving the request
    //will know what to do with it
    httpost.setHeader("Accept", "application/json");
    httpost.setHeader("Content-type", "application/json");

    //Handles what is returned from the page 
    ResponseHandler responseHandler = new BasicResponseHandler();
    return httpclient.execute(httpost, responseHandler);
}

Map

If you are not familiar with the Map data structure please take a look at the Java Map reference. In short, a map is similar to a dictionary or a hash.

private static JSONObject getJsonObjectFromMap(Map params) throws JSONException {

    //all the passed parameters from the post request
    //iterator used to loop through all the parameters
    //passed in the post request
    Iterator iter = params.entrySet().iterator();

    //Stores JSON
    JSONObject holder = new JSONObject();

    //using the earlier example your first entry would get email
    //and the inner while would get the value which would be '[email protected]' 
    //{ fan: { email : '[email protected]' } }

    //While there is another entry
    while (iter.hasNext()) 
    {
        //gets an entry in the params
        Map.Entry pairs = (Map.Entry)iter.next();

        //creates a key for Map
        String key = (String)pairs.getKey();

        //Create a new map
        Map m = (Map)pairs.getValue();   

        //object for storing Json
        JSONObject data = new JSONObject();

        //gets the value
        Iterator iter2 = m.entrySet().iterator();
        while (iter2.hasNext()) 
        {
            Map.Entry pairs2 = (Map.Entry)iter2.next();
            data.put((String)pairs2.getKey(), (String)pairs2.getValue());
        }

        //puts email and '[email protected]'  together in map
        holder.put(key, data);
    }
    return holder;
}

Please feel free to comment on any questions that arise about this post or if I have not made something clear or if I have not touched on something that your still confused about... etc whatever pops in your head really.

(I will take down if Justin Grammens does not approve. But if not then thanks Justin for being cool about it.)

Update

I just happend to get a comment about how to use the code and realized that there was a mistake in the return type. The method signature was set to return a string but in this case it wasnt returning anything. I changed the signature to HttpResponse and will refer you to this link on Getting Response Body of HttpResponse the path variable is the url and I updated to fix a mistake in the code.

Why does modern Perl avoid UTF-8 by default?

?:

  1. Set your PERL_UNICODE envariable to AS. This makes all Perl scripts decode @ARGV as UTF-8 strings, and sets the encoding of all three of stdin, stdout, and stderr to UTF-8. Both these are global effects, not lexical ones.

  2. At the top of your source file (program, module, library, dohickey), prominently assert that you are running perl version 5.12 or better via:

    use v5.12;  # minimal for unicode string feature
    use v5.14;  # optimal for unicode string feature
    
  3. Enable warnings, since the previous declaration only enables strictures and features, not warnings. I also suggest promoting Unicode warnings into exceptions, so use both these lines, not just one of them. Note however that under v5.14, the utf8 warning class comprises three other subwarnings which can all be separately enabled: nonchar, surrogate, and non_unicode. These you may wish to exert greater control over.

    use warnings;
    use warnings qw( FATAL utf8 );
    
  4. Declare that this source unit is encoded as UTF-8. Although once upon a time this pragma did other things, it now serves this one singular purpose alone and no other:

    use utf8;
    
  5. Declare that anything that opens a filehandle within this lexical scope but not elsewhere is to assume that that stream is encoded in UTF-8 unless you tell it otherwise. That way you do not affect other module’s or other program’s code.

    use open qw( :encoding(UTF-8) :std );
    
  6. Enable named characters via \N{CHARNAME}.

    use charnames qw( :full :short );
    
  7. If you have a DATA handle, you must explicitly set its encoding. If you want this to be UTF-8, then say:

    binmode(DATA, ":encoding(UTF-8)");
    

There is of course no end of other matters with which you may eventually find yourself concerned, but these will suffice to approximate the state goal to “make everything just work with UTF-8”, albeit for a somewhat weakened sense of those terms.

One other pragma, although it is not Unicode related, is:

      use autodie;

It is strongly recommended.


? ?


My own boilerplate these days tends to look like this:

use 5.014;

use utf8;
use strict;
use autodie;
use warnings; 
use warnings    qw< FATAL  utf8     >;
use open        qw< :std  :utf8     >;
use charnames   qw< :full >;
use feature     qw< unicode_strings >;

use File::Basename      qw< basename >;
use Carp                qw< carp croak confess cluck >;
use Encode              qw< encode decode >;
use Unicode::Normalize  qw< NFD NFC >;

END { close STDOUT }

if (grep /\P{ASCII}/ => @ARGV) { 
   @ARGV = map { decode("UTF-8", $_) } @ARGV;
}

$0 = basename($0);  # shorter messages
$| = 1;

binmode(DATA, ":utf8");

# give a full stack dump on any untrapped exceptions
local $SIG{__DIE__} = sub {
    confess "Uncaught exception: @_" unless $^S;
};

# now promote run-time warnings into stack-dumped
#   exceptions *unless* we're in an try block, in
#   which case just cluck the stack dump instead
local $SIG{__WARN__} = sub {
    if ($^S) { cluck   "Trapped warning: @_" } 
    else     { confess "Deadly warning: @_"  }
};

while (<>)  {
    chomp;
    $_ = NFD($_);
    ...
} continue {
    say NFC($_);
}

__END__


Saying that “Perl should [somehow!] enable Unicode by default” doesn’t even start to begin to think about getting around to saying enough to be even marginally useful in some sort of rare and isolated case. Unicode is much much more than just a larger character repertoire; it’s also how those characters all interact in many, many ways.

Even the simple-minded minimal measures that (some) people seem to think they want are guaranteed to miserably break millions of lines of code, code that has no chance to “upgrade” to your spiffy new Brave New World modernity.

It is way way way more complicated than people pretend. I’ve thought about this a huge, whole lot over the past few years. I would love to be shown that I am wrong. But I don’t think I am. Unicode is fundamentally more complex than the model that you would like to impose on it, and there is complexity here that you can never sweep under the carpet. If you try, you’ll break either your own code or somebody else’s. At some point, you simply have to break down and learn what Unicode is about. You cannot pretend it is something it is not.

goes out of its way to make Unicode easy, far more than anything else I’ve ever used. If you think this is bad, try something else for a while. Then come back to : either you will have returned to a better world, or else you will bring knowledge of the same with you so that we can make use of your new knowledge to make better at these things.


?


At a minimum, here are some things that would appear to be required for to “enable Unicode by default”, as you put it:

  1. All source code should be in UTF-8 by default. You can get that with use utf8 or export PERL5OPTS=-Mutf8.

  2. The DATA handle should be UTF-8. You will have to do this on a per-package basis, as in binmode(DATA, ":encoding(UTF-8)").

  3. Program arguments to scripts should be understood to be UTF-8 by default. export PERL_UNICODE=A, or perl -CA, or export PERL5OPTS=-CA.

  4. The standard input, output, and error streams should default to UTF-8. export PERL_UNICODE=S for all of them, or I, O, and/or E for just some of them. This is like perl -CS.

  5. Any other handles opened by should be considered UTF-8 unless declared otherwise; export PERL_UNICODE=D or with i and o for particular ones of these; export PERL5OPTS=-CD would work. That makes -CSAD for all of them.

  6. Cover both bases plus all the streams you open with export PERL5OPTS=-Mopen=:utf8,:std. See uniquote.

  7. You don’t want to miss UTF-8 encoding errors. Try export PERL5OPTS=-Mwarnings=FATAL,utf8. And make sure your input streams are always binmoded to :encoding(UTF-8), not just to :utf8.

  8. Code points between 128–255 should be understood by to be the corresponding Unicode code points, not just unpropertied binary values. use feature "unicode_strings" or export PERL5OPTS=-Mfeature=unicode_strings. That will make uc("\xDF") eq "SS" and "\xE9" =~ /\w/. A simple export PERL5OPTS=-Mv5.12 or better will also get that.

  9. Named Unicode characters are not by default enabled, so add export PERL5OPTS=-Mcharnames=:full,:short,latin,greek or some such. See uninames and tcgrep.

  10. You almost always need access to the functions from the standard Unicode::Normalize module various types of decompositions. export PERL5OPTS=-MUnicode::Normalize=NFD,NFKD,NFC,NFKD, and then always run incoming stuff through NFD and outbound stuff from NFC. There’s no I/O layer for these yet that I’m aware of, but see nfc, nfd, nfkd, and nfkc.

  11. String comparisons in using eq, ne, lc, cmp, sort, &c&cc are always wrong. So instead of @a = sort @b, you need @a = Unicode::Collate->new->sort(@b). Might as well add that to your export PERL5OPTS=-MUnicode::Collate. You can cache the key for binary comparisons.

  12. built-ins like printf and write do the wrong thing with Unicode data. You need to use the Unicode::GCString module for the former, and both that and also the Unicode::LineBreak module as well for the latter. See uwc and unifmt.

  13. If you want them to count as integers, then you are going to have to run your \d+ captures through the Unicode::UCD::num function because ’s built-in atoi(3) isn’t currently clever enough.

  14. You are going to have filesystem issues on filesystems. Some filesystems silently enforce a conversion to NFC; others silently enforce a conversion to NFD. And others do something else still. Some even ignore the matter altogether, which leads to even greater problems. So you have to do your own NFC/NFD handling to keep sane.

  15. All your code involving a-z or A-Z and such MUST BE CHANGED, including m//, s///, and tr///. It’s should stand out as a screaming red flag that your code is broken. But it is not clear how it must change. Getting the right properties, and understanding their casefolds, is harder than you might think. I use unichars and uniprops every single day.

  16. Code that uses \p{Lu} is almost as wrong as code that uses [A-Za-z]. You need to use \p{Upper} instead, and know the reason why. Yes, \p{Lowercase} and \p{Lower} are different from \p{Ll} and \p{Lowercase_Letter}.

  17. Code that uses [a-zA-Z] is even worse. And it can’t use \pL or \p{Letter}; it needs to use \p{Alphabetic}. Not all alphabetics are letters, you know!

  18. If you are looking for variables with /[\$\@\%]\w+/, then you have a problem. You need to look for /[\$\@\%]\p{IDS}\p{IDC}*/, and even that isn’t thinking about the punctuation variables or package variables.

  19. If you are checking for whitespace, then you should choose between \h and \v, depending. And you should never use \s, since it DOES NOT MEAN [\h\v], contrary to popular belief.

  20. If you are using \n for a line boundary, or even \r\n, then you are doing it wrong. You have to use \R, which is not the same!

  21. If you don’t know when and whether to call Unicode::Stringprep, then you had better learn.

  22. Case-insensitive comparisons need to check for whether two things are the same letters no matter their diacritics and such. The easiest way to do that is with the standard Unicode::Collate module. Unicode::Collate->new(level => 1)->cmp($a, $b). There are also eq methods and such, and you should probably learn about the match and substr methods, too. These are have distinct advantages over the built-ins.

  23. Sometimes that’s still not enough, and you need the Unicode::Collate::Locale module instead, as in Unicode::Collate::Locale->new(locale => "de__phonebook", level => 1)->cmp($a, $b) instead. Consider that Unicode::Collate::->new(level => 1)->eq("d", "ð") is true, but Unicode::Collate::Locale->new(locale=>"is",level => 1)->eq("d", " ð") is false. Similarly, "ae" and "æ" are eq if you don’t use locales, or if you use the English one, but they are different in the Icelandic locale. Now what? It’s tough, I tell you. You can play with ucsort to test some of these things out.

  24. Consider how to match the pattern CVCV (consonsant, vowel, consonant, vowel) in the string “niño”. Its NFD form — which you had darned well better have remembered to put it in — becomes “nin\x{303}o”. Now what are you going to do? Even pretending that a vowel is [aeiou] (which is wrong, by the way), you won’t be able to do something like (?=[aeiou])\X) either, because even in NFD a code point like ‘ø’ does not decompose! However, it will test equal to an ‘o’ using the UCA comparison I just showed you. You can’t rely on NFD, you have to rely on UCA.



And that’s not all. There are a million broken assumptions that people make about Unicode. Until they understand these things, their code will be broken.

  1. Code that assumes it can open a text file without specifying the encoding is broken.

  2. Code that assumes the default encoding is some sort of native platform encoding is broken.

  3. Code that assumes that web pages in Japanese or Chinese take up less space in UTF-16 than in UTF-8 is wrong.

  4. Code that assumes Perl uses UTF-8 internally is wrong.

  5. Code that assumes that encoding errors will always raise an exception is wrong.

  6. Code that assumes Perl code points are limited to 0x10_FFFF is wrong.

  7. Code that assumes you can set $/ to something that will work with any valid line separator is wrong.

  8. Code that assumes roundtrip equality on casefolding, like lc(uc($s)) eq $s or uc(lc($s)) eq $s, is completely broken and wrong. Consider that the uc("s") and uc("?") are both "S", but lc("S") cannot possibly return both of those.

  9. Code that assumes every lowercase code point has a distinct uppercase one, or vice versa, is broken. For example, "ª" is a lowercase letter with no uppercase; whereas both "?" and "?" are letters, but they are not lowercase letters; however, they are both lowercase code points without corresponding uppercase versions. Got that? They are not \p{Lowercase_Letter}, despite being both \p{Letter} and \p{Lowercase}.

  10. Code that assumes changing the case doesn’t change the length of the string is broken.

  11. Code that assumes there are only two cases is broken. There’s also titlecase.

  12. Code that assumes only letters have case is broken. Beyond just letters, it turns out that numbers, symbols, and even marks have case. In fact, changing the case can even make something change its main general category, like a \p{Mark} turning into a \p{Letter}. It can also make it switch from one script to another.

  13. Code that assumes that case is never locale-dependent is broken.

  14. Code that assumes Unicode gives a fig about POSIX locales is broken.

  15. Code that assumes you can remove diacritics to get at base ASCII letters is evil, still, broken, brain-damaged, wrong, and justification for capital punishment.

  16. Code that assumes that diacritics \p{Diacritic} and marks \p{Mark} are the same thing is broken.

  17. Code that assumes \p{GC=Dash_Punctuation} covers as much as \p{Dash} is broken.

  18. Code that assumes dash, hyphens, and minuses are the same thing as each other, or that there is only one of each, is broken and wrong.

  19. Code that assumes every code point takes up no more than one print column is broken.

  20. Code that assumes that all \p{Mark} characters take up zero print columns is broken.

  21. Code that assumes that characters which look alike are alike is broken.

  22. Code that assumes that characters which do not look alike are not alike is broken.

  23. Code that assumes there is a limit to the number of code points in a row that just one \X can match is wrong.

  24. Code that assumes \X can never start with a \p{Mark} character is wrong.

  25. Code that assumes that \X can never hold two non-\p{Mark} characters is wrong.

  26. Code that assumes that it cannot use "\x{FFFF}" is wrong.

  27. Code that assumes a non-BMP code point that requires two UTF-16 (surrogate) code units will encode to two separate UTF-8 characters, one per code unit, is wrong. It doesn’t: it encodes to single code point.

  28. Code that transcodes from UTF-16 or UTF-32 with leading BOMs into UTF-8 is broken if it puts a BOM at the start of the resulting UTF-8. This is so stupid the engineer should have their eyelids removed.

  29. Code that assumes the CESU-8 is a valid UTF encoding is wrong. Likewise, code that thinks encoding U+0000 as "\xC0\x80" is UTF-8 is broken and wrong. These guys also deserve the eyelid treatment.

  30. Code that assumes characters like > always points to the right and < always points to the left are wrong — because they in fact do not.

  31. Code that assumes if you first output character X and then character Y, that those will show up as XY is wrong. Sometimes they don’t.

  32. Code that assumes that ASCII is good enough for writing English properly is stupid, shortsighted, illiterate, broken, evil, and wrong. Off with their heads! If that seems too extreme, we can compromise: henceforth they may type only with their big toe from one foot. (The rest will be duct taped.)

  33. Code that assumes that all \p{Math} code points are visible characters is wrong.

  34. Code that assumes \w contains only letters, digits, and underscores is wrong.

  35. Code that assumes that ^ and ~ are punctuation marks is wrong.

  36. Code that assumes that ü has an umlaut is wrong.

  37. Code that believes things like ? contain any letters in them is wrong.

  38. Code that believes \p{InLatin} is the same as \p{Latin} is heinously broken.

  39. Code that believe that \p{InLatin} is almost ever useful is almost certainly wrong.

  40. Code that believes that given $FIRST_LETTER as the first letter in some alphabet and $LAST_LETTER as the last letter in that same alphabet, that [${FIRST_LETTER}-${LAST_LETTER}] has any meaning whatsoever is almost always complete broken and wrong and meaningless.

  41. Code that believes someone’s name can only contain certain characters is stupid, offensive, and wrong.

  42. Code that tries to reduce Unicode to ASCII is not merely wrong, its perpetrator should never be allowed to work in programming again. Period. I’m not even positive they should even be allowed to see again, since it obviously hasn’t done them much good so far.

  43. Code that believes there’s some way to pretend textfile encodings don’t exist is broken and dangerous. Might as well poke the other eye out, too.

  44. Code that converts unknown characters to ? is broken, stupid, braindead, and runs contrary to the standard recommendation, which says NOT TO DO THAT! RTFM for why not.

  45. Code that believes it can reliably guess the encoding of an unmarked textfile is guilty of a fatal mélange of hubris and naïveté that only a lightning bolt from Zeus will fix.

  46. Code that believes you can use printf widths to pad and justify Unicode data is broken and wrong.

  47. Code that believes once you successfully create a file by a given name, that when you run ls or readdir on its enclosing directory, you’ll actually find that file with the name you created it under is buggy, broken, and wrong. Stop being surprised by this!

  48. Code that believes UTF-16 is a fixed-width encoding is stupid, broken, and wrong. Revoke their programming licence.

  49. Code that treats code points from one plane one whit differently than those from any other plane is ipso facto broken and wrong. Go back to school.

  50. Code that believes that stuff like /s/i can only match "S" or "s" is broken and wrong. You’d be surprised.

  51. Code that uses \PM\pM* to find grapheme clusters instead of using \X is broken and wrong.

  52. People who want to go back to the ASCII world should be whole-heartedly encouraged to do so, and in honor of their glorious upgrade they should be provided gratis with a pre-electric manual typewriter for all their data-entry needs. Messages sent to them should be sent via an ??????s telegraph at 40 characters per line and hand-delivered by a courier. STOP.



I don’t know how much more “default Unicode in ” you can get than what I’ve written. Well, yes I do: you should be using Unicode::Collate and Unicode::LineBreak, too. And probably more.

As you see, there are far too many Unicode things that you really do have to worry about for there to ever exist any such thing as “default to Unicode”.

What you’re going to discover, just as we did back in 5.8, that it is simply impossible to impose all these things on code that hasn’t been designed right from the beginning to account for them. Your well-meaning selfishness just broke the entire world.

And even once you do, there are still critical issues that require a great deal of thought to get right. There is no switch you can flip. Nothing but brain, and I mean real brain, will suffice here. There’s a heck of a lot of stuff you have to learn. Modulo the retreat to the manual typewriter, you simply cannot hope to sneak by in ignorance. This is the 21?? century, and you cannot wish Unicode away by willful ignorance.

You have to learn it. Period. It will never be so easy that “everything just works,” because that will guarantee that a lot of things don’t work — which invalidates the assumption that there can ever be a way to “make it all work.”

You may be able to get a few reasonable defaults for a very few and very limited operations, but not without thinking about things a whole lot more than I think you have.

As just one example, canonical ordering is going to cause some real headaches. "\x{F5}" ‘õ’, "o\x{303}" ‘õ’, "o\x{303}\x{304}" ‘?’, and "o\x{304}\x{303}" ‘o~’ should all match ‘õ’, but how in the world are you going to do that? This is harder than it looks, but it’s something you need to account for.

If there’s one thing I know about Perl, it is what its Unicode bits do and do not do, and this thing I promise you: “ _?_?_?_?_?_ _?_s_ _?_?_ _U_?_?_?_?_?_?_ _?_?_?_?_?_ _?_?_?_?_?_?_ _ ”

You cannot just change some defaults and get smooth sailing. It’s true that I run with PERL_UNICODE set to "SA", but that’s all, and even that is mostly for command-line stuff. For real work, I go through all the many steps outlined above, and I do it very, ** very** carefully.


¡?dl?? ???? ?do? pu? ???p ???u ? ???? ???nl poo?

Returning JSON response from Servlet to Javascript/JSP page

Got it working! I should have been building a JSONArray of JSONObjects and then add the array to a final "Addresses" JSONObject. Observe the following:

JSONObject json      = new JSONObject();
JSONArray  addresses = new JSONArray();
JSONObject address;
try
{
   int count = 15;

   for (int i=0 ; i<count ; i++)
   {
       address = new JSONObject();
       address.put("CustomerName"     , "Decepticons" + i);
       address.put("AccountId"        , "1999" + i);
       address.put("SiteId"           , "1888" + i);
       address.put("Number"            , "7" + i);
       address.put("Building"          , "StarScream Skyscraper" + i);
       address.put("Street"            , "Devestator Avenue" + i);
       address.put("City"              , "Megatron City" + i);
       address.put("ZipCode"          , "ZZ00 XX1" + i);
       address.put("Country"           , "CyberTron" + i);
       addresses.add(address);
   }
   json.put("Addresses", addresses);
}
catch (JSONException jse)
{ 

}
response.setContentType("application/json");
response.getWriter().write(json.toString());

This worked and returned valid and parse-able JSON. Hopefully this helps someone else in the future. Thanks for your help Marcel

How to align a div to the top of its parent but keeping its inline-block behaviour?

Use vertical-align:top; for the element you want at the top, as I have demonstrated on your jsfiddle.

http://www.brunildo.org/test/inline-block.html

JMS Topic vs Queues

That means a topic is appropriate. A queue means a message goes to one and only one possible subscriber. A topic goes to each and every subscriber.

How to copy to clipboard using Access/VBA?

I couldn't figure out how to use the API using the first Google results. Fortunately a thread somewhere pointed me to this link: http://access.mvps.org/access/api/api0049.htm

Which works nicely. :)

Why is this program erroneously rejected by three C++ compilers?

Your compilers are expecting ASCII, but that program is obviously written using EBCDIC.

Unable to find the wrapper "https" - did you forget to enable it when you configured PHP?

I have enable the openssl extention and it work for me :)

;extension=php_openssl.dll

to

extension=php_openssl.dll

Generate Row Serial Numbers in SQL Query

Sometime we might don't want to apply ordering on our result set to add serial number. But if we are going to use ROW_NUMBER() then we have to have a ORDER BY clause. So, for that we can simply apply a tricks to avoid any ordering on the result set.

SELECT ROW_NUMBER() OVER(ORDER BY (SELECT 1)) AS ItemNo, ItemName FROM ItemMastetr

For that we don't need to apply order by on our result set. We'll just add ItemNo on our given result set.

How to use CURL via a proxy?

Here is a well tested function which i used for my projects with detailed self explanatory comments


There are many times when the ports other than 80 are blocked by server firewall so the code appears to be working fine on localhost but not on the server

function get_page($url){

global $proxy;
$ch = curl_init();
curl_setopt($ch, CURLOPT_URL, $url);
//curl_setopt($ch, CURLOPT_PROXY, $proxy);
curl_setopt($ch, CURLOPT_HEADER, 0); // return headers 0 no 1 yes
curl_setopt($ch, CURLOPT_RETURNTRANSFER, true); // return page 1:yes
curl_setopt($ch, CURLOPT_TIMEOUT, 200); // http request timeout 20 seconds
curl_setopt($ch, CURLOPT_FOLLOWLOCATION, true); // Follow redirects, need this if the url changes
curl_setopt($ch, CURLOPT_MAXREDIRS, 2); //if http server gives redirection responce
curl_setopt($ch, CURLOPT_USERAGENT,
    "Mozilla/5.0 (Windows; U; Windows NT 5.1; en-US; rv:1.8.1.7) Gecko/20070914 Firefox/2.0.0.7");
curl_setopt($ch, CURLOPT_COOKIEJAR, "cookies.txt"); // cookies storage / here the changes have been made
curl_setopt($ch, CURLOPT_COOKIEFILE, "cookies.txt");
curl_setopt($ch, CURLOPT_SSL_VERIFYPEER, false); // false for https
curl_setopt($ch, CURLOPT_ENCODING, "gzip"); // the page encoding

$data = curl_exec($ch); // execute the http request
curl_close($ch); // close the connection
return $data;
}

rmagick gem install "Can't find Magick-config"

Ubuntu:

sudo apt-get install imagemagick libmagickwand-dev libmagickcore-dev
gem install rmagick

CentOS:

yum remove ImageMagick
gem uninstall rmagick
yum install ImageMagick ImageMagick-devel ImageMagick-last-libs ImageMagick-c++ ImageMagick-c++-devel
gem install rmagick

MacOS:

download and install http://xquartz.macosforge.org/trac/wiki/X112.7.2

after:

brew uninstall imagemagick
brew link xz jpeg freetype    
brew install imagemagick
brew link --overwrite imagemagick
gem install rmagick

Fastest method of screen capturing on Windows

In my Impression, the GDI approach and the DX approach are different in its nature. painting using GDI applies the FLUSH method, the FLUSH approach draws the frame then clear it and redraw another frame in the same buffer, this will result in flickering in games require high frame rate.

  1. WHY DX quicker? in DX (or graphics world), a more mature method called double buffer rendering is applied, where two buffers are present, when present the front buffer to the hardware, you can render to the other buffer as well, then after the frame 1 is finished rendering, the system swap to the other buffer( locking it for presenting to hardware , and release the previous buffer ), in this way the rendering inefficiency is greatly improved.
  2. WHY turning down hardware acceleration quicker? although with double buffer rendering, the FPS is improved, but the time for rendering is still limited. modern graphic hardware usually involves a lot of optimization during rendering typically like anti-aliasing, this is very computation intensive, if you don't require that high quality graphics, of course you can just disable this option. and this will save you some time.

I think what you really need is a replay system, which I totally agree with what people discussed.

Converting a string to a date in DB2

In format function your can use timestamp_format function. Example, if the format is YYYYMMDD you can do it :

select TIMESTAMP_FORMAT(yourcolumnchar, 'YYYYMMDD') as YouTimeStamp 
from yourtable

you can then adapt then format with elements format foundable here

When to use DataContract and DataMember attributes?

In terms of WCF, we can communicate with the server and client through messages. For transferring messages, and from a security prospective, we need to make a data/message in a serialized format.

For serializing data we use [datacontract] and [datamember] attributes. In your case if you are using datacontract WCF uses DataContractSerializer else WCF uses XmlSerializer which is the default serialization technique.

Let me explain in detail:

basically WCF supports 3 types of serialization:

  1. XmlSerializer
  2. DataContractSerializer
  3. NetDataContractSerializer

XmlSerializer :- Default order is Same as class

DataContractSerializer/NetDataContractSerializer :- Default order is Alphabetical

XmlSerializer :- XML Schema is Extensive

DataContractSerializer/NetDataContractSerializer :- XML Schema is Constrained

XmlSerializer :- Versioning support not possible

DataContractSerializer/NetDataContractSerializer :- Versioning support is possible

XmlSerializer :- Compatibility with ASMX

DataContractSerializer/NetDataContractSerializer :- Compatibility with .NET Remoting

XmlSerializer :- Attribute not required in XmlSerializer

DataContractSerializer/NetDataContractSerializer :- Attribute required in this serializing

so what you use depends on your requirements...

Replace contents of factor column in R dataframe

I had the same problem. This worked better:

Identify which level you want to modify: levels(iris$Species)

    "setosa" "versicolor" "virginica" 

So, setosa is the first.

Then, write this:

     levels(iris$Species)[1] <-"new name"

How does DateTime.Now.Ticks exactly work?

You can get the milliseconds since 1/1/1970 using such code:

private static DateTime JanFirst1970 = new DateTime(1970, 1, 1);
public static long getTime()
{
    return (long)((DateTime.Now.ToUniversalTime() - JanFirst1970).TotalMilliseconds + 0.5);
}

Download files from server php

If the folder is accessible from the browser (not outside the document root of your web server), then you just need to output links to the locations of those files. If they are outside the document root, you will need to have links, buttons, whatever, that point to a PHP script that handles getting the files from their location and streaming to the response.

Pass a JavaScript function as parameter

You can also use eval() to do the same thing.

//A function to call
function needToBeCalled(p1, p2)
{
    alert(p1+"="+p2);
}

//A function where needToBeCalled passed as an argument with necessary params
//Here params is comma separated string
function callAnotherFunction(aFunction, params)
{
    eval(aFunction + "("+params+")");
}

//A function Call
callAnotherFunction("needToBeCalled", "10,20");

That's it. I was also looking for this solution and tried solutions provided in other answers but finally got it work from above example.

jQuery to serialize only elements within a div

You can improve the speed of your code if you restrict the items jQuery will look at.

Use the selector :input instead of * to achieve it.

$('#divId :input').serialize()

This will make your code faster because the list of items is shorter.

convert strtotime to date time format in php

<?php
  echo date('d - m - Y',strtotime('2013-01-19 01:23:42'));
 ?>       
Out put : 19 - 01 - 2013

Change span text?

document.getElementById("serverTime").innerHTML = ...;

Extract the maximum value within each group in a dataframe

There are many possibilities to do this in R. Here are some of them:

df <- read.table(header = TRUE, text = 'Gene   Value
A      12
A      10
B      3
B      5
B      6
C      1
D      3
D      4')

# aggregate
aggregate(df$Value, by = list(df$Gene), max)
aggregate(Value ~ Gene, data = df, max)

# tapply
tapply(df$Value, df$Gene, max)

# split + lapply
lapply(split(df, df$Gene), function(y) max(y$Value))

# plyr
require(plyr)
ddply(df, .(Gene), summarise, Value = max(Value))

# dplyr
require(dplyr)
df %>% group_by(Gene) %>% summarise(Value = max(Value))

# data.table
require(data.table)
dt <- data.table(df)
dt[ , max(Value), by = Gene]

# doBy
require(doBy)
summaryBy(Value~Gene, data = df, FUN = max)

# sqldf
require(sqldf)
sqldf("select Gene, max(Value) as Value from df group by Gene", drv = 'SQLite')

# ave
df[as.logical(ave(df$Value, df$Gene, FUN = function(x) x == max(x))),]

Getting value of select (dropdown) before change

Combine the focus event with the change event to achieve what you want:

(function () {
    var previous;

    $("select").on('focus', function () {
        // Store the current value on focus and on change
        previous = this.value;
    }).change(function() {
        // Do something with the previous value after the change
        alert(previous);

        // Make sure the previous value is updated
        previous = this.value;
    });
})();

Working example: http://jsfiddle.net/x5PKf/766

/usr/lib/libstdc++.so.6: version `GLIBCXX_3.4.15' not found

I have been avoiding this issue in the past by simply linking libstdc++ statically with this parameter sent to g++ when linking my executable:

-static-libstdc++

If linking in the library statically is an option this is probably the quickest work-around.

Underline text in UIlabel

NSUnderlineStyleAttributeName which takes an NSNumber (where 0 is no underline) can be added to an attribute dictionary. I don't know if this is any easier. But, it was easier for my purposes.

    NSDictionary *attributes; 
    attributes = @{NSFontAttributeName:font,   NSParagraphStyleAttributeName: style, NSUnderlineStyleAttributeName:[NSNumber numberWithInteger:1]};

    [text drawInRect:CGRectMake(self.contentRect.origin.x, currentY, maximumSize.width, textRect.size.height) withAttributes:attributes];

What is makeinfo, and how do I get it?

For Centos , I solve it by installing these packages.

yum install texi2html texinfo 

Dont worry if there is no entry for makeinfo. Just run

make all

You can do it similarly for ubuntu using sudo.

Prevent scroll-bar from adding-up to the Width of page on Chrome

It doesn't seem my other answer is working quite right at the moment (but I'll continue to try to get it operational).

But basically what you'll need to do, and what it was trying to do dynamically, is set the contents' width to slightly less than that of the parent, scrollable pane.
So that when the scrollbar appears it has no affect on the content.

This EXAMPLE shows a more hacky way of attaining that goal, by hardcoding widths instead of trying to get the browser to do it for us via padding.
If this is feasible this is the most simplest solution if you don't want a permanent scrollbar.

Function names in C++: Capitalize or not?

Unlike Java, C++ doesn't have a "standard style". Pretty much very company I've ever worked at has its own C++ coding style, and most open source projects have their own styles too. A few coding conventions you might want to look at:

It's interesting to note that C++ coding standards often specify which parts of the language not to use. For example, the Google C++ Style Guide says "We do not use C++ exceptions". Almost everywhere I've worked has prohibited certain parts of C++. (One place I worked basically said, "program in C, but new and delete are okay"!)

Why do I get a warning icon when I add a reference to an MEF plugin project?

To fix some not working stuff it has sense to remove some libraries sometimes, how would not that sound weird.

Anyways, I believe the problem is too wide and might be caused by different factors, so want to share my situation/solution.

I had a project (brought by customer) with Xamarin Forms and Telerik libraries. The thing was in general related to the components, which libraries are not included into packages folder, nor available via Nuget (paid ones).

The whole project References were "yellow", it looked horribly and scary.

The solution was just to remove those Telerik references (including a few controls in code which were using that). Right after that all the references magically got their common normal grey color and the errors (mostly) disappeared.

"Mostly" - because "all red around" error messages about "the element is not defined anywhere" sometimes happen still. That's weird, and brings inconvenience, but I still able to compile and run the project(s): just need to clean solution, restart Visual Studio, pray a little bit, clean again, remove obj/bin folders, restart again, and it works well.

The key thing is remove not available libraries references, as the error messages say absolutely another stuff. (For instance, something like "Xamarin.Build.Download.XamarinDownloadArchives not found or cannot find something" etc., but that just might mean you don't have some references available.

Then remove packages folder, reload/reopen the project/solution, go to "Manage Nuget Packages" and click "Restore" button.

undefined offset PHP error

How to reproduce this error in PHP:

Create an empty array and ask for the value given a key like this:

php> $foobar = array();

php> echo gettype($foobar);
array

php> echo $foobar[0];

PHP Notice:  Undefined offset: 0 in 
/usr/local/lib/python2.7/dist-packages/phpsh/phpsh.php(578) : 
eval()'d code on line 1

What happened?

You asked an array to give you the value given a key that it does not contain. It will give you the value NULL then put the above error in the errorlog.

It looked for your key in the array, and found undefined.

How to make the error not happen?

Ask if the key exists first before you go asking for its value.

php> echo array_key_exists(0, $foobar) == false;
1

If the key exists, then get the value, if it doesn't exist, no need to query for its value.

Why do you need to put #!/bin/bash at the beginning of a script file?

It's a convention so the *nix shell knows what kind of interpreter to run.

For example, older flavors of ATT defaulted to sh (the Bourne shell), while older versions of BSD defaulted to csh (the C shell).

Even today (where most systems run bash, the "Bourne Again Shell"), scripts can be in bash, python, perl, ruby, PHP, etc, etc. For example, you might see #!/bin/perl or #!/bin/perl5.

PS: The exclamation mark (!) is affectionately called "bang". The shell comment symbol (#) is sometimes called "hash".

PPS: Remember - under *nix, associating a suffix with a file type is merely a convention, not a "rule". An executable can be a binary program, any one of a million script types and other things as well. Hence the need for #!/bin/bash.

Check for database connection, otherwise display message

very basic:

<?php 
$username = 'user';
$password = 'password';
$server = 'localhost'; 
// Opens a connection to a MySQL server
$connection = mysql_connect ($server, $username, $password) or die('try again in some minutes, please');
//if you want to suppress the error message, substitute the connection line for:
//$connection = @mysql_connect($server, $username, $password) or die('try again in some minutes, please');
?>

result:

Warning: mysql_connect() [function.mysql-connect]: Access denied for user 'user'@'localhost' (using password: YES) in /home/user/public_html/zdel1.php on line 6 try again in some minutes, please

as per Wrikken's recommendation below, check out a complete error handler for more complex, efficient and elegant solutions: http://www.php.net/manual/en/function.set-error-handler.php

How to change the port number for Asp.Net core app?

If you want to run on a specific port 60535 while developing locally but want to run app on port 80 in stage/prod environment servers, this does it.

Add to environmentVariables section in launchSettings.json

"ASPNETCORE_DEVELOPER_OVERRIDES": "Developer-Overrides",

and then modify Program.cs to

public static IHostBuilder CreateHostBuilder(string[] args) =>
            Host.CreateDefaultBuilder(args)
                .ConfigureWebHostDefaults(webBuilder =>
                {
                    webBuilder.UseKestrel(options =>
                    {
                        var devOverride = Environment.GetEnvironmentVariable("ASPNETCORE_DEVELOPER_OVERRIDES");
                        if (!string.IsNullOrWhiteSpace(devOverride))
                        {
                            options.ListenLocalhost(60535);
                        }
                        else
                        {
                            options.ListenAnyIP(80);
                        }
                    })
                    .UseStartup<Startup>()
                    .UseNLog();                   
                });

How to print to the console in Android Studio?

Android Studio 3.0 and earlier:

If the other solutions don't work, you can always see the output in the Android Monitor.


android studio screen shot


Make sure to set your filter to Show only selected application or create a custom filter.

enter image description here

Absolute positioning ignoring padding of parent

1.) you cannot use big border on parent -- in case you want to have a specific border

2.) you cannot add margin -- in case your parent is a part of some other container and you want your parent to take the full width of that grand parent.

The Only Solution that should be applicable is to wrap your children's in an another container so that your parent becomes the grandparent and then apply padding to the new children's Wrapper / Parent. Or you can directly apply padding to the children.

In your case:

.css-sux{
  padding: 0px 10px 10px 10px;
}

starting file download with JavaScript

If this is your own server application then i suggest using the following header

Content-disposition: attachment; filename=fname.ext

This will force any browser to download the file and not render it in the browser window.

How to create a blank/empty column with SELECT query in oracle?

I think you should use null

SELECT CustomerName AS Customer, null AS Contact 
FROM Customers;

And Remember that Oracle

treats a character value with a length of zero as null.

Creating an empty bitmap and drawing though canvas in Android

Do not use Bitmap.Config.ARGB_8888

Instead use int w = WIDTH_PX, h = HEIGHT_PX;

Bitmap.Config conf = Bitmap.Config.ARGB_4444; // see other conf types
Bitmap bmp = Bitmap.createBitmap(w, h, conf); // this creates a MUTABLE bitmap
Canvas canvas = new Canvas(bmp);

// ready to draw on that bitmap through that canvas

ARGB_8888 can land you in OutOfMemory issues when dealing with more bitmaps or large bitmaps. Or better yet, try avoiding usage of ARGB option itself.

DateTime group by date and hour

SELECT [activity_dt], COUNT(*) as [Count]
  FROM 
 (SELECT dateadd(hh, datediff(hh, '20010101', [activity_dt]), '20010101') as [activity_dt]
    FROM table) abc
 GROUP BY [activity_dt]

Detect if HTML5 Video element is playing

var video_switch  = 0;

function play() {

    var media = document.getElementById('video');

    if (video_switch == 0)
    {
        media.play();
        video_switch = 1;
    }
    else if (video_switch == 1)
    {
        media.pause();
        video_switch = 0;
    }
}

jQuery.click() vs onClick

Go for this as it will give you both standard and performance.

 $('#myDiv').click(function(){
      //Some code
 });

As the second method is simple JavaScript code and is faster than jQuery. But here performance will be approximately the same.

How do I check whether an array contains a string in TypeScript?

If your code is ES7 based (or upper versions):

channelArray.includes('three'); //will return true or false

If not, for example you are using IE with no babel transpile:

channelArray.indexOf('three') !== -1; //will return true or false

the indexOf method will return the position the element has into the array, because of that we use !== different from -1 if the needle is found at the first position.

data.frame Group By column

require(reshape2)

T <- melt(df, id = c("A"))

T <- dcast(T, A ~ variable, sum)

I am not certain the exact advantages over aggregate.

java.text.ParseException: Unparseable date

Your pattern does not correspond to the input string at all... It is not surprising that it does not work. This would probably work better:

SimpleDateFormat sdf = new SimpleDateFormat("EE MMM dd HH:mm:ss z yyyy",
                                            Locale.ENGLISH);

Then to print with your required format you need a second SimpleDateFormat:

Date parsedDate = sdf.parse(date);
SimpleDateFormat print = new SimpleDateFormat("MMM d, yyyy HH:mm:ss");
System.out.println(print.format(parsedDate));

Notes:

  • you should include the locale as if your locale is not English, the day name might not be recognised
  • IST is ambiguous and can lead to problems so you should use the proper time zone name if possible in your input.

JS - window.history - Delete a state

There is no way to delete or read the past history.

You could try going around it by emulating history in your own memory and calling history.pushState everytime window popstate event is emitted (which is proposed by the currently accepted Mike's answer), but it has a lot of disadvantages that will result in even worse UX than not supporting the browser history at all in your dynamic web app, because:

  • popstate event can happen when user goes back ~2-3 states to the past
  • popstate event can happen when user goes forward

So even if you try going around it by building virtual history, it's very likely that it can also lead into a situation where you have blank history states (to which going back/forward does nothing), or where that going back/forward skips some of your history states totally.

Laravel - Form Input - Multiple select for a one to many relationship

This might be a better approach than top answer if you need to compare 2 output arrays to each other but use the first array to populate the options.

This is also helpful when you have a non-numeric or offset index (key) in your array.

<select name="roles[]" multiple>
    @foreach($roles as $key => $value)
        <option value="{{$key}}" @if(in_array($value, $compare_roles))selected="selected"@endif>
            {{$value}}
        </option>
    @endforeach
</select>

Reading a file line by line in Go

// strip '\n' or read until EOF, return error if read error  
func readline(reader io.Reader) (line []byte, err error) {   
    line = make([]byte, 0, 100)                              
    for {                                                    
        b := make([]byte, 1)                                 
        n, er := reader.Read(b)                              
        if n > 0 {                                           
            c := b[0]                                        
            if c == '\n' { // end of line                    
                break                                        
            }                                                
            line = append(line, c)                           
        }                                                    
        if er != nil {                                       
            err = er                                         
            return                                           
        }                                                    
    }                                                        
    return                                                   
}                                    

Can I clear cell contents without changing styling?

you can use ClearContents. ex,

Range("X").Cells.ClearContents

How to find the highest value of a column in a data frame in R?

Try this solution:

Oz<-subset(data, data$Month==5,select=Ozone) # select ozone  value in the month of                 
                                             #May (i.e. Month = 5)
summary(T)                                   #gives caracteristics of table( contains 1 column of Ozone) including max, min ...

How to place a file on classpath in Eclipse?

Copy the file into your src folder. Go to the Project Explorer in Eclipse, Right-click on your project, and click on "Refresh". The file should appear on the Project Explorer pane as well.

Reading entire html file to String?

Here's a solution to retrieve the html of a webpage using only standard java libraries:

import java.io.*;
import java.net.*;

String urlToRead = "https://google.com";
URL url; // The URL to read
HttpURLConnection conn; // The actual connection to the web page
BufferedReader rd; // Used to read results from the web page
String line; // An individual line of the web page HTML
String result = ""; // A long string containing all the HTML
try {
 url = new URL(urlToRead);
 conn = (HttpURLConnection) url.openConnection();
 conn.setRequestMethod("GET");
 rd = new BufferedReader(new InputStreamReader(conn.getInputStream()));
 while ((line = rd.readLine()) != null) {
  result += line;
 }
 rd.close();
} catch (Exception e) {
 e.printStackTrace();
}

System.out.println(result);

SRC

SQL Server - Adding a string to a text column (concat equivalent)

UPDATE test SET a = CONCAT(a, "more text")

Jquery find nearest matching element

var otherInput = $(this).closest('.row').find('.inputQty');

That goes up to a row level, then back down to .inputQty.

How to make asynchronous HTTP requests in PHP

As of 2018, Guzzle has become the defacto standard library for HTTP requests, used in several modern frameworks. It's written in pure PHP and does not require installing any custom extensions.

It can do asynchronous HTTP calls very nicely, and even pool them such as when you need to make 100 HTTP calls, but don't want to run more than 5 at a time.

Concurrent request example

use GuzzleHttp\Client;
use GuzzleHttp\Promise;

$client = new Client(['base_uri' => 'http://httpbin.org/']);

// Initiate each request but do not block
$promises = [
    'image' => $client->getAsync('/image'),
    'png'   => $client->getAsync('/image/png'),
    'jpeg'  => $client->getAsync('/image/jpeg'),
    'webp'  => $client->getAsync('/image/webp')
];

// Wait on all of the requests to complete. Throws a ConnectException
// if any of the requests fail
$results = Promise\unwrap($promises);

// Wait for the requests to complete, even if some of them fail
$results = Promise\settle($promises)->wait();

// You can access each result using the key provided to the unwrap
// function.
echo $results['image']['value']->getHeader('Content-Length')[0]
echo $results['png']['value']->getHeader('Content-Length')[0]

See http://docs.guzzlephp.org/en/stable/quickstart.html#concurrent-requests

How to do scanf for single char in C

Provides a space before %c conversion specifier so that compiler will ignore white spaces. The program may be written as below:

#include <stdio.h>
#include <stdlib.h>
int main()
{
    char ch;
    printf("Enter one char");
    scanf(" %c", &ch); /*Space is given before %c*/
    printf("%c\n",ch);
return 0;
}

Catching exceptions from Guzzle

Depending on your project, disabling exceptions for guzzle might be necessary. Sometimes coding rules disallow exceptions for flow control. You can disable exceptions for Guzzle 3 like this:

$client = new \Guzzle\Http\Client($httpBase, array(
  'request.options' => array(
     'exceptions' => false,
   )
));

This does not disable curl exceptions for something like timeouts, but now you can get every status code easily:

$request = $client->get($uri);
$response = $request->send();
$statuscode = $response->getStatusCode();

To check, if you got a valid code, you can use something like this:

if ($statuscode > 300) {
  // Do some error handling
}

... or better handle all expected codes:

if (200 === $statuscode) {
  // Do something
}
elseif (304 === $statuscode) {
  // Nothing to do
}
elseif (404 === $statuscode) {
  // Clean up DB or something like this
}
else {
  throw new MyException("Invalid response from api...");
}

For Guzzle 5.3

$client = new \GuzzleHttp\Client(['defaults' => [ 'exceptions' => false ]] );

Thanks to @mika

For Guzzle 6

$client = new \GuzzleHttp\Client(['http_errors' => false]);

Private class declaration

Private outer class would be useless as nothing can access it.

See more details:

Java: Why can we define a top level class as private?

Error handling in AngularJS http get then construct

You need to add an additional parameter:

$http.get(url).then(
    function(response) {
        console.log('get',response)
    },
    function(data) {
        // Handle error here
    })

Angular 4 default radio button checked by default

getting following error

_x000D_
_x000D_
It happens:  Error: 
      ngModel cannot be used to register form controls with a parent formGroup directive.  Try using
      formGroup's partner directive "formControlName" instead.  Example:
_x000D_
_x000D_
_x000D_

numpy: most efficient frequency counts for unique values in an array

Even though it has already been answered, I suggest a different approach that makes use of numpy.histogram. Such function given a sequence it returns the frequency of its elements grouped in bins.

Beware though: it works in this example because numbers are integers. If they where real numbers, then this solution would not apply as nicely.

>>> from numpy import histogram
>>> y = histogram (x, bins=x.max()-1)
>>> y
(array([5, 3, 0, 0, 1, 0, 0, 0, 0, 0, 0, 0, 0, 0, 0, 0, 0, 0, 0, 0, 0, 0, 0,
       1]),
 array([  1.,   2.,   3.,   4.,   5.,   6.,   7.,   8.,   9.,  10.,  11.,
        12.,  13.,  14.,  15.,  16.,  17.,  18.,  19.,  20.,  21.,  22.,
        23.,  24.,  25.]))

Why won't eclipse switch the compiler to Java 8?

<build>
        <plugins>
            <plugin>
                <groupId>org.apache.maven.plugins</groupId>
                <artifactId>maven-compiler-plugin</artifactId>
                <version>3.6.1</version>
                <configuration>
                    <source>1.8</source>
                    <target>1.8</target>
                </configuration>
            </plugin>
        </plugins>
    </build>

Compare two objects' properties to find differences?

Yes, with reflection - assuming each property type implements Equals appropriately. An alternative would be to use ReflectiveEquals recursively for all but some known types, but that gets tricky.

public bool ReflectiveEquals(object first, object second)
{
    if (first == null && second == null)
    {
        return true;
    }
    if (first == null || second == null)
    {
        return false;
    }
    Type firstType = first.GetType();
    if (second.GetType() != firstType)
    {
        return false; // Or throw an exception
    }
    // This will only use public properties. Is that enough?
    foreach (PropertyInfo propertyInfo in firstType.GetProperties())
    {
        if (propertyInfo.CanRead)
        {
            object firstValue = propertyInfo.GetValue(first, null);
            object secondValue = propertyInfo.GetValue(second, null);
            if (!object.Equals(firstValue, secondValue))
            {
                return false;
            }
        }
    }
    return true;
}

How to connect to mysql with laravel?

In Laravel 5, there is a .env file,

It looks like

APP_ENV=local
APP_DEBUG=true
APP_KEY=YOUR_API_KEY

DB_HOST=YOUR_HOST
DB_DATABASE=YOUR_DATABASE
DB_USERNAME=YOUR_USERNAME
DB_PASSWORD=YOUR_PASSWORD

CACHE_DRIVER=file
SESSION_DRIVER=file
QUEUE_DRIVER=sync

MAIL_DRIVER=smtp
MAIL_HOST=mailtrap.io
MAIL_PORT=2525
MAIL_USERNAME=null
MAIL_PASSWORD=null

Edit that .env There is .env.sample is there , try to create from that if no such .env file found.

Deleting queues in RabbitMQ

In RabbitMQ versions > 3.0, you can also utilize the HTTP API if the rabbitmq_management plugin is enabled. Just be sure to set the content-type to 'application/json' and provide the vhost and queue name:

I.E. Using curl with a vhost 'test' and queue name 'testqueue':

$ curl -i -u guest:guest -H "content-type:application/json" -XDELETE http://localhost:15672/api/queues/test/testqueue
HTTP/1.1 204 No Content
Server: MochiWeb/1.1 WebMachine/1.9.0 (someone had painted it blue)
Date: Tue, 16 Apr 2013 10:37:48 GMT
Content-Type: application/json
Content-Length: 0

How to get a json string from url?

AFAIK JSON.Net does not provide functionality for reading from a URL. So you need to do this in two steps:

using (var webClient = new System.Net.WebClient()) {
    var json = webClient.DownloadString(URL);
    // Now parse with JSON.Net
}

Java Swing - how to show a panel on top of another panel?

Use a 1 by 1 GridLayout on the existing JPanel, then add your Panel to that JPanel. The only problem with a GridLayout that's 1 by 1 is that you won't be able to place other items on the JPanel. In this case, you will have to figure out a layout that is suitable. Each panel that you use can use their own layout so that wouldn't be a problem.

Am I understanding this question correctly?

Count number of records returned by group by

You can do both in one query using the OVER clause on another COUNT

select
    count(*) RecordsPerGroup,
    COUNT(*) OVER () AS TotalRecords
from temptable
group by column_1, column_2, column_3, column_4

How to fix the error "Windows SDK version 8.1" was not found?

Another way (worked for 2015) is open "Install/remove programs" (Apps & features), find Visual Studio, select Modify. In opened window, press Modify, check

  • Languages -> Visual C++ -> Common tools for Visual C++
  • Windows and web development -> Tools for universal windows apps -> Tools (1.4.1) and Windows 10 SDK ([version])
  • Windows and web development -> Tools for universal windows apps -> Windows 10 SDK ([version])

and install. Then right click on solution -> Re-target and it will compile

OAuth2 and Google API: access token expiration time?

The default expiry_date for google oauth2 access token is 1 hour. The expiry_date is in the Unix epoch time in milliseconds. If you want to read this in human readable format then you can simply check it here..Unix timestamp to human readable time

Active Menu Highlight CSS

As the given tag to this question is CSS, I will provide the way I accomplished it.

  1. Create a class in the .css file.

    a.activePage{ color: green; border-bottom: solid; border-width: 3px;}

  2. Nav-bar will be like:

    • Home
    • Contact Us
    • About Us

NOTE: If you are already setting the style for all Nav-Bar elements using a class, you can cascade the special-case class we created with a white-space after the generic class in the html-tag.

Example: Here, I was already importing my style from a class called 'navList' I created for all list-items . But the special-case styling-attributes are part of class 'activePage'

.CSS file:

a.navList{text-decoration: none; color: gray;}
a.activePage{ color: green; border-bottom: solid; border-width: 3px;}

.HTML file:

<div id="sub-header">
    <ul>
        <li> <a href="index.php" class= "navList activePage" >Home</a> </li>
        <li> <a href="contact.php" class= "navList">Contact Us</a> </li>
        <li> <a href="about.php" class= "navList">About Us</a> </li>
    </ul>
</div>

Look how I've cascaded one class-name behind other.

How do I disable right click on my web page?

Put this code into your <head> tag of your page.

<script type="text/javascript"> 
function disableselect(e){  
return false  
}  

function reEnable(){  
return true  
}  

//if IE4+  
document.onselectstart=new Function ("return false")  
document.oncontextmenu=new Function ("return false")  
//if NS6  
if (window.sidebar){  
document.onmousedown=disableselect  
document.onclick=reEnable  
}
</script>

This will disable right click on your whole web page, but only when JavaScript is enabled.

how to check if the input is a number or not in C?

if (sscanf(command_level[2], "%f%c", &check_f, &check_c)!=1)
{
        is_num=false;
}
else
{
        is_num=true;
}   

if(sscanf(command_level[2],"%f",&check_f) != 1) 
{
    is_num=false;
}

how about this?

Make child div stretch across width of page

I know this post is old, in case someone stumbles upon it in 2019, this would work try it.

//html
<div id="container">
<div id="help_panel">
<div class="help_panel_extra_if_you_want"> //then if you want to add some height and width if you want, do this.

</div>
</div>
</div>

//css
#container{
left: 0px;
top: 0px;
right: 0px;
z-index: 100;
display: -webkit-box;
display: -webkit-flex;
display: -ms-flexbox;
display: flex;
-webkit-box-orient: vertical;
-webkit-box-direction: normal;
-webkit-flex-direction: column;
-ms-flex-direction: column;
flex-direction: column;
-webkit-box-pack: start;
-webkit-justify-content: flex-start;
-ms-flex-pack: start;
justify-content: flex-start;    
position: relative;
height:650px;
margin-top:55px;
margin-bottom:-20px;
}

#help_panel {
display: -webkit-box;
display: -webkit-flex;
display: -ms-flexbox;
display: flex;
padding-right: 24px;
padding-left: 18px;
-webkit-box-orient: vertical;
-webkit-box-direction: normal;
-webkit-flex-direction: column;
-ms-flex-direction: column;
flex-direction: column;
-webkit-box-align: center;
-webkit-align-items: center;
-ms-flex-align: center;
align-items: center;

.help_panel_extra_if_you_want{
height:650px;
position: relative;
-webkit-box-orient: horizontal;
-webkit-box-direction: normal;
-webkit-flex-direction: row;
-ms-flex-direction: row;
flex-direction: row;
-webkit-box-pack: justify;
-webkit-justify-content: space-between;
-ms-flex-pack: justify;
justify-content: space-between;
align-items: center;
display: flex;
width: 95%;
max-width: 1200px;
}

SHOULD GIVE YOU SOMETHING LIKE THIS This is how it would look, adjust the width and margin and padding to achieve what you want

Why does Math.Round(2.5) return 2 instead of 3?

Here's the way i had to work it around :

Public Function Round(number As Double, dec As Integer) As Double
    Dim decimalPowerOfTen = Math.Pow(10, dec)
    If CInt(number * decimalPowerOfTen) = Math.Round(number * decimalPowerOfTen, 2) Then
        Return Math.Round(number, 2, MidpointRounding.AwayFromZero)
    Else
        Return CInt(number * decimalPowerOfTen + 0.5) / 100
    End If
End Function

Trying with 1.905 with 2 decimals will give 1.91 as expected but Math.Round(1.905,2,MidpointRounding.AwayFromZero) gives 1.90! Math.Round method is absolutely inconsistent and unusable for most of the basics problems programmers may encounter. I have to check if (int) 1.905 * decimalPowerOfTen = Math.Round(number * decimalPowerOfTen, 2) cause i don not want to round up what should be round down.

How to drop SQL default constraint without knowing its name?

Following solution will drop specific default constraint of a column from the table

Declare @Const NVARCHAR(256)

SET @Const = (
              SELECT TOP 1 'ALTER TABLE' + YOUR TABLE NAME +' DROP CONSTRAINT '+name
              FROM Sys.default_constraints A
              JOIN sysconstraints B on A.parent_object_id = B.id
              WHERE id = OBJECT_ID('YOUR TABLE NAME')
              AND COL_NAME(id, colid)='COLUMN NAME'
              AND OBJECTPROPERTY(constid,'IsDefaultCnst')=1
            )
 EXEC (@Const)

get string value from HashMap depending on key name

If you will use Generics and define your map as

Map<String,String> map = new HashMap<String,String>();

then fetching value as

 String s = map.get("keyStr"); 

you wont be required to typecast the map.get() or call toString method to get String value

Find a string between 2 known values

Extracting contents between two known values can be useful for later as well. So why not create an extension method for it. Here is what i do, Short and simple...

  public static string GetBetween(this string content, string startString, string endString)
    {
        int Start=0, End=0;
        if (content.Contains(startString) && content.Contains(endString))
        {
            Start = content.IndexOf(startString, 0) + startString.Length;
            End = content.IndexOf(endString, Start);
            return content.Substring(Start, End - Start);
        }
        else
            return string.Empty;
    }

Using python PIL to turn a RGB image into a pure black and white image

A PIL only solution for creating a bi-level (black and white) image with a custom threshold:

from PIL import Image
img = Image.open('mB96s.png')
thresh = 200
fn = lambda x : 255 if x > thresh else 0
r = img.convert('L').point(fn, mode='1')
r.save('foo.png')

With just

r = img.convert('1')
r.save('foo.png')

you get a dithered image.

From left to right the input image, the black and white conversion result and the dithered result:

Input Image Black and White Result Dithered Result

You can click on the images to view the unscaled versions.

Margin on child element moves parent element

Although all of the answers fix the issue but they come with trade-offs/adjustments/compromises like

  • floats, You have to float elements
  • border-top, This pushes the parent at least 1px downwards which should then be adjusted with introducing -1px margin to the parent element itself. This can create problems when parent already has margin-top in relative units.
  • padding-top, same effect as using border-top
  • overflow: hidden, Can't be used when parent should display overflowing content, like a drop down menu
  • overflow: auto, Introduces scrollbars for parent element that has (intentionally) overflowing content (like shadows or tool tip's triangle)

The issue can be resolved by using CSS3 pseudo elements as follows

.parent::before {
  clear: both;
  content: "";
  display: table;
  margin-top: -1px;
  height: 0;
}

https://jsfiddle.net/hLgbyax5/1/

How to Remove Line Break in String

No one has ever suggested a RegExp solution. So here is one:

Function TrimTrailingLineBreak(pText)
    Dim oRE: Set oRE = New RegExp: oRE.Global = True
    oRE.Pattern = "(.*?)(\n|(\r\n)){1}$"
    TrimTrailingLineBreak = oRE.Replace(pText, "$1")
End Function

It captures and returns everything up until a single ({1}) trailing new line (\n), or carriage return & new line (\r\n), at the end of the text ($).
To remove all trailing line breaks change {1} to *.
And to remove all trailing whitespace (including line breaks) use oRE.Pattern = "(.*?)\s*$".

Cannot start session without errors in phpMyAdmin

For Xampp, Deleting temp flies from the 'root' folder works for me.

TH

COPY with docker but with exclusion

Excluding node_modules from current directory

node_modules

Excluding node_modules in any immediate subdirectories

*/node_modules

Here is the official docs

Getting the length of two-dimensional array

You can do :

 System.out.println(nir[0].length);

But be aware that there's no real two-dimensional array in Java. Each "first level" array contains another array. Each of these arrays can be of different sizes. nir[0].length isn't necessarily the same size as nir[1].length.

Installing Tomcat 7 as Service on Windows Server 2008

  1. Edit service.bat – Swap two lines so that they appear in following order: if not “%JAVA_HOME%“ == ““ goto got JdkHome if not “%JRE_HOME%“ == ““ goto got JreHome
  2. Open cmd and run command service.bat install
  3. Open Services and find Apache Tomcat 7.0 Tomcat7. Right click and Properties. Change its startup type to Automatic (with delay).
  4. Reboot machine to verify if the service started automatically

Clear MySQL query cache without restarting server

according the documentation, this should do it...

RESET QUERY CACHE 

PHP: How to get current time in hour:minute:second?

You can have both formats as an argument to the function date():

date("d-m-Y H:i:s")

Check the manual for more info : http://php.net/manual/en/function.date.php

As pointed out by @ThomasVdBerge to display minutes you need the 'i' character

java.rmi.ConnectException: Connection refused to host: 127.0.1.1;

Maybe your rmiregistry not be created before client trying connect to your server and it would lead to this exception.In Linux, you can use "netstat" to check your rmiregistry be bond on the right port you assigned in java code.

Change Primary Key

Assuming that your table name is city and your existing Primary Key is pk_city, you should be able to do the following:

ALTER TABLE city
DROP CONSTRAINT pk_city;

ALTER TABLE city
ADD CONSTRAINT pk_city PRIMARY KEY (city_id, buildtime, time);

Make sure that there are no records where time is NULL, otherwise you won't be able to re-create the constraint.

How to `wget` a list of URLs in a text file?

Run it in parallel with

cat text_file.txt | parallel --gnu "wget {}"

How to display a Yes/No dialog box on Android?

Thanks nikki your answer has helped me improve an existing simply by adding my desired action as follows

AlertDialog.Builder builder = new AlertDialog.Builder(this);

builder.setTitle("Do this action");
builder.setMessage("do you want confirm this action?");

builder.setPositiveButton("YES", new DialogInterface.OnClickListener() {

    public void onClick(DialogInterface dialog, int which) {
        // Do do my action here

        dialog.dismiss();
    }

});

builder.setNegativeButton("NO", new DialogInterface.OnClickListener() {

    @Override
    public void onClick(DialogInterface dialog, int which) {
        // I do not need any action here you might
        dialog.dismiss();
    }
});

AlertDialog alert = builder.create();
alert.show();

Border for an Image view in Android?

you must create a background.xml in res/drawable this code

<?xml version="1.0" encoding="UTF-8"?>
<shape xmlns:android="http://schemas.android.com/apk/res/android">
<solid android:color="#FFFFFF" />
<corners android:radius="6dp" />
<stroke
    android:width="6dp"
    android:color="@android:color/white" />
<padding
    android:bottom="6dp"
    android:left="6dp"
    android:right="6dp"
    android:top="6dp" />
</shape>

Programmatically get the version number of a DLL

var versionAttrib = new AssemblyName(Assembly.GetExecutingAssembly().FullName);

Javascript "Cannot read property 'length' of undefined" when checking a variable's length

There's a difference between an empty string "" and an undefined variable. You should be checking whether or not theHref contains a defined string, rather than its lenght:

if(theHref){
   // ---
}

If you still want to check for the length, then do this:

if(theHref && theHref.length){
   // ...
}

Rename package in Android Studio

If your package name is more than two dot separated, say com.hello.world and moreover, you did not put anything in com/ and com/hello/. All of your classes are putting into com/hello/world/, you might DO the following steps to refactoring your package name(s) in Android Studio or IntelliJ:

  • [FIRST] Add something under your directories(com/, com/hello/). You can achieve this by first add two files to package com.hello.world, say
   com.hello.world.PackageInfo1.java
   com.hello.world.PackageInfo2.java

then refactor them by moving them to com and com.hello respectively. You will see com and com.hello sitting there at the Project(Alt+1 or Command+1 for shortcut) and rename directories refactoring is waiting there as you expected.

  • Refactor to rename one or more of these directories to reach your aim. The only thing you should notice here is you must choose the directories rather than Packages when a dialog ask you.

  • If you've got lots of classes in your project, it will take you a while to wait for its auto-scan-and-rename.

  • Besides, you need to rename the package name inside the AndroidManifest.xml manually, I guess, such that other names in this file can benefit the prefix.

  • [ALSO], it might need you to replace all com.hello.world.R to the new XXX.XXX.XXX.R(Command+Shift+R for short)

  • Rebuild and run your project to see whether it work. And use "Find in Path" to find other non-touch names you'd like to rename.

  • Enjoy it.

Checking if a variable exists in javascript

It is important to note that 'undefined' is a perfectly valid value for a variable to hold. If you want to check if the variable exists at all,

if (window.variableName)

is a more complete check, since it is verifying that the variable has actually been defined. However, this is only useful if the variable is guaranteed to be an object! In addition, as others have pointed out, this could also return false if the value of variableName is false, 0, '', or null.

That said, that is usually not enough for our everyday purposes, since we often don't want to have an undefined value. As such, you should first check to see that the variable is defined, and then assert that it is not undefined using the typeof operator which, as Adam has pointed out, will not return undefined unless the variable truly is undefined.

if ( variableName  && typeof variableName !== 'undefined' )

Django - makemigrations - No changes detected

One more edge case and solution:

I added a boolean field, and at the same time added an @property referencing it, with the same name (doh). Commented the property and migration sees and adds the new field. Renamed the property and all is good.

Pure CSS checkbox image replacement

Using javascript seems to be unnecessary if you choose CSS3.

By using :before selector, you can do this in two lines of CSS. (no script involved).

Another advantage of this approach is that it does not rely on <label> tag and works even it is missing.

Note: in browsers without CSS3 support, checkboxes will look normal. (backward compatible).

input[type=checkbox]:before { content:""; display:inline-block; width:12px; height:12px; background:red; }
input[type=checkbox]:checked:before { background:green; }?

You can see a demo here: http://jsfiddle.net/hqZt6/1/

and this one with images:

http://jsfiddle.net/hqZt6/6/

How to check the function's return value if true or false

Wrong syntax. You can't compare a Boolean to a string like "false" or "true". In your case, just test it's inverse:

if(!ValidateForm()) { ...

You could test against the constant false, but it's rather ugly and generally frowned upon:

if(ValidateForm() == false) { ...

When to use AtomicReference in Java?

When do we use AtomicReference?

AtomicReference is flexible way to update the variable value atomically without use of synchronization.

AtomicReference support lock-free thread-safe programming on single variables.

There are multiple ways of achieving Thread safety with high level concurrent API. Atomic variables is one of the multiple options.

Lock objects support locking idioms that simplify many concurrent applications.

Executors define a high-level API for launching and managing threads. Executor implementations provided by java.util.concurrent provide thread pool management suitable for large-scale applications.

Concurrent collections make it easier to manage large collections of data, and can greatly reduce the need for synchronization.

Atomic variables have features that minimize synchronization and help avoid memory consistency errors.

Provide a simple example where AtomicReference should be used.

Sample code with AtomicReference:

String initialReference = "value 1";

AtomicReference<String> someRef =
    new AtomicReference<String>(initialReference);

String newReference = "value 2";
boolean exchanged = someRef.compareAndSet(initialReference, newReference);
System.out.println("exchanged: " + exchanged);

Is it needed to create objects in all multithreaded programs?

You don't have to use AtomicReference in all multi threaded programs.

If you want to guard a single variable, use AtomicReference. If you want to guard a code block, use other constructs like Lock /synchronized etc.

How do I escape only single quotes?

You can use the addcslashes function to get this done like so:

echo addcslashes($text, "'\\");

Extract and delete all .gz in a directory- Linux

for foo in *.gz
do
  tar xf "$foo"
  rm "$foo"
done

Using BufferedReader to read Text File

or

public String getFileStream(final String inputFile) {
        String result = "";
        Scanner s = null;

        try {
            s = new Scanner(new BufferedReader(new FileReader(inputFile)));
            while (s.hasNext()) {
                result = result + s.nextLine();
            }
        } catch (final IOException ex) {
            ex.printStackTrace();
        } finally {
            if (s != null) {
                s.close();
            }
        }
        return result;
}

This gets first line as well.

What is the .idea folder?

There is no problem in deleting this. It's not only the WebStorm IDE creating this file, but also PhpStorm and all other of JetBrains' IDEs.

It is safe to delete it but if your project is from GitLab or GitHub then you will see a warning.

How to restart Postgresql

Try this as root (maybe you can use sudo or su):

/etc/init.d/postgresql restart

Without any argument the script also gives you a hint on how to restart a specific version

[Uqbar@Feynman ~] /etc/init.d/postgresql
Usage: /etc/init.d/postgresql {start|stop|restart|reload|force-reload|status} [version ...]

Similarly, in case you have it, you can also use the service tool:

[Uqbar@Feynman ~] service postgresql
Usage: /etc/init.d/postgresql {start|stop|restart|reload|force reload|status} [version ...]

Please, pay attention to the optional [version ...] trailing argument. That's meant to allow you, the user, to act on a specific version, in case you were running multiple ones. So you can restart version X while keeping version Y and Z untouched and running.

Finally, in case you are running systemd, then you can use systemctl like this:

[support@Feynman ~] systemctl status postgresql
? postgresql.service - PostgreSQL database server
   Loaded: loaded (/usr/lib/systemd/system/postgresql.service; enabled; vendor preset: disabled)
   Active: active (running) since Wed 2017-11-14 12:33:35 CET; 7min ago
...

You can replace status with stop, start or restart as well as other actions. Please refer to the documentation for full details. In order to operate on multiple concurrent versions, the syntax is slightly different. For example to stop v12 and reload v13 you can run:

systemctl stop postgresql-12.service
systemctl reload postgresql-13.service

Thanks to @Jojo for pointing me to this very one. Finally Keep in mind that root permissions may be needed for non-informative tasks as in the other cases seen earlier.

How to move or copy files listed by 'find' command in unix?

If you're using GNU find,

find . -mtime 1 -exec cp -t ~/test/ {} +

This works as well as piping the output into xargs while avoiding the pitfalls of doing so (it handles embedded spaces and newlines without having to use find ... -print0 | xargs -0 ...).

Is there a sleep function in JavaScript?

A naive, CPU-intensive method to block execution for a number of milliseconds:

/**
* Delay for a number of milliseconds
*/
function sleep(delay) {
    var start = new Date().getTime();
    while (new Date().getTime() < start + delay);
}

Python strftime - date without leading 0?

Some platforms may support width and precision specification between % and the letter (such as 'd' for day of month), according to http://docs.python.org/library/time.html -- but it's definitely a non-portable solution (e.g. doesn't work on my Mac;-). Maybe you can use a string replace (or RE, for really nasty format) after the strftime to remedy that? e.g.:

>>> y
(2009, 5, 7, 17, 17, 17, 3, 127, 1)
>>> time.strftime('%Y %m %d', y)
'2009 05 07'
>>> time.strftime('%Y %m %d', y).replace(' 0', ' ')
'2009 5 7'

How to solve SyntaxError on autogenerated manage.py?

I solved same situation.

INSTALLED VERSION

python 3.6, django 2.1

SITUATION

I installed Node.js in Windows 10. After python manage.py runserver caused error.

ERROR

File "manage.py", line 14
) from exc
^
SyntaxError: invalid syntax

REASON

My python path changed to python-2.7 from python-3.6. (3.6 is correct in my PC.)

SOLUTION

Fix python path.

How to fix Error: "Could not find schema information for the attribute/element" by creating schema

In my case I had an ambiguous reference in my code. I restarted Visual Studio and was able to see the error message. When I resolved this the other error disappeared.

Job for httpd.service failed because the control process exited with error code. See "systemctl status httpd.service" and "journalctl -xe" for details

In my case I got the error simply because I had changed the Listen 80 to listen 443 in the file

/etc/httpd/conf/httpd.conf 

Since I had installed mod_ssl using the yum commands

yum -y install mod_ssl  

there was a duplicate listen 443 directive in the file ssl.conf created during mod_ssl installation.

You can verify this if you have duplicate listen 80 or 443 by running the below command in linux centos (My linux)

grep  '443' /etc/httpd/conf.d/*

below is sample output

/etc/httpd/conf.d/ssl.conf:Listen 443 https
/etc/httpd/conf.d/ssl.conf:<VirtualHost _default_:443>
/etc/httpd/conf.d/ssl.conf:#ServerName www.example.com:443

Simply reverting the listen 443 in httd.conf to listen 80 fixed my issue.

MySQL: Quick breakdown of the types of joins

I have 2 tables like this:

> SELECT * FROM table_a;
+------+------+
| id   | name |
+------+------+
|    1 | row1 |
|    2 | row2 |
+------+------+

> SELECT * FROM table_b;
+------+------+------+
| id   | name | aid  |
+------+------+------+
|    3 | row3 |    1 |
|    4 | row4 |    1 |
|    5 | row5 | NULL |
+------+------+------+

INNER JOIN cares about both tables

INNER JOIN cares about both tables, so you only get a row if both tables have one. If there is more than one matching pair, you get multiple rows.

> SELECT * FROM table_a a INNER JOIN table_b b ON a.id=b.aid;
+------+------+------+------+------+
| id   | name | id   | name | aid  |
+------+------+------+------+------+
|    1 | row1 |    3 | row3 | 1    |
|    1 | row1 |    4 | row4 | 1    |
+------+------+------+------+------+

It makes no difference to INNER JOIN if you reverse the order, because it cares about both tables:

> SELECT * FROM table_b b INNER JOIN table_a a ON a.id=b.aid;
+------+------+------+------+------+
| id   | name | aid  | id   | name |
+------+------+------+------+------+
|    3 | row3 | 1    |    1 | row1 |
|    4 | row4 | 1    |    1 | row1 |
+------+------+------+------+------+

You get the same rows, but the columns are in a different order because we mentioned the tables in a different order.

LEFT JOIN only cares about the first table

LEFT JOIN cares about the first table you give it, and doesn't care much about the second, so you always get the rows from the first table, even if there is no corresponding row in the second:

> SELECT * FROM table_a a LEFT JOIN table_b b ON a.id=b.aid;
+------+------+------+------+------+
| id   | name | id   | name | aid  |
+------+------+------+------+------+
|    1 | row1 |    3 | row3 | 1    |
|    1 | row1 |    4 | row4 | 1    |
|    2 | row2 | NULL | NULL | NULL |
+------+------+------+------+------+

Above you can see all rows of table_a even though some of them do not match with anything in table b, but not all rows of table_b - only ones that match something in table_a.

If we reverse the order of the tables, LEFT JOIN behaves differently:

> SELECT * FROM table_b b LEFT JOIN table_a a ON a.id=b.aid;
+------+------+------+------+------+
| id   | name | aid  | id   | name |
+------+------+------+------+------+
|    3 | row3 | 1    |    1 | row1 |
|    4 | row4 | 1    |    1 | row1 |
|    5 | row5 | NULL | NULL | NULL |
+------+------+------+------+------+

Now we get all rows of table_b, but only matching rows of table_a.

RIGHT JOIN only cares about the second table

a RIGHT JOIN b gets you exactly the same rows as b LEFT JOIN a. The only difference is the default order of the columns.

> SELECT * FROM table_a a RIGHT JOIN table_b b ON a.id=b.aid;
+------+------+------+------+------+
| id   | name | id   | name | aid  |
+------+------+------+------+------+
|    1 | row1 |    3 | row3 | 1    |
|    1 | row1 |    4 | row4 | 1    |
| NULL | NULL |    5 | row5 | NULL |
+------+------+------+------+------+

This is the same rows as table_b LEFT JOIN table_a, which we saw in the LEFT JOIN section.

Similarly:

> SELECT * FROM table_b b RIGHT JOIN table_a a ON a.id=b.aid;
+------+------+------+------+------+
| id   | name | aid  | id   | name |
+------+------+------+------+------+
|    3 | row3 | 1    |    1 | row1 |
|    4 | row4 | 1    |    1 | row1 |
| NULL | NULL | NULL |    2 | row2 |
+------+------+------+------+------+

Is the same rows as table_a LEFT JOIN table_b.

No join at all gives you copies of everything

If you write your tables with no JOIN clause at all, just separated by commas, you get every row of the first table written next to every row of the second table, in every possible combination:

> SELECT * FROM table_b b, table_a;
+------+------+------+------+------+
| id   | name | aid  | id   | name |
+------+------+------+------+------+
|    3 | row3 | 1    |    1 | row1 |
|    3 | row3 | 1    |    2 | row2 |
|    4 | row4 | 1    |    1 | row1 |
|    4 | row4 | 1    |    2 | row2 |
|    5 | row5 | NULL |    1 | row1 |
|    5 | row5 | NULL |    2 | row2 |
+------+------+------+------+------+

(This is from my blog post Examples of SQL join types)

Ways to implement data versioning in MongoDB

Another option is to use mongoose-history plugin.

let mongoose = require('mongoose');
let mongooseHistory = require('mongoose-history');
let Schema = mongoose.Schema;

let MySchema = Post = new Schema({
    title: String,
    status: Boolean
});

MySchema.plugin(mongooseHistory);
// The plugin will automatically create a new collection with the schema name + "_history".
// In this case, collection with name "my_schema_history" will be created.

How to set cookie in node js using express framework?

The order in which you use middleware in Express matters: middleware declared earlier will get called first, and if it can handle a request, any middleware declared later will not get called.

If express.static is handling the request, you need to move your middleware up:

// need cookieParser middleware before we can do anything with cookies
app.use(express.cookieParser());

// set a cookie
app.use(function (req, res, next) {
  // check if client sent cookie
  var cookie = req.cookies.cookieName;
  if (cookie === undefined) {
    // no: set a new cookie
    var randomNumber=Math.random().toString();
    randomNumber=randomNumber.substring(2,randomNumber.length);
    res.cookie('cookieName',randomNumber, { maxAge: 900000, httpOnly: true });
    console.log('cookie created successfully');
  } else {
    // yes, cookie was already present 
    console.log('cookie exists', cookie);
  } 
  next(); // <-- important!
});

// let static middleware do its job
app.use(express.static(__dirname + '/public'));

Also, middleware needs to either end a request (by sending back a response), or pass the request to the next middleware. In this case, I've done the latter by calling next() when the cookie has been set.

Update

As of now the cookie parser is a seperate npm package, so instead of using

app.use(express.cookieParser());

you need to install it separately using npm i cookie-parser and then use it as:

const cookieParser = require('cookie-parser');
app.use(cookieParser());

Error: Can't set headers after they are sent to the client

I got a similar error when I tried to send response within a loop function. The simple solution was to move the

res.send('send response');

out of the loop since you can only send response header once.

https://www.tutorialspoint.com/nodejs/nodejs_response_object.htm

an htop-like tool to display disk activity in linux

You could use iotop. It doesn't rely on a kernel patch. It Works with stock Ubuntu kernel

There is a package for it in the Ubuntu repos. You can install it using

sudo apt-get install iotop

iotop

What does "O(1) access time" mean?

"Big O notation" is a way to express the speed of algorithms. n is the amount of data the algorithm is working with. O(1) means that, no matter how much data, it will execute in constant time. O(n) means that it is proportional to the amount of data.

Jquery mouseenter() vs mouseover()

Old question, but still no good up-to-date answer with insight imo.

As jQuery uses Javascript wording for events and handlers, but does its own undocumented, but different interpretation of those, let me first shed light on the difference from the pure Javascript viewpoint:

  • both event pairs
    • the mouse can “jump” from outside/outer elements to inner/innermost elements when moved faster than the browser samples its position
    • any enter/over gets a corresponding leave/out (possibly late/jumpy)
    • events go to the visible element below the pointer (invisible elements can’t be target)
  • mouseenter/mouseleave
    • does not bubble (event not useful for delegate handlers)
    • the event registration itself defines the area of observation and abstraction
    • works on the target area, like a park with a pond: the pond is considered part of the park
    • the event is emitted on the target/area whenever the element itself or any descendant directly is entered/left the first time
    • entering a descendant, moving from one descendant to another or moving back into the target does not finish/restart the mouseenter/mouseleave cycle (i.e. no events fire)
    • if you want to observe multiple areas with one handler, register it on each area/element or use the other event pair discussed next
    • descendants of registered areas/elements can have their own handlers, creating an independent observation area with its independent mouseenter/mouseleave event cycles
    • if you think about how a bubbling version of mouseenter/mouseleave could look like, you end up with with something like mouseover/mouseout
  • mouseover/mouseout
    • events bubble
    • events fire whenever the element below the pointer changes
      • mouseout on the previously sampled element
      • followed by mouseover on the new element
      • the events don’t “nest”: before e.g. a child is “overed” the parent will be “out”
    • target/relatedTarget indicate new and previous element
    • if you want to watch different areas
      • register one handler on a common parent (or multiple parents, which together cover all elements you want to watch)
      • look for the element you are interested in between the handler element and the target element; maybe $(event.target).closest(...) suits your needs

Not-so-trivial mouseover/mouseout example:

$('.side-menu, .top-widget')
  .on('mouseover mouseout', event => {
    const target = event.type === 'mouseover' ? event.target : event.relatedTarget;
    const thing = $(target).closest('[data-thing]').attr('data-thing') || 'default';
    // do something with `thing`
  });

These days, all browsers support mouseover/mouseout and mouseenter/mouseleave natively. Nevertheless, jQuery does not register your handler to mouseenter/mouseleave, but silently puts them on a wrappers around mouseover/mouseout as the code below exposes.

The emulation is unnecessary, imperfect and a waste of CPU cycles: it filters out mouseover/mouseout events that a mouseenter/mouseleave would not get, but the target is messed. The real mouseenter/mouseleave would give the handler element as target, the emulation might indicate children of that element, i.e. whatever the mouseover/mouseout carried.

For that reason I do not use jQuery for those events, but e.g.:

$el[0].addEventListener('mouseover', e => ...);

_x000D_
_x000D_
const list = document.getElementById('log');
const outer = document.getElementById('outer');
const $outer = $(outer);

function log(tag, event) {
  const li = list.insertBefore(document.createElement('li'), list.firstChild);
  // only jQuery handlers have originalEvent
  const e = event.originalEvent || event;
  li.append(`${tag} got ${e.type} on ${e.target.id}`);
}

outer.addEventListener('mouseenter', log.bind(null, 'JSmouseenter'));
$outer.on('mouseenter', log.bind(null, '$mouseenter'));
_x000D_
div {
  margin: 20px;
  border: solid black 2px;
}

#inner {
  min-height: 80px;
}
_x000D_
<script src="https://cdnjs.cloudflare.com/ajax/libs/jquery/3.3.1/jquery.min.js"></script>

<body>
  <div id=outer>
    <ul id=log>
    </ul>
  </div>
</body>
_x000D_
_x000D_
_x000D_


Note: For delegate handlers never use jQuery’s “delegate handlers with selector registration”. (Reason in another answer.) Use this (or similar):

$(parent).on("mouseover", e => {
  if ($(e.target).closest('.gold').length) {...};
});

instead of

$(parent).on("mouseover", '.gold', e => {...});

event.preventDefault() vs. return false

When using jQuery, return false is doing 3 separate things when you call it:

  1. event.preventDefault();
  2. event.stopPropagation();
  3. Stops callback execution and returns immediately when called.

See jQuery Events: Stop (Mis)Using Return False for more information and examples.

What are the differences between a clustered and a non-clustered index?

Clustered Index

  1. There can be only one clustered index for a table.
  2. Usually made on the primary key.
  3. The leaf nodes of a clustered index contain the data pages.

Non-Clustered Index

  1. There can be only 249 non-clustered indexes for a table(till sql version 2005 later versions support upto 999 non-clustered indexes).
  2. Usually made on the any key.
  3. The leaf node of a nonclustered index does not consist of the data pages. Instead, the leaf nodes contain index rows.

type checking in javascript

Quite a few utility libraries such as YourJS offer functions for determining if something is an array or if something is an integer or a lot of other types as well. YourJS defines isInt by checking if the value is a number and then if it is divisible by 1:

function isInt(x) {
  return typeOf(x, 'Number') && x % 1 == 0;
}

The above snippet was taken from this YourJS snippet and thusly only works because typeOf is defined by the library. You can download a minimalistic version of YourJS which mainly only has type checking functions such as typeOf(), isInt() and isArray(): http://yourjs.com/snippets/build/34,2

DELETE ... FROM ... WHERE ... IN

You can achieve this using exists:

DELETE
  FROM table1
 WHERE exists(
           SELECT 1
             FROM table2
            WHERE table2.stn = table1.stn
              and table2.jaar = year(table1.datum)
       )

Anchor links in Angularjs?

The best choice to me was to create a directive to do the work, because $location.hash() and $anchorScroll() hijack the URL creating lots of problems to my SPA routing.

MyModule.directive('myAnchor', function() {
  return {
    restrict: 'A',
    require: '?ngModel',
    link: function(scope, elem, attrs, ngModel) {
      return elem.bind('click', function() {
        //other stuff ...
        var el;
        el = document.getElementById(attrs['myAnchor']);
        return el.scrollIntoView();
      });      
    }
  };
});

How to hide the border for specified rows of a table?

I use this with good results:

border-style:hidden;

It also works for:

border-right-style:hidden; /*if you want to hide just a border on a cell*/

Example:

_x000D_
_x000D_
<style type="text/css">_x000D_
      table, th, td {_x000D_
       border: 2px solid green;_x000D_
      }_x000D_
      tr.hide_right > td, td.hide_right{_x000D_
        border-right-style:hidden;_x000D_
      }_x000D_
      tr.hide_all > td, td.hide_all{_x000D_
        border-style:hidden;_x000D_
      }_x000D_
  }_x000D_
</style>_x000D_
<table>_x000D_
  <tr>_x000D_
    <td class="hide_right">11</td>_x000D_
    <td>12</td>_x000D_
    <td class="hide_all">13</td>_x000D_
  </tr>_x000D_
  <tr class="hide_right">_x000D_
    <td>21</td>_x000D_
    <td>22</td>_x000D_
    <td>23</td>_x000D_
  </tr>_x000D_
  <tr class="hide_all">_x000D_
    <td>31</td>_x000D_
    <td>32</td>_x000D_
    <td>33</td>_x000D_
  </tr>_x000D_
</table>
_x000D_
_x000D_
_x000D_

Here is the result: enter image description here

How to run a Maven project from Eclipse?

(Alt + Shift + X) , then M to Run Maven Build. You will need to specify the Maven goals you want on Run -> Run Configurations

How to get user name using Windows authentication in asp.net?

Username you get like this:

var userName = HttpContext.Current.Request.LogonUserIdentity?.Name;

Xcode 10, Command CodeSign failed with a nonzero exit code

Fix

Remove extended file attributes in your resource files for good, not in the compiled application bundle:

  1. Open Terminal

  2. Change directory to the root of your source files

    $ cd /Users/rjobidon/Documents/My\ Project

  3. List all extended attributes

$ xattr -lr .
  1. Remove all extended attributes
$ xattr -cr .

Xcode errors

  • "Command CodeSign failed with a nonzero exit code"
  • "Resource fork, Finder information, or similar detritus not allowed"

Cause

Apple introduced a security hardening change, thus code signing no longer allows any file in an app bundle to have an extended attribute containing a resource fork or Finder info.

Sources

How to get all of the IDs with jQuery?

Not a real array, but objs are all associative arrays in javascript.

I chose not to use a real array with [] and [].push because technically, you can have multiple ID's on a page even though that is incorrect to do so. So just another option in case some html has duplicated ID's

$(function() {

       var oArr = {};
       $("*[id]").each(function() {
           var id = $(this).attr('id');
           if (!oArr[id]) oArr[id] = true;
       });

       for (var prop in oArr)
           alert(prop);

   });

push object into array

Well, ["Title", "Ramones"] is an array of strings. But [{"01":"Title", "02", "Ramones"}] is an array of object.

If you are willing to push properties or value into one object, you need to access that object and then push data into that. Example: nietos[indexNumber].yourProperty=yourValue; in real application:

nietos[0].02 = "Ramones";

If your array of object is already empty, make sure it has at least one object, or that object in which you are going to push data to.

Let's say, our array is myArray[], so this is now empty array, the JS engine does not know what type of data does it have, not string, not object, not number nothing. So, we are going to push an object (maybe empty object) into that array. myArray.push({}), or myArray.push({""}).

This will push an empty object into myArray which will have an index number 0, so your exact object is now myArray[0]

Then push property and value into that like this:

myArray[0].property = value;
//in your case:
myArray[0]["01"] = "value";

Splitting on last delimiter in Python string?

You can use rsplit

string.rsplit('delimeter',1)[1]

To get the string from reverse.

Error retrieving parent for item: No resource found that matches the given name after upgrading to AppCompat v23

As pointed out by Tanis.7x, the support library version (23) does not match the targetSdkVersion (22)

You can fix this by doing the following:

In the build.grade file of your app module, change the following line of code

compile 'com.android.support:appcompat-v7:23.0.0'

To

compile 'com.android.support:appcompat-v7:22.+'

This will use the latest version of the appcompat version that is compatible with SdkVersion 22.

Google Play on Android 4.0 emulator

Have you ever tried Genymotion? I've read about it last week and it is great. They have several Android Images that you run (with their own software). The images are INCREDIBLY fast and they have Google Play installed on them. Check it out if it is the kind of thing that you need.

http://www.genymotion.com/

Returning value from Thread

Using Future described in above answers does the job, but a bit less significantly as f.get(), blocks the thread until it gets the result, which violates concurrency.

Best solution is to use Guava's ListenableFuture. An example :

    ListenableFuture<Void> future = MoreExecutors.listeningDecorator(Executors.newFixedThreadPool(1, new NamedThreadFactory).submit(new Callable<Void>()
    {
        @Override
        public Void call() throws Exception
        {
            someBackgroundTask();
        }
    });
    Futures.addCallback(future, new FutureCallback<Long>()
    {
        @Override
        public void onSuccess(Long result)
        {
            doSomething();
        }

        @Override
        public void onFailure(Throwable t)
        {

        }
    };

Session 'app': Error Installing APK

I tried invalidating cache, deleting build folder and gradle sync. Also, I couldn't uninstall because the app is not visible on device. So I tried uninstalling through ADB and it worked.

adb uninstall <package_name>

How to change style of a default EditText

edittext_selector.xml

<?xml version="1.0" encoding="utf-8"?>
<selector xmlns:android="http://schemas.android.com/apk/res/android">
    <item android:drawable="@drawable/edittext_pressed" android:state_pressed="true" /> <!-- pressed -->
    <item android:drawable="@drawable/edittext_disable" android:state_enabled="false" /> <!-- focused -->
    <item android:drawable="@drawable/edittext_default" /> <!-- default -->
</selector>

edittext_default.xml

       <?xml version="1.0" encoding="utf-8"?>
<layer-list xmlns:android="http://schemas.android.com/apk/res/android" >
    <item>
        <shape android:shape="rectangle" >
            <solid android:color="#BBDEFB" />
            <padding android:bottom="2dp" />
        </shape>
    </item>
    <item android:bottom="5dp">
        <shape android:shape="rectangle" >
            <solid android:color="#fff" />

            <padding
                android:left="0dp"
                android:right="0dp" />
        </shape>
    </item>
    <item>
        <shape android:shape="rectangle" >
            <solid android:color="#fff" />
        </shape>
    </item>
</layer-list>

edittext_pressed.xml

 <?xml version="1.0" encoding="utf-8"?>
<layer-list xmlns:android="http://schemas.android.com/apk/res/android" >
    <item>
        <shape android:shape="rectangle" >
            <solid android:color="#00f" />
            <padding android:bottom="2dp" />
        </shape>
    </item>
    <item android:bottom="5dp">
        <shape android:shape="rectangle" >
            <solid android:color="#fff" />

            <padding
                android:left="0dp"
                android:right="0dp" />
        </shape>
    </item>
    <item>
        <shape android:shape="rectangle" >
            <solid android:color="#fff" />
        </shape>
    </item>

</layer-list>

edittext_disable.xml

    <?xml version="1.0" encoding="utf-8"?>
    <layer-list xmlns:android="http://schemas.android.com/apk/res/android" >
        <item>
            <shape android:shape="rectangle" >
                <solid android:color="#aaaaaa" />
                <padding android:bottom="2dp" />
            </shape>
        </item>
        <item android:bottom="5dp">
            <shape android:shape="rectangle" >
                <solid android:color="#fff" />

                <padding
                    android:left="0dp"
                    android:right="0dp" />
            </shape>
        </item>
        <item>
            <shape android:shape="rectangle" >
                <solid android:color="#fff" />
            </shape>
        </item>

    </layer-list>

it works fine without nine-patch Api 10+ enter image description here

What are the various "Build action" settings in Visual Studio project properties and what do they do?

VS2010 has a property for 'Build Action', and also for 'Copy to Output Directory'. So an action of 'None' will still copy over to the build directory if the copy property is set to 'Copy if Newer' or 'Copy Always'.

So a Build Action of 'Content' should be reserved to indicate content you will access via 'Application.GetContentStream'

I used the 'Build Action' setting of 'None' and the 'Copy to Output Direcotry' setting of 'Copy if Newer' for some externally linked .config includes.

G.

How do I shutdown, restart, or log off Windows via a bat file?

No one has mentioned -m option for remote shutdown:

shutdown -r -f -m \\machinename

Also:

  • The -r parameter causes a reboot (which is usually what you want on a remote machine, since physically starting it might be difficult).
  • The -f parameter option forces the reboot.
  • You must have appropriate privileges to shut down the remote machine, of course.

Unable to resolve host "<insert URL here>" No address associated with hostname

This error because of you host cann't be translate to IP addresses via DNS.

Solve of this problem :

1- Make sure you connect to the internet (check quality of network).

2- Make sure you take proper permission to access network

<uses-permission android:name="android.permission.INTERNET" />
<uses-permission android:name="android.permission.ACCESS_NETWORK_STATE" />

What is the equivalent of 'describe table' in SQL Server?

This is the code I use within the EntityFramework Reverse POCO Generator (available here)

Table SQL:

SELECT  c.TABLE_SCHEMA AS SchemaName,
        c.TABLE_NAME AS TableName,
        t.TABLE_TYPE AS TableType,
        c.ORDINAL_POSITION AS Ordinal,
        c.COLUMN_NAME AS ColumnName,
        CAST(CASE WHEN IS_NULLABLE = 'YES' THEN 1
                  ELSE 0
             END AS BIT) AS IsNullable,
        DATA_TYPE AS TypeName,
        ISNULL(CHARACTER_MAXIMUM_LENGTH, 0) AS [MaxLength],
        CAST(ISNULL(NUMERIC_PRECISION, 0) AS INT) AS [Precision],
        ISNULL(COLUMN_DEFAULT, '') AS [Default],
        CAST(ISNULL(DATETIME_PRECISION, 0) AS INT) AS DateTimePrecision,
        ISNULL(NUMERIC_SCALE, 0) AS Scale,
        CAST(COLUMNPROPERTY(OBJECT_ID(QUOTENAME(c.TABLE_SCHEMA) + '.' + QUOTENAME(c.TABLE_NAME)), c.COLUMN_NAME, 'IsIdentity') AS BIT) AS IsIdentity,
        CAST(CASE WHEN COLUMNPROPERTY(OBJECT_ID(QUOTENAME(c.TABLE_SCHEMA) + '.' + QUOTENAME(c.TABLE_NAME)), c.COLUMN_NAME, 'IsIdentity') = 1 THEN 1
                  WHEN COLUMNPROPERTY(OBJECT_ID(QUOTENAME(c.TABLE_SCHEMA) + '.' + QUOTENAME(c.TABLE_NAME)), c.COLUMN_NAME, 'IsComputed') = 1 THEN 1
                  WHEN DATA_TYPE = 'TIMESTAMP' THEN 1
                  ELSE 0
             END AS BIT) AS IsStoreGenerated,
        CAST(CASE WHEN pk.ORDINAL_POSITION IS NULL THEN 0
                  ELSE 1
             END AS BIT) AS PrimaryKey,
        ISNULL(pk.ORDINAL_POSITION, 0) PrimaryKeyOrdinal,
        CAST(CASE WHEN fk.COLUMN_NAME IS NULL THEN 0
                  ELSE 1
             END AS BIT) AS IsForeignKey
FROM    INFORMATION_SCHEMA.COLUMNS c
        LEFT OUTER JOIN (SELECT u.TABLE_SCHEMA,
                                u.TABLE_NAME,
                                u.COLUMN_NAME,
                                u.ORDINAL_POSITION
                         FROM   INFORMATION_SCHEMA.KEY_COLUMN_USAGE u
                                INNER JOIN INFORMATION_SCHEMA.TABLE_CONSTRAINTS tc
                                    ON u.TABLE_SCHEMA = tc.CONSTRAINT_SCHEMA
                                       AND u.TABLE_NAME = tc.TABLE_NAME
                                       AND u.CONSTRAINT_NAME = tc.CONSTRAINT_NAME
                         WHERE  CONSTRAINT_TYPE = 'PRIMARY KEY') pk
            ON c.TABLE_SCHEMA = pk.TABLE_SCHEMA
               AND c.TABLE_NAME = pk.TABLE_NAME
               AND c.COLUMN_NAME = pk.COLUMN_NAME
        LEFT OUTER JOIN (SELECT DISTINCT
                                u.TABLE_SCHEMA,
                                u.TABLE_NAME,
                                u.COLUMN_NAME
                         FROM   INFORMATION_SCHEMA.KEY_COLUMN_USAGE u
                                INNER JOIN INFORMATION_SCHEMA.TABLE_CONSTRAINTS tc
                                    ON u.TABLE_SCHEMA = tc.CONSTRAINT_SCHEMA
                                       AND u.TABLE_NAME = tc.TABLE_NAME
                                       AND u.CONSTRAINT_NAME = tc.CONSTRAINT_NAME
                         WHERE  CONSTRAINT_TYPE = 'FOREIGN KEY') fk
            ON c.TABLE_SCHEMA = fk.TABLE_SCHEMA
               AND c.TABLE_NAME = fk.TABLE_NAME
               AND c.COLUMN_NAME = fk.COLUMN_NAME
        INNER JOIN INFORMATION_SCHEMA.TABLES t
            ON c.TABLE_SCHEMA = t.TABLE_SCHEMA
               AND c.TABLE_NAME = t.TABLE_NAME
WHERE c.TABLE_NAME NOT IN ('EdmMetadata', '__MigrationHistory')

Foreign Key SQL:

SELECT  FK.name AS FK_Table,
        FkCol.name AS FK_Column,
        PK.name AS PK_Table,
        PkCol.name AS PK_Column,
        OBJECT_NAME(f.object_id) AS Constraint_Name,
        SCHEMA_NAME(FK.schema_id) AS fkSchema,
        SCHEMA_NAME(PK.schema_id) AS pkSchema,
        PkCol.name AS primarykey,
        k.constraint_column_id AS ORDINAL_POSITION
FROM    sys.objects AS PK
        INNER JOIN sys.foreign_keys AS f
            INNER JOIN sys.foreign_key_columns AS k
                ON k.constraint_object_id = f.object_id
            INNER JOIN sys.indexes AS i
                ON f.referenced_object_id = i.object_id
                   AND f.key_index_id = i.index_id
            ON PK.object_id = f.referenced_object_id
        INNER JOIN sys.objects AS FK
            ON f.parent_object_id = FK.object_id
        INNER JOIN sys.columns AS PkCol
            ON f.referenced_object_id = PkCol.object_id
               AND k.referenced_column_id = PkCol.column_id
        INNER JOIN sys.columns AS FkCol
            ON f.parent_object_id = FkCol.object_id
               AND k.parent_column_id = FkCol.column_id
ORDER BY FK_Table, FK_Column

Extended Properties:

SELECT  s.name AS [schema],
        t.name AS [table],
        c.name AS [column],
        value AS [property]
FROM    sys.extended_properties AS ep
        INNER JOIN sys.tables AS t
            ON ep.major_id = t.object_id
        INNER JOIN sys.schemas AS s
            ON s.schema_id = t.schema_id
        INNER JOIN sys.columns AS c
            ON ep.major_id = c.object_id
               AND ep.minor_id = c.column_id
WHERE   class = 1
ORDER BY t.name

Javascript, Time and Date: Getting the current minute, hour, day, week, month, year of a given millisecond time

The variable names should be descriptive:

var date = new Date;
date.setTime(result_from_Date_getTime);

var seconds = date.getSeconds();
var minutes = date.getMinutes();
var hour = date.getHours();

var year = date.getFullYear();
var month = date.getMonth(); // beware: January = 0; February = 1, etc.
var day = date.getDate();

var dayOfWeek = date.getDay(); // Sunday = 0, Monday = 1, etc.
var milliSeconds = date.getMilliseconds();

The days of a given month do not change. In a leap year, February has 29 days. Inspired by http://www.javascriptkata.com/2007/05/24/how-to-know-if-its-a-leap-year/ (thanks Peter Bailey!)

Continued from the previous code:

var days_in_months = [31, 28, 31, 30, 31, 30, 31, 31, 30, 31, 30, 31];
// for leap years, February has 29 days. Check whether
// February, the 29th exists for the given year
if( (new Date(year, 1, 29)).getDate() == 29 ) days_in_month[1] = 29;

There is no straightforward way to get the week of a year. For the answer on that question, see Is there a way in javascript to create a date object using year & ISO week number?

Vim multiline editing like in sublimetext?

I'm not sure what vim is doing, but it is an interesting effect. The way you're describing what you want sounds more like how macros work (:help macro). Something like this would do what you want with macros (starting in normal-mode):

  1. qa: Record macro to a register.
  2. 0w: 0 goto start of line, w jump one word.
  3. i"<Esc>: Enter insert-mode, insert a " and return to normal-mode.
  4. 2e: Jump to end of second word.
  5. a"<Esc>: Append a ".
  6. jq Move to next line and end macro recording.

Taken together: qa0wi"<Esc>2ea"<Esc>

Now you can execute the macro with @a, repeat last macro with @@. To apply to the rest of the file, do something like 99@a which assumes you do not have more than 99 lines, macro execution will end when it reaches end of file.

Here is how to achieve what you want with visual-block-mode (starting in normal mode):

  1. Navigate to where you want the first quote to be.
  2. Enter visual-block-mode, select the lines you want to affect, G to go to the bottom of the file.
  3. Hit I"<Esc>.
  4. Move to the next spot you want to insert a ".
  5. You want to repeat what you just did so a simple . will suffice.

c++ parse int from string

Some handy quick functions (if you're not using Boost):

template<typename T>
std::string ToString(const T& v)
{
    std::ostringstream ss;
    ss << v;
    return ss.str();
}

template<typename T>
T FromString(const std::string& str)
{
    std::istringstream ss(str);
    T ret;
    ss >> ret;
    return ret;
}

Example:

int i = FromString<int>(s);
std::string str = ToString(i);

Works for any streamable types (floats etc). You'll need to #include <sstream> and possibly also #include <string>.

How to show another window from mainwindow in QT

  1. Implement a slot in your QMainWindow where you will open your new Window,
  2. Place a widget on your QMainWindow,
  3. Connect a signal from this widget to a slot from the QMainWindow (for example: if the widget is a QPushButton connect the signal click() to the QMainWindow custom slot you have created).

Code example:

MainWindow.h

// ...
include "newwindow.h"
// ...
public slots:
   void openNewWindow();
// ...
private:
   NewWindow *mMyNewWindow;
// ...
}

MainWindow.cpp

// ...
   MainWindow::MainWindow()
   {
      // ...
      connect(mMyButton, SIGNAL(click()), this, SLOT(openNewWindow()));
      // ...
   }
// ...
void MainWindow::openNewWindow()
{
   mMyNewWindow = new NewWindow(); // Be sure to destroy your window somewhere
   mMyNewWindow->show();
   // ...
}

This is an example on how display a custom new window. There are a lot of ways to do this.

Store mysql query output into a shell variable

If you have particular database name and a host on which you want the query to be executed then follow below query:

outputofquery=$(mysql -u"$dbusername" -p"$dbpassword" -h"$dbhostname" -e "SELECT A, B, C FROM table_a;" $dbname)

So to run the mysql queries you need to install mysql client on linux

Type safety: Unchecked cast

Well, first of all, you're wasting memory with the new HashMap creation call. Your second line completely disregards the reference to this created hashmap, making it then available to the garbage collector. So, don't do that, use:

private Map<String, String> someMap = (HashMap<String, String>)getApplicationContext().getBean("someMap");

Secondly, the compiler is complaining that you cast the object to a HashMap without checking if it is a HashMap. But, even if you were to do:

if(getApplicationContext().getBean("someMap") instanceof HashMap) {
    private Map<String, String> someMap = (HashMap<String, String>)getApplicationContext().getBean("someMap");
}

You would probably still get this warning. The problem is, getBean returns Object, so it is unknown what the type is. Converting it to HashMap directly would not cause the problem with the second case (and perhaps there would not be a warning in the first case, I'm not sure how pedantic the Java compiler is with warnings for Java 5). However, you are converting it to a HashMap<String, String>.

HashMaps are really maps that take an object as a key and have an object as a value, HashMap<Object, Object> if you will. Thus, there is no guarantee that when you get your bean that it can be represented as a HashMap<String, String> because you could have HashMap<Date, Calendar> because the non-generic representation that is returned can have any objects.

If the code compiles, and you can execute String value = map.get("thisString"); without any errors, don't worry about this warning. But if the map isn't completely of string keys to string values, you will get a ClassCastException at runtime, because the generics cannot block this from happening in this case.

Google Play app description formatting

Currently (July 2015), HTML escape sequences (&bull; &#8226;) do not work in browser version of Play Store, they're displayed as text. Though, Play Store app handles them as expected.

So, if you're after the unicode bullet point in your app/update description [that's what's got you here, most likely], just copy-paste the bullet character

PS You can also use unicode input combo to get the character

Linux: CtrlShiftu 2022 Enter or Space

Mac: Hold ? 2022 release ?

Windows: Hold Alt 2022 release Alt

Mac and Windows require some setup, read on Wikipedia

PPS If you're feeling creative, here's a good link with more copypastable symbols, but don't go too crazy, nobody likes clutter in what they read.

Difference between a Seq and a List in Scala

As @daniel-c-sobral said, List extends the trait Seq and is an abstract class implemented by scala.collection.immutable.$colon$colon (or :: for short), but technicalities aside, mind that most of lists and seqs we use are initialized in the form of Seq(1, 2, 3) or List(1, 2, 3) which both return scala.collection.immutable.$colon$colon, hence one can write:

var x: scala.collection.immutable.$colon$colon[Int] = null
x = Seq(1, 2, 3).asInstanceOf[scala.collection.immutable.$colon$colon[Int]]
x = List(1, 2, 3).asInstanceOf[scala.collection.immutable.$colon$colon[Int]]

As a result, I'd argue than the only thing that matters are the methods you want to expose, for instance to prepend you can use :: from List that I find redundant with +: from Seq and I personally stick to Seq by default.

Flexbox and Internet Explorer 11 (display:flex in <html>?)

Use another flex container to fix the min-height issue in IE10 and IE11:

HTML

<div class="ie-fixMinHeight">
    <div id="page">
        <div id="header"></div>
        <div id="content"></div>
        <div id="footer"></div>
    </div>
</div>

CSS

.ie-fixMinHeight {
    display:flex;
}

#page {
    min-height:100vh;
    width:100%;
    display:flex;
    flex-direction:column;
}

#content {
    flex-grow:1;
}

See a working demo.

  • Don't use flexbox layout directly on body because it screws up elements inserted via jQuery plugins (autocomplete, popup, etc.).
  • Don't use height:100% or height:100vh on your container because the footer will stick at the bottom of window and won't adapt to long content.
  • Use flex-grow:1 rather than flex:1 cause IE10 and IE11 default values for flex are 0 0 auto and not 0 1 auto.

Returning IEnumerable<T> vs. IQueryable<T>

In general you want to preserve the original static type of the query until it matters.

For this reason, you can define your variable as 'var' instead of either IQueryable<> or IEnumerable<> and you will know that you are not changing the type.

If you start out with an IQueryable<>, you typically want to keep it as an IQueryable<> until there is some compelling reason to change it. The reason for this is that you want to give the query processor as much information as possible. For example, if you're only going to use 10 results (you've called Take(10)) then you want SQL Server to know about that so that it can optimize its query plans and send you only the data you'll use.

A compelling reason to change the type from IQueryable<> to IEnumerable<> might be that you are calling some extension function that the implementation of IQueryable<> in your particular object either cannot handle or handles inefficiently. In that case, you might wish to convert the type to IEnumerable<> (by assigning to a variable of type IEnumerable<> or by using the AsEnumerable extension method for example) so that the extension functions you call end up being the ones in the Enumerable class instead of the Queryable class.

How do I add a auto_increment primary key in SQL Server database?

you can try this... ALTER TABLE Your_Table ADD table_ID int NOT NULL PRIMARY KEY auto_increment;

What is the difference between dim and set in vba

There's no reason to use set unless referring to an object reference. It's good practice to only use it in that context. For all other simple data types, just use an assignment operator. It's a good idea to dim (dimension) ALL variables however:

Examples of simple data types would be integer, long, boolean, string. These are just data types and do not have their own methods and properties.

Dim i as Integer
i = 5

Dim myWord as String
myWord = "Whatever I want"

An example of an object would be a Range, a Worksheet, or a Workbook. These have their own methods and properties.

Dim myRange as Range
Set myRange = Sheet1.Range("A1")

If you try to use the last line without Set, VB will throw an error. Now that you have an object declared you can access its properties and methods.

myString = myRange.Value

Different ways of loading a file as an InputStream

Use MyClass.class.getClassLoader().getResourceAsStream(path) to load resource associated with your code. Use MyClass.class.getResourceAsStream(path) as a shortcut, and for resources packaged within your class' package.

Use Thread.currentThread().getContextClassLoader().getResourceAsStream(path) to get resources that are part of client code, not tightly bounds to the calling code. You should be careful with this as the thread context class loader could be pointing at anything.

Bootstrap Collapse not Collapsing

jQuery is required ;-)

DEMO

<html>
<head>
    <link rel="stylesheet" href="http://netdna.bootstrapcdn.com/bootstrap/3.1.1/css/bootstrap.min.css">
    <!-- THIS LINE -->
    <script src="//code.jquery.com/jquery-1.11.0.min.js"></script>
    <script src="http://netdna.bootstrapcdn.com/bootstrap/3.1.1/js/bootstrap.min.js"></script>
</head>
<body>

<div class="panel-group" id="accordion">
    <div class="panel panel-default">
        <div class="panel-heading">
            <h4 class="panel-title">
                <a data-toggle="collapse" data-parent="#accordion" href="#collapseOne">
                    Collapsible Group Item #1
                </a>
            </h4>
        </div>
        <div id="collapseOne" class="panel-collapse collapse in">
            <div class="panel-body">
                Anim pariatur cliche reprehenderit
            </div>
        </div>
    </div>
</div>

</body>
</html>

How to upgrade PostgreSQL from version 9.6 to version 10.1 without losing data?

My solution for upgrading from Postgresql 11 to Postgresql 12 on Windows 10 is the following.

As a first remark you will need to be able stop and start the Postgresql service. You can do this by the following commands in Powershell.

Start: pg_ctl start -D “d:\postgresql\11\data”

Stop: pg_ctl stop -D “d:\postgresql\11\data”

Status: pg_ctl status -D “d:\postgresql\11\data”

It would be wise to make a backup before doing the upgrade. The Postgresql 11 instance must be running. Then to copy the globals do

pg_dumpall -U postgres -g -f d:\bakup\postgresql\11\globals.sql

and then for each database

pg_dump -U postgres -Fc <database> > d:\backup\postgresql\11\<database>.fc

or

pg_dump -U postgres -Fc -d <database> -f d:\backup\postgresql\11\<database>.fc

If not already done install Postgresql 12 (as Postgresql 11 is also installed this will be on port 5433)

Then to do the upgrade as follows:

1) Stop Postgresql 11 service (see above)

2) Edit the postgresql.conf file in d:\postgresql\12\data and change port = 5433 to port = 5432

3) Edit the windows user environment path (windows start then type env) to point to Postgresql 12 instead of Postresql 11

4) Run upgrade by entering the following command.

pg_upgrade `
-b “c:\program files\postgresql\11\bin” `
-B “c:\program files\postgresql\12\bin” `
-d “d:\postgresql\11\data” `
-D “d:\postgresql\12\data” --username=postgres

(In powershell use backtick (or backquote) ` to continue the command on the next line)

5) and finally start the new Postgresql 12 service

pg_ctl start -D “d:\postgresql\12\data”

Why am I getting the message, "fatal: This operation must be run in a work tree?"

Edited the config file and changed bare = true to bare = false

Decimal to Hexadecimal Converter in Java

Consider dec2m method below for conversion from dec to hex, oct or bin.

Sample output is

28 dec == 11100 bin 28 dec == 34 oct 28 dec == 1C hex

public class Conversion {
    public static void main(String[] argv) {
        int x = 28;                           // sample number
        if (argv.length > 0)
            x = Integer.parseInt(argv[0]);    // number from command line

        System.out.printf("%d dec == %s bin\n", i, dec2m(x, 2));
        System.out.printf("%d dec == %s oct\n", i, dec2m(x, 8));
        System.out.printf("%d dec == %s hex\n", i, dec2m(x, 16));
    }

    static String dec2m(int N, int m) {
        String s = "";
        for (int n = N; n > 0; n /= m) {
            int r = n % m;
            s = r < 10 ? r + s : (char) ('A' - 10 + r) + s;
        }
        return s;
    }
}

Vertically align text within a div

Add a vertical align to the CSS content #column-content strong too:

#column-content strong {
    ...
    vertical-align: middle;
}

Also see your updated example.

=== UPDATE ===

With a span around the other text and another vertical align:

HTML:

... <span>yet another text content that should be centered vertically</span> ...

CSS:

#column-content span {
    vertical-align: middle;
}

Also see the next example.

How do I enumerate the properties of a JavaScript object?

I found it... for (property in object) { // do stuff } will list all the properties, and therefore all the globally declared variables on the window object..

Relative paths based on file location instead of current working directory

@Martin Konecny's answer provides the correct answer, but - as he mentions - it only works if the actual script is not invoked through a symlink residing in a different directory.

This answer covers that case: a solution that also works when the script is invoked through a symlink or even a chain of symlinks:


Linux / GNU readlink solution:

If your script needs to run on Linux only or you know that GNU readlink is in the $PATH, use readlink -f, which conveniently resolves a symlink to its ultimate target:

 scriptDir=$(dirname -- "$(readlink -f -- "$BASH_SOURCE")")

Note that GNU readlink has 3 related options for resolving a symlink to its ultimate target's full path: -f (--canonicalize), -e (--canonicalize-existing), and -m (--canonicalize-missing) - see man readlink.
Since the target by definition exists in this scenario, any of the 3 options can be used; I've chosen -f here, because it is the most well-known one.


Multi-(Unix-like-)platform solution (including platforms with a POSIX-only set of utilities):

If your script must run on any platform that:

  • has a readlink utility, but lacks the -f option (in the GNU sense of resolving a symlink to its ultimate target) - e.g., macOS.

    • macOS uses an older version of the BSD implementation of readlink; note that recent versions of FreeBSD/PC-BSD do support -f.
  • does not even have readlink, but has POSIX-compatible utilities - e.g., HP-UX (thanks, @Charles Duffy).

The following solution, inspired by https://stackoverflow.com/a/1116890/45375, defines helper shell function, rreadlink(), which resolves a given symlink to its ultimate target in a loop - this function is in effect a POSIX-compliant implementation of GNU readlink's -e option, which is similar to the -f option, except that the ultimate target must exist.

Note: The function is a bash function, and is POSIX-compliant only in the sense that only POSIX utilities with POSIX-compliant options are used. For a version of this function that is itself written in POSIX-compliant shell code (for /bin/sh), see here.

  • If readlink is available, it is used (without options) - true on most modern platforms.

  • Otherwise, the output from ls -l is parsed, which is the only POSIX-compliant way to determine a symlink's target.
    Caveat: this will break if a filename or path contains the literal substring -> - which is unlikely, however.
    (Note that platforms that lack readlink may still provide other, non-POSIX methods for resolving a symlink; e.g., @Charles Duffy mentions HP-UX's find utility supporting the %l format char. with its -printf primary; in the interest of brevity the function does NOT try to detect such cases.)

  • An installable utility (script) form of the function below (with additional functionality) can be found as rreadlink in the npm registry; on Linux and macOS, install it with [sudo] npm install -g rreadlink; on other platforms (assuming they have bash), follow the manual installation instructions.

If the argument is a symlink, the ultimate target's canonical path is returned; otherwise, the argument's own canonical path is returned.

#!/usr/bin/env bash

# Helper function.
rreadlink() ( # execute function in a *subshell* to localize the effect of `cd`, ...

  local target=$1 fname targetDir readlinkexe=$(command -v readlink) CDPATH= 

  # Since we'll be using `command` below for a predictable execution
  # environment, we make sure that it has its original meaning.
  { \unalias command; \unset -f command; } &>/dev/null

  while :; do # Resolve potential symlinks until the ultimate target is found.
      [[ -L $target || -e $target ]] || { command printf '%s\n' "$FUNCNAME: ERROR: '$target' does not exist." >&2; return 1; }
      command cd "$(command dirname -- "$target")" # Change to target dir; necessary for correct resolution of target path.
      fname=$(command basename -- "$target") # Extract filename.
      [[ $fname == '/' ]] && fname='' # !! curiously, `basename /` returns '/'
      if [[ -L $fname ]]; then
        # Extract [next] target path, which is defined
        # relative to the symlink's own directory.
        if [[ -n $readlinkexe ]]; then # Use `readlink`.
          target=$("$readlinkexe" -- "$fname")
        else # `readlink` utility not available.
          # Parse `ls -l` output, which, unfortunately, is the only POSIX-compliant 
          # way to determine a symlink's target. Hypothetically, this can break with
          # filenames containig literal ' -> ' and embedded newlines.
          target=$(command ls -l -- "$fname")
          target=${target#* -> }
        fi
        continue # Resolve [next] symlink target.
      fi
      break # Ultimate target reached.
  done
  targetDir=$(command pwd -P) # Get canonical dir. path
  # Output the ultimate target's canonical path.
  # Note that we manually resolve paths ending in /. and /.. to make sure we
  # have a normalized path.
  if [[ $fname == '.' ]]; then
    command printf '%s\n' "${targetDir%/}"
  elif  [[ $fname == '..' ]]; then
    # Caveat: something like /var/.. will resolve to /private (assuming
    # /var@ -> /private/var), i.e. the '..' is applied AFTER canonicalization.
    command printf '%s\n' "$(command dirname -- "${targetDir}")"
  else
    command printf '%s\n' "${targetDir%/}/$fname"
  fi
)

# Determine ultimate script dir. using the helper function.
# Note that the helper function returns a canonical path.
scriptDir=$(dirname -- "$(rreadlink "$BASH_SOURCE")")

Cannot find the '@angular/common/http' module

Is this issue resolved.

I am getting this error: ERROR in node_modules/ngx-restangular/lib/ngx-restangular-http.d.ts(3,27): error TS2307: Cannot find module '@angular/common/http/src/response'.

After updating my angular to version=8

How to Save Console.WriteLine Output to Text File

Try if this works:

FileStream filestream = new FileStream("out.txt", FileMode.Create);
var streamwriter = new StreamWriter(filestream);
streamwriter.AutoFlush = true;
Console.SetOut(streamwriter);
Console.SetError(streamwriter);

What is .Net Framework 4 extended?

It's the part of the .NET Framework that isn't contained within the Client Profile. See MSDN for more info; specifically:

The .NET Framework is made up of the .NET Framework 4 Client Profile and .NET Framework 4 Extended components that exist separately in Programs and Features.

Deserializing a JSON into a JavaScript object

I think this should help:

Also documentations also prove that you can use require() for json files: https://www.bennadel.com/blog/2908-you-can-use-require-to-load-json-javascript-object-notation-files-in-node-js.htm

var jsonfile = require("./path/to/jsonfile.json");
node = jsonfile.adjacencies.nodeTo;
node2 = jsonfile.adjacencies.nodeFrom;
node3 = jsonfile.adjacencies.data.$color;
//other things.

Calculate time difference in minutes in SQL Server

Try this..

select starttime,endtime, case
  when DATEDIFF(minute,starttime,endtime) < 60  then DATEDIFF(minute,starttime,endtime) 
  when DATEDIFF(minute,starttime,endtime) >= 60
  then '60,'+ cast( (cast(DATEDIFF(minute,starttime,endtime) as int )-60) as nvarchar(50) )
end from TestTable123416

All You need is DateDiff..

Scale iFrame css width 100% like an image

I like this solution best. Simple, scalable, responsive. The idea here is to create a zero-height outer div with bottom padding set to the aspect ratio of the video. The iframe is scaled to 100% in both width and height, completely filling the outer container. The outer container automatically adjusts its height according to its width, and the iframe inside adjusts itself accordingly.

<div style="position:relative; width:100%; height:0px; padding-bottom:56.25%;">
    <iframe style="position:absolute; left:0; top:0; width:100%; height:100%"
        src="http://www.youtube.com/embed/RksyMaJiD8Y">
    </iframe>
</div>

The only variable here is the padding-bottom value in the outer div. It's 75% for 4:3 aspect ratio videos, and 56.25% for widescreen 16:9 aspect ratio videos.

An error occurred while collecting items to be installed (Access is denied)

I got this error on my ubuntu box until I ran eclipse as root and installed from there:

$ gksudo eclipse

Eclipse was trying to download the packages to /usr/lib/* where I don't have write permissions

Update style of a component onScroll in React.js

An example using classNames, React hooks useEffect, useState and styled-jsx:

import classNames from 'classnames'
import { useEffect, useState } from 'react'

const Header = _ => {
  const [ scrolled, setScrolled ] = useState()
  const classes = classNames('header', {
    scrolled: scrolled,
  })
  useEffect(_ => {
    const handleScroll = _ => { 
      if (window.pageYOffset > 1) {
        setScrolled(true)
      } else {
        setScrolled(false)
      }
    }
    window.addEventListener('scroll', handleScroll)
    return _ => {
      window.removeEventListener('scroll', handleScroll)
    }
  }, [])
  return (
    <header className={classes}>
      <h1>Your website</h1>
      <style jsx>{`
        .header {
          transition: background-color .2s;
        }
        .header.scrolled {
          background-color: rgba(0, 0, 0, .1);
        }
      `}</style>
    </header>
  )
}
export default Header

What is content-type and datatype in an AJAX request?

See http://api.jquery.com/jQuery.ajax/, there's mention of datatype and contentType there.

They are both used in the request to the server so the server knows what kind of data to receive/send.

Why does adb return offline after the device string?

Try the following:

  1. Unplug the usb and plug it back again.

  2. Go to the Settings -> Applications -> Development of your device and uncheck the USB debugging mode and then check it back again.

  3. Restart the adb on your PC. adb kill-server and then adb start-server

  4. Restart your device and try again.

How can I rotate an HTML <div> 90 degrees?

You need CSS to achieve this, e.g.:

#container_2 {
    -webkit-transform: rotate(90deg);
    -moz-transform: rotate(90deg);
    -o-transform: rotate(90deg);
    -ms-transform: rotate(90deg);
    transform: rotate(90deg);
}

Demo:

_x000D_
_x000D_
#container_2 {_x000D_
  width: 100px;_x000D_
  height: 100px;_x000D_
  border: 1px solid red;_x000D_
  -webkit-transform: rotate(45deg);_x000D_
  -moz-transform: rotate(45deg);_x000D_
  -o-transform: rotate(45deg);_x000D_
  -ms-transform: rotate(45deg);_x000D_
  transform: rotate(45deg);_x000D_
}
_x000D_
<div id="container_2"></div>
_x000D_
_x000D_
_x000D_

(There's 45 degrees rotation in the demo, so you can see the effect)

Note: The -o- and -moz- prefixes are no longer relevant and probably not required. IE9 requires -ms- and Safari and the Android browser require -webkit-


Update 2018: Vendor prefixes are not needed anymore. Only transform is sufficient. (thanks @rinogo)

calling server side event from html button control

just use this at the end of your button click event

protected void btnAddButton_Click(object sender, EventArgs e)
{
   ... save data routin 
     Response.Redirect(Request.Url.AbsoluteUri);
}

Align vertically using CSS 3

Note: This example uses the draft version of the Flexible Box Layout Module. It has been superseded by the incompatible modern specification.

Center the child elements of a div box by using the box-align and box-pack properties together.

Example:

div
{
width:350px;
height:100px;
border:1px solid black;

/* Internet Explorer 10 */
display:-ms-flexbox;
-ms-flex-pack:center;
-ms-flex-align:center;

/* Firefox */
display:-moz-box;
-moz-box-pack:center;
-moz-box-align:center;

/* Safari, Opera, and Chrome */
display:-webkit-box;
-webkit-box-pack:center;
-webkit-box-align:center;

/* W3C */
display:box;
box-pack:center;
box-align:center;
} 

How to format a float in javascript?

Be careful when using toFixed():

First, rounding the number is done using the binary representation of the number, which might lead to unexpected behaviour. For example

(0.595).toFixed(2) === '0.59'

instead of '0.6'.

Second, there's an IE bug with toFixed(). In IE (at least up to version 7, didn't check IE8), the following holds true:

(0.9).toFixed(0) === '0'

It might be a good idea to follow kkyy's suggestion or to use a custom toFixed() function, eg

function toFixed(value, precision) {
    var power = Math.pow(10, precision || 0);
    return String(Math.round(value * power) / power);
}

"Eliminate render-blocking CSS in above-the-fold content"

Consider using a package to automatically generate inline styles from your css files. A good one is Grunt Critical or Critical css for Laravel.

Retina displays, high-res background images

If you are planing to use the same image for retina and non-retina screen then here is the solution. Say that you have a image of 200x200 and have two icons in top row and two icon in bottom row. So, it's four quadrants.

.sprite-of-icons {
  background: url("../images/icons-in-four-quad-of-200by200.png") no-repeat;
  background-size: 100px 100px /* Scale it down to 50% rather using 200x200 */
}

.sp-logo-1 { background-position: 0 0; }

/* Reduce positioning of the icons down to 50% rather using -50px */
.sp-logo-2 { background-position: -25px 0 }
.sp-logo-3 { background-position: 0 -25px }
.sp-logo-3 { background-position: -25px -25px }

Scaling and positioning of the sprite icons to 50% than actual value, you can get the expected result.


Another handy SCSS mixin solution by Ryan Benhase.

/****************************
 HIGH PPI DISPLAY BACKGROUNDS
*****************************/

@mixin background-2x($path, $ext: "png", $w: auto, $h: auto, $pos: left top, $repeat: no-repeat) {

  $at1x_path: "#{$path}.#{$ext}";
  $at2x_path: "#{$path}@2x.#{$ext}";

  background-image: url("#{$at1x_path}");
  background-size: $w $h;
  background-position: $pos;
  background-repeat: $repeat;

  @media all and (-webkit-min-device-pixel-ratio : 1.5),
  all and (-o-min-device-pixel-ratio: 3/2),
  all and (min--moz-device-pixel-ratio: 1.5),
  all and (min-device-pixel-ratio: 1.5) {
    background-image: url("#{$at2x_path}"); 
  }
}

div.background {
  @include background-2x( 'path/to/image', 'jpg', 100px, 100px, center center, repeat-x );
}

For more info about above mixin READ HERE.

Get the last item in an array

Here's more Javascript art if you came here looking for it

In the spirit of another answer that used reduceRight(), but shorter:

[3, 2, 1, 5].reduceRight(a => a);

It relies on the fact that, in case you don't provide an initial value, the very last element is selected as the initial one (check the docs here). Since the callback just keeps returning the initial value, the last element will be the one being returned in the end.

Beware that this should be considered Javascript art and is by no means the way I would recommend doing it, mostly because it runs in O(n) time, but also because it hurts readability.

And now for the serious answer

The best way I see (considering you want it more concise than array[array.length - 1]) is this:

const last = a => a[a.length - 1];

Then just use the function:

last([3, 2, 1, 5])

The function is actually useful in case you're dealing with an anonymous array like [3, 2, 1, 5] used above, otherwise you'd have to instantiate it twice, which would be inefficient and ugly:

[3, 2, 1, 5][[3, 2, 1, 5].length - 1]

Ugh.

For instance, here's a situation where you have an anonymous array and you'd have to define a variable, but you can use last() instead:

last("1.2.3".split("."));

PHPMailer - SMTP ERROR: Password command failed when send mail from my server

Your mail won't be sent online unless you complete the two-step verification for your g-mail account and use that password.

What's the foolproof way to tell which version(s) of .NET are installed on a production Windows Server?

You can programmatically check the registry and a few other things as per this blog entry.

The registry key to look at is

[HKEY_LOCAL_MACHINE\SOFTWARE\Microsoft\NET Framework Setup\NDP\...]

C multi-line macro: do/while(0) vs scope block

Andrey Tarasevich provides the following explanation:

  1. On Google Groups
  2. On bytes.com

[Minor changes to formatting made. Parenthetical annotations added in square brackets []].

The whole idea of using 'do/while' version is to make a macro which will expand into a regular statement, not into a compound statement. This is done in order to make the use of function-style macros uniform with the use of ordinary functions in all contexts.

Consider the following code sketch:

if (<condition>)
  foo(a);
else
  bar(a);

where foo and bar are ordinary functions. Now imagine that you'd like to replace function foo with a macro of the above nature [named CALL_FUNCS]:

if (<condition>)
  CALL_FUNCS(a);
else
  bar(a);

Now, if your macro is defined in accordance with the second approach (just { and }) the code will no longer compile, because the 'true' branch of if is now represented by a compound statement. And when you put a ; after this compound statement, you finished the whole if statement, thus orphaning the else branch (hence the compilation error).

One way to correct this problem is to remember not to put ; after macro "invocations":

if (<condition>)
  CALL_FUNCS(a)
else
  bar(a);

This will compile and work as expected, but this is not uniform. The more elegant solution is to make sure that macro expand into a regular statement, not into a compound one. One way to achieve that is to define the macro as follows:

#define CALL_FUNCS(x) \
do { \
  func1(x); \
  func2(x); \
  func3(x); \
} while (0)

Now this code:

if (<condition>)
  CALL_FUNCS(a);
else
  bar(a);

will compile without any problems.

However, note the small but important difference between my definition of CALL_FUNCS and the first version in your message. I didn't put a ; after } while (0). Putting a ; at the end of that definition would immediately defeat the entire point of using 'do/while' and make that macro pretty much equivalent to the compound-statement version.

I don't know why the author of the code you quoted in your original message put this ; after while (0). In this form both variants are equivalent. The whole idea behind using 'do/while' version is not to include this final ; into the macro (for the reasons that I explained above).

Cannot make Project Lombok work on Eclipse

I am on Eclipse Neon, and after following the above steps, it still didnt work. import lombok.Data; was not being recognized.

After about an hour of looking around, i switched the version to 1.16.14 and it worked.

Now my thought is, whether the 1 hour spent will be a good investment for long term :-)

Waiting till the async task finish its work

In your AsyncTask add one ProgressDialog like:

private final ProgressDialog dialog = new ProgressDialog(YourActivity.this);

you can setMessage in onPreExecute() method like:

this.dialog.setMessage("Processing..."); 
this.dialog.show();

and in your onPostExecute(Void result) method dismiss your ProgressDialog.

Sending Arguments To Background Worker?

You can use the DoWorkEventArgs.Argument property.

A full example (even using an int argument) can be found on Microsoft's site:

react-router getting this.props.location in child components

(Update) V5.1 & Hooks (Requires React >= 16.8)

You can use useHistory, useLocation and useRouteMatch in your component to get match, history and location .

const Child = () => {
  const location = useLocation();
  const history = useHistory();
  const match = useRouteMatch("write-the-url-you-want-to-match-here");

  return (
    <div>{location.pathname}</div>
  )
}

export default Child

(Update) V4 & V5

You can use withRouter HOC in order to inject match, history and location in your component props.

class Child extends React.Component {
  static propTypes = {
    match: PropTypes.object.isRequired,
    location: PropTypes.object.isRequired,
    history: PropTypes.object.isRequired
  }

  render() {
    const { match, location, history } = this.props

    return (
      <div>{location.pathname}</div>
    )
  }
}

export default withRouter(Child)

(Update) V3

You can use withRouter HOC in order to inject router, params, location, routes in your component props.

class Child extends React.Component {

  render() {
    const { router, params, location, routes } = this.props

    return (
      <div>{location.pathname}</div>
    )
  }
}

export default withRouter(Child)

Original answer

If you don't want to use the props, you can use the context as described in React Router documentation

First, you have to set up your childContextTypes and getChildContext

class App extends React.Component{

  getChildContext() {
    return {
      location: this.props.location
    }
  }

  render() {
    return <Child/>;
  }
}

App.childContextTypes = {
    location: React.PropTypes.object
}

Then, you will be able to access to the location object in your child components using the context like this

class Child extends React.Component{

   render() {
     return (
       <div>{this.context.location.pathname}</div>
     )
   }

}

Child.contextTypes = {
    location: React.PropTypes.object
 }

How to execute the start script with Nodemon

In package json:

"scripts": {
  "start": "node index",
  "dev": "nodemon index"
},

"devDependencies": {
  "nodemon": "^2.0.2"
}

And in the terminal for developing:

npm run dev

And for starting the server regular:

npm start

Inserting a text where cursor is using Javascript/jquery

This jQuery plugin gives you a pre-made way of selection/caret manipulation.

jQuery $.ajax request of dataType json will not retrieve data from PHP script

The $.ajax error function takes three arguments, not one:

error: function(xhr, status, thrown)

You need to dump the 2nd and 3rd parameters to find your cause, not the first one.

SimpleXML - I/O warning : failed to load external entity

You can also load the content with cURL, if file_get_contents insn't enabled on your server.

Example:

$ch = curl_init();  

curl_setopt($ch,CURLOPT_URL,"http://feeds.bbci.co.uk/sport/0/football/rss.xml?edition=int");
curl_setopt($ch,CURLOPT_RETURNTRANSFER,true);

$output = curl_exec($ch);

curl_close($ch);

$items = simplexml_load_string($output);

Typescript import/as vs import/require?

import * as express from "express";

This is the suggested way of doing it because it is the standard for JavaScript (ES6/2015) since last year.

In any case, in your tsconfig.json file, you should target the module option to commonjs which is the format supported by nodejs.

MongoDB vs. Cassandra

I saw a presentation on mongodb yesterday. I can definitely say that setup was "simple", as simple as unpacking it and firing it up. Done.

I believe that both mongodb and cassandra will run on virtually any regular linux hardware so you should not find to much barrier in that area.

I think in this case, at the end of the day, it will come down to which do you personally feel more comfortable with and which has a toolset that you prefer. As far as the presentation on mongodb, the presenter indicated that the toolset for mongodb was pretty light and that there werent many (they said any really) tools similar to whats available for MySQL. This was of course their experience so YMMV. One thing that I did like about mongodb was that there seemed to be lots of language support for it (Python, and .NET being the two that I primarily use).

The list of sites using mongodb is pretty impressive, and I know that twitter just switched to using cassandra.

Null vs. False vs. 0 in PHP

I have just wasted 1/2 a day trying to get either a 0, null, false to return from strops!

Here's all I was trying to do, before I found that the logic wasn't flowing in the right direction, seeming that there was a blackhole in php coding:

Concept take a domain name hosted on a server, and make sure it's not root level, OK several different ways to do this, but I chose different due to other php functions/ constructs I have done.

Anyway here was the basis of the cosing:

if (strpos($_SERVER ['SERVER_NAME'], dirBaseNAME ()) 
{ 
    do this 
} else {
    or that
}

{
echo strpos(mydomain.co.uk, mydomain);  

if ( strpos(mydomain, xmas) == null ) 
    {
        echo "\n1 is null"; 
    }

if ( (strpos(mydomain.co.uk, mydomain)) == 0 ) 
    {
        echo "\n2 is 0"; 
    } else {
        echo "\n2 Something is WRONG"; 
    }

if ( (mydomain.co.uk, mydomain)) != 0 ) 
    {
        echo "\n3 is 0"; 
    } else {
        echo "\n3 it is not 0"; 
    }

if ( (mydomain.co.uk, mydomain)) == null ) 
    {
        echo "\n4 is null"; 
    } else {
        echo "\n4 Something is WRONG"; 
    }
}

FINALLY after reading this Topic, I found that this worked!!!

{
if ((mydomain.co.uk, mydomain)) !== false ) 
    {
        echo "\n5 is True"; 
    } else {
        echo "\n5 is False"; 
    }
}

Thanks for this article, I now understand that even though it's Christmas, it may not be Christmas as false, as its also can be a NULL day!

After wasting a day of debugging some simple code, wished I had known this before, as I would have been able to identify the problem, rather than going all over the place trying to get it to work. It didn't work, as False, NULL and 0 are not all the same as True or False or NULL?

error: resource android:attr/fontVariationSettings not found

I removed all the unused plugins in the pubspec.yaml and in the External Libraries to solve the problem.

How to bind to a PasswordBox in MVVM

You can use this XAML:

<PasswordBox>
    <i:Interaction.Triggers>
        <i:EventTrigger EventName="PasswordChanged">
            <i:InvokeCommandAction Command="{Binding RelativeSource={RelativeSource Mode=FindAncestor, AncestorType=PasswordBox}}" CommandParameter="{Binding ElementName=PasswordBox}"/>
        </i:EventTrigger>
    </i:Interaction.Triggers>
</PasswordBox>

And this command execute method:

private void ExecutePasswordChangedCommand(PasswordBox obj)
{ 
   if (obj != null)
     Password = obj.Password;
}

This requires adding the System.Windows.Interactivity assembly to your project and referencing it via xmlns:i="clr-namespace:System.Windows.Interactivity;assembly=System.Windows.Interactivity".

I don't have "Dynamic Web Project" option in Eclipse new Project wizard

Not all of them are required (I think), but after installing the components listed below I got the "Dynamic Web Project" template added to my Eclipse (Indigo). The list is:

  • Eclipse Java EE Developer Tools
  • Eclipse Java Web Developer Tools
  • Eclipse Web Developer Tools
  • Eclipse XML Editors and Tools

You can install those packages by clicking on "Help" > "Install New Software", selecting the repository that corresponds to your Eclipse build (i.e http://download.eclipse.org/releases/indigo for Indigo). The packages are grouped under "Web, XML, Java EE and OSGi Enterprise Development".

Uninitialized Constant MessagesController

Your model is @Messages, change it to @message.

To change it like you should use migration:

def change   rename_table :old_table_name, :new_table_name end 

Of course do not create that file by hand but use rails generator:

rails g migration ChangeMessagesToMessage 

That will generate new file with proper timestamp in name in 'db dir. Then run:

rake db:migrate 

And your app should be fine since then.

Convert to binary and keep leading zeros in Python

You can use zfill:

print str(1).zfill(2) 
print str(10).zfill(2) 
print str(100).zfill(2)

prints:

01
10
100

I like this solution, as it helps not only when outputting the number, but when you need to assign it to a variable... e.g. - x = str(datetime.date.today().month).zfill(2) will return x as '02' for the month of feb.

Does GPS require Internet?

In Android 4

Go to Setting->Location services->

Uncheck Google`s location service.
Check GPS satelites.

For test you can use GPS Test.Please test Outdoor!
Offline maps are available on new version of Google map.

Android Activity as a dialog

You can define this style in values/styles.xml to perform a more former Splash :

   <style name="Theme.UserDialog" parent="android:style/Theme.Dialog">
        <item name="android:windowFrame">@null</item>
        <item name="android:windowIsFloating">true</item>
        <item name="android:windowIsTranslucent">true</item>
        <item name="android:windowNoTitle">true</item>
        <item name="android:background">@android:color/transparent</item>
        <item name="android:windowBackground">@drawable/trans</item>
    </style>

And use it AndroidManifest.xml:

   <activity android:name=".SplashActivity"
          android:configChanges="orientation"
          android:screenOrientation="sensor"
          android:theme="@style/Theme.UserDialog">

How to display an unordered list in two columns?

Here's a possible solution:

Snippet:

_x000D_
_x000D_
ul {_x000D_
  width: 760px;_x000D_
  margin-bottom: 20px;_x000D_
  overflow: hidden;_x000D_
  border-top: 1px solid #ccc;_x000D_
}_x000D_
li {_x000D_
  line-height: 1.5em;_x000D_
  border-bottom: 1px solid #ccc;_x000D_
  float: left;_x000D_
  display: inline;_x000D_
}_x000D_
#double li {_x000D_
  width: 50%;_x000D_
}
_x000D_
<ul id="double">_x000D_
  <li>first</li>_x000D_
  <li>second</li>_x000D_
  <li>third</li>_x000D_
  <li>fourth</li>_x000D_
</ul>
_x000D_
_x000D_
_x000D_

And it is done.
For 3 columns use li width as 33%, for 4 columns use 25% and so on.

Where is jarsigner?

If you are on Mac or Linux, just go to the terminal and type in:

whereis jarsigner

It will give you the location of the jarsigner

Links in <select> dropdown options

Add an onchange event handler and set the pages location to the value

<select id="foo">
    <option value="">Pick a site</option>
    <option value="http://www.google.com">x</option>
    <option value="http://www.yahoo.com">y</option>
</select>

<script>
    document.getElementById("foo").onchange = function() {
        if (this.selectedIndex!==0) {
            window.location.href = this.value;
        }        
    };
</script>

How to reload a page after the OK click on the Alert Page

I may be wrong here but I had the same problem, after spending more time than I'm proud of I realised I had set chrome to block all pop ups and hence kept reloading without showing me the alert box. So close your window and open the page again.

If that doesn't work then you problem might be something deeper because all the solutions already given should work.

How to start a background process in Python?

You probably want the answer to "How to call an external command in Python".

The simplest approach is to use the os.system function, e.g.:

import os
os.system("some_command &")

Basically, whatever you pass to the system function will be executed the same as if you'd passed it to the shell in a script.

Resolving tree conflict

Basically, tree conflicts arise if there is some restructure in the folder structure on the branch. You need to delete the conflict folder and use svn clean once. Hope this solves your conflict.

Get Current Session Value in JavaScript?

i tested this method and it worked for me. hope its useful.

assuming that you have a file named index.php
and when the user logs in, you store the username in a php session; ex. $_SESSION['username'];

you can do something like this in you index.php file

<script type='text/javascript'>
  var userName = "<?php echo $_SESSION['username'] ?>"; //dont forget to place the PHP code block inside the quotation 
</script>

now you can access the variable userId in another script block placed in the index.php file, or even in a .js file linked to the index.php

ex. in the index.js file

$(document).ready(function(){
    alert(userId);   
})

it could be very useful, since it enables us to use the username and other user data stoerd as cookies in ajax queries, for updating database tables and such.

if you dont want to use a javascript variable to contain the info, you can use an input with "type='hidden';

ex. in your index.php file, write:

<?php echo "<input type='hidden' id='username' value='".$_SESSION['username']."'/>";
?>

however, this way the user can see the hidden input if they ask the browser to show the source code of the page. in the source view, the hidden input is visible with its content. you may find that undesireble.

Displaying standard DataTables in MVC

This is not "wrong" at all, it's just not what the cool guys typically do with MVC. As an aside, I wish some of the early demos of ASP.NET MVC didn't try to cram in Linq-to-Sql at the same time. It's pretty awesome and well suited for MVC, sure, but it's not required. There is nothing about MVC that prevents you from using ADO.NET. For example:

Controller action:

public ActionResult Index()
{
    ViewData["Message"] = "Welcome to ASP.NET MVC!";

    DataTable dt = new DataTable("MyTable");
    dt.Columns.Add(new DataColumn("Col1", typeof(string)));
    dt.Columns.Add(new DataColumn("Col2", typeof(string)));
    dt.Columns.Add(new DataColumn("Col3", typeof(string)));

    for (int i = 0; i < 3; i++)
    {
        DataRow row = dt.NewRow();
        row["Col1"] = "col 1, row " + i;
        row["Col2"] = "col 2, row " + i;
        row["Col3"] = "col 3, row " + i;
        dt.Rows.Add(row);
    }

    return View(dt); //passing the DataTable as my Model
}

View: (w/ Model strongly typed as System.Data.DataTable)

<table border="1">
    <thead>
        <tr>
            <%foreach (System.Data.DataColumn col in Model.Columns) { %>
                <th><%=col.Caption %></th>
            <%} %>
        </tr>
    </thead>
    <tbody>
    <% foreach(System.Data.DataRow row in Model.Rows) { %>
        <tr>
            <% foreach (var cell in row.ItemArray) {%>
                <td><%=cell.ToString() %></td>
            <%} %>
        </tr>
    <%} %>         
    </tbody>
</table>

Now, I'm violating a whole lot of principles and "best-practices" of ASP.NET MVC here, so please understand this is just a simple demonstration. The code creating the DataTable should reside somewhere outside of the controller, and the code in the View might be better isolated to a partial, or html helper, to name a few ways you should do things.

You absolutely are supposed to pass objects to the View, if the view is supposed to present them. (Separation of concerns dictates the view shouldn't be responsible for creating them.) In this case I passed the DataTable as the actual view Model, but you could just as well have put it in ViewData collection. Alternatively you might make a specific IndexViewModel class that contains the DataTable and other objects, such as the welcome message.

I hope this helps!

How to get the latest record in each group using GROUP BY?

You should find out last timestamp values in each group (subquery), and then join this subquery to the table -

SELECT t1.* FROM messages t1
  JOIN (SELECT from_id, MAX(timestamp) timestamp FROM messages GROUP BY from_id) t2
    ON t1.from_id = t2.from_id AND t1.timestamp = t2.timestamp;

How to convert number of minutes to hh:mm format in TSQL?

I'm not sure these are the best options but they'll definitely get the job done:

declare @durations table
(
    Duration int
)

Insert into @durations(Duration)
values(60),(80),(90),(150),(180),(1000)

--Option 1 - Manually concatenate the values together
select right('0' + convert(varchar,Duration / 60),2) + ':' + right('0' + convert(varchar,Duration % 60),2)
from @Durations

--Option 2 - Make use of the time variable available since SQL Server 2008
select left(convert(time,DATEADD(minute,Duration,0)),5)
from @durations

GO

SQL variable to hold list of integers

In the end i came to the conclusion that without modifying how the query works i could not store the values in variables. I used SQL profiler to catch the values and then hard coded them into the query to see how it worked. There were 18 of these integer arrays and some had over 30 elements in them.

I think that there is a need for MS/SQL to introduce some aditional datatypes into the language. Arrays are quite common and i don't see why you couldn't use them in a stored proc.

substring index range

The substring starts at, and includes the character at the location of the first number given and goes to, but does not include the character at the last number given.

Negation in Python

try instead:

if not os.path.exists(pathName):
    do this

Declaring variables inside or outside of a loop

I think the size of the object matters as well. In one of my projects, we had declared and initialized a large two dimensional array that was making the application throw an out-of-memory exception. We moved the declaration out of the loop instead and cleared the array at the start of every iteration.